You are on page 1of 514

Tutorial-01, B.Tech.

Semester-I
(Leibnitzs rule, Partial Differentiation)
Problem 1: Find the th derivative of the following functions:
(a) tan (b) sin 4 cos 6 Ans: (a)(1) ( 1)! . sin
( ) ( )
(b) (10 + 10) (2 + tan 10)
Problem 2: If = ( log ), prove that = +( 1)! and hence show that
= ! {log +1+ + + + }
Problem 3: If = ( 1) , use Leibnitzs theorem to show that
(1 ) 2 + ( + 1) = 0.
Problem 4: If = sin( sin ), show that
(1 ) (2 + 1) +( ) = 0 and hence evaluate ( ) .
Problem 5: If cos = log , prove that + (2 + 1) +2 = 0.
Problem 6: If = [ ( 1)] ,
prove that (1 ) (2 + 1) +( ) = 0.
Problem 7:(a) If = establish the relation that = .
( ) /
(b) If = + + , show that . + . + = and +
+ = 0.
(c) If = ( ) + ( + ) , show that = 0.
(d) If = sin + tan , show that . + . = 0.

(e) If = , then show that + + = 0.


1
1 1
(f) If = , show that = (1 + 3 + ) .
(g) If =log( + + 3 ), then prove that
+ + = ( )
.
Problem 8:(a) If = ( ) and r= + , then + = "( ) + ( ).
(b) If = , show that at = = , = ( )
.

(c) If + + = 1 prove that


( ) + +( ) = 2( . + . + . )
Problem 9:(a) If = , find the value of n will make ( )= Ans: =
(b) If = + , where = = sin find
cos , ?
Ans:3( cos sin )
(c) If = ( , ), where = cos , = sin find the value of .
Then prove that ( ) + =( ) + ( ) .
(d) If + = , find the value of . Ans:
(e) If = ( , , ) , then prove that + + = 0.

1
Problem 10: State and prove Eulers theorem for a homogeneous function ( , ) of degree
in two variables x and y. Also deduce that
(i) . + . = ( 1)
(ii) . + . = ( 1)
(iii) . + . +2 = ( 1)
Problem 11: Prove the following results:

u Result
( / ) . + . = 0.
+ . + . = sin 2
tan
+
+ . + . = tan
sin
+
+ . + . = tan
sin
+
+ . + . =3
log
+
x sin . + . +2 =0

. + . +2 =2
+
+ . + . + cot =0
cos
+

Problem 12: If = , Establish the relation


(1 ) (2 + 1) ( + ) = 0 and hence evaluate ( )

Problem 13: If = sin + cos prove that =m {1 + (1) sin 2mx}

Problem 14: If = [ + ( + 1)] ,prove that (1 + ) + = 0 and hence


evaluate ( ) .

2
Tutorial-02, B.Tech Semester-I
(Expansions of functions of several variables and Curve Tracing)
Problem 1: Find the equation of the tangent plane and the normal to the surface
= 4(1 + + ) at (2,2,6) Ans: 4 + 4 3 = 2 ; = =
Problem 2: Expand e cos near the point (1, ) by Taylors theorem.
( )
Ans: 1 + ( 1) + ( 1) +

Problem 3: Obtain Taylors expansion of tan about (1,1) upto and including the second
degree terms. Hence compute (1.1,0.9).
Ans: tan = ( 1) + ( 1) + ( 1) ( 1) + and 0.7862
Problem 4: Expand x in powers of ( 1) and ( 1)upto the third degree terms.
Ans: x =1+( 1)+( 1)( 1) + ( 1) ( 1)+
Problem 5: Expand e sin in powers of and upto the third degree terms.
( )
Ans: + + !
+
Problem 6: Trace the following curves
1. a y = x ; Cubical parabola 2. = 4 (2 );
3. (2 ) = Cissoid 4. + =5
5. = Catenary 6. ( 1)=( + 1)
7. ( + 4 ) = 8 8. = (2 )
9. x + y = a or = = Astroid
10.. = ( ), = (1 ) Cycloid 11. = + , >
12. = (1 + ) Cardiod 13. = 3 Three leaves rose
14. = (1 ) Cardiod 15. = 2 Four leaves rose
16. = ( + ), = (1 + ) Cycloid 17. = (2 )
18. = + , < Limecon 19. = 2
20. (1 )=2 21.. (1 + )=2
22. 2 = Hyperbola 23.y = x Semi- cubical parabola

24. + = 1 or = = Hypocycloid
25. = ( 1)( 2)( 3) 26. = cos + , = sin t { Tractrix }
27. = ; = 28. = ; = 29. = , =
30. = ( + ), = (1 + ) Cycloid 31. = ( ), = (1 + ) Cycloid
32. = ( + ), = (1 ) Cycloid

3
Tutorial-3 B.Tech. Semester-I
(Double integrals and their applications)
1. Find the area of the loop of the curve + = . Also find the area bounded between the
curve and its asymptote. Ans: and .
2. Evaluate (x + y ) dxdy over the area A enclosed by the curves = 4 , + = 3, =
0 and = 2. Ans:
3. Evaluate the following double integrals
(i) (ii) (iii)

(iv) ( + ) (v)

Ans: (i) .(ii) (iii) ( + ) (iv)-1 (v)

4. Show that ( + ) = ( + )
5. Show that =
6. Show that =
{ }{ } { }{ }
7. Show that ( )
( )
, also find the values of two integrals.
Ans: . . = , . . = Give a conclusion on the basis of the results in Q. 4 to 7.
8. Evaluate ( ) over the region x + y 1. Ans:
9. Evaluate ( ) over the region A bounded by the curves = , = , and +
= . Ans:
10. Evaluate r ddr over the area of the circle = Ans:

11. Find by double integration the area lying inside the circle = and outside the
( )
parabola (1 + )= Ans:

12. Change the order of the following double integrations:

(i) Ans:

(ii) ( , ) Ans: ( , )

(iii) ( , ) Ans: ( , ) + ( , )

(iv) ( , ) Ans: ( , ) + ( , )

4
(v) Ans: +

13. Express as single integral and evaluate:


+

Ans:
;

14. Convert into polar co-ordinates Ans:


15. Using transformation + = , = show that =
16. Using transformation = , + = show that = Where R is
the region bounded by = , = , + = .
17. Find the whole area of the curve = ( ) by double integration. Ans: a
18. Find the area enclosed by the curve = + by double integration. Ans: 11
19. Find the volume of the torus generated by revolving the circle + = about the line
= 3. Ans: 24
20. Find the Center of gravity of the area bounded by the parabola = and the line
+ = . Ans: ( , )

21. Find the Center of gravity of the loop of the curve = . Ans:( , 0)
22. Find the Center of gravity of an arc of the curve = ( + ),
= ( ) in the positive quadrant. Ans: [ ( , ]

5
6
Tutorial-05, B. Tech Sem-I
Jacobians
( , . , )
1. If = + + + , = + , = , = + show that ( , , ,)
=

0 and hence find a relation between , , and . Ans: = +2


2. Prove that the following functions are not independent. Find the relation between them
(i) = + + , = + + , = + + 3 . Ans: u = 3uv + w
(ii) = + + , = + + , = + + . Ans: v = u + 2w
( , )
3. If , , are connected by a functional relation ( , , ) = 0, show that = .
( , ) .

4. If , , are the roots of the equation in , + + = , prove that


( , , ) ( )( )( )
( , , )
=( )( )( )
.

5. If = ( ) , = ( ) , = ( ) , where = + + show
( , . )
that =( ) .
( , , )
( , . ) ( )( )( )( )
6. If = , = + + , = + + , show that ( , , )
= ( )
.

7. Find the Jacobian of , , ,, . . , being given = ( ), = ( ) ,..,


= ( ), = find J( , ,., )
{Hint: + + , + ..+ = , Ans: }.
( , , )
8. Find the Jacobian being given = cos , = sin 1 ,
( , . )
= sin (1 ) where m + n = 1.
( )
{Hint: = + + , Ans: }.

9. Prove that J =1
( , , )
10. If = , = + , = + + , Find Ans:
( , . ) ( )
11. If (0) = 0 and f (x) = , prove without using the method of integration, that ( )+ ( )
=f( ). {Hint: Let = ( ) + ( ) and = the ind J(u, v)}

7
Tutorial-6, B. Tech. Sem-I,
(Triple integrals and their application)
Problem 1: Evaluate the triple integrals Ans:

Problem 2: Evaluate( + + ) over the region bounded by ,

, , + + . Ans:
Problem 3: Evaluate the following triple integrals

(i) ( + + ) (ii)

(iii)


(iv) (v) ( + )
( )
Ans: (i) .(ii) (iii) ( ) (iv)8 (v)

Problem 4: Evaluate the triple integral ( + ) , where R is the region

determined by 0 , 0, 0 + . Ans:

Problem 5: Find the volume of the region bounded by the surface = , = and the

planes = , = . Ans:
Problem 6: Find the volume of the tetrahedron bounded by the co-ordinate planes and the
( ) ( )
plane + + = . { Hint: } Ans:

Problem 7: Find the volume common to the cylinder x +y = a and x +z = a .

( ) ( )
{ Hint: ( ) ( )
} Ans:
Problem 8: Find the mass of the tetrahedron bounded by the co-ordinate planes and the plane

+ + = the variable density =


( ) ( )
{ Hint: } Ans: ( )
Problem 9: Find the moment of inertia of the solid about its major axes generated by revolving

the ellipse = about minor axes.

8
Tutorial-07, B.Tech. Sem-I
(Matrices)
1 0 0
1. Find rank of 3 1 0 Ans: 3.
0 5 2
1 1 2
1 2 3 ( ) = 2.
2. Find the rank of the matrices (i) A= (ii) B= 1 2 3 Ans: (i)
3 1 2
0 1 1
(ii) ( ) = 2.
3. Each entry of a matrix is unity. Show that its rank is one.
4 2 3
4. Determine the rank of the matrix (i) = 8 4 6 Ans: ( )=1
2 1 3/2
1 5 4 1 1 1
5. If A= 0 3 2 , B= 2 2 2 , Find ( ), ( ), ( + ), ( ) and ( ).
2 3 10 3 3 3
Ans: ( ) = 2, ( ) = 1, ( ) = 1and , ( ) = 1
Hint: ( ) ( )
1 1 1 1
6. Find the rank of the matrix = 1 1 2 1 Ans: ( )=
3 1 0 1
1 2 3 2
7. Find the rank of the matrix 2 3 5 1 Ans: =
1 3 4 5
3 2 0 1 7
0 2 2 1 5
8. Find the rank of the matrix Ans: = .
1 2 3 2 1
0 1 2 1 6
1
1 1 0
4
4 3 1
9. Determine the value of b if the rank of 3, where =
2 2 2
9 9 3
{Hint: After using Row transformations
1 1 1 0
~ 0 0 1 1
R 3R 0 0 +6 0
2 0 0 2
(1) If b=2, Then | | = 0 , ( ) = 3.
(2) If = 6 Then, number of non-zero rows is 3. Therefore ( ) = 3.
1 2 1 4
2 4 3 4
10. Reduce the matrix A to its Normal form when A= Hence, find the
1 2 3 4
1 2 6 7
rank of the matrix? Ans: 3.
1 1 1
11. Find non-singular matrices , so that is a normal form where = 1 1 1
3 1 1
Also find ( ) . Ans: 2
12. Find the rank of the followings matrices:

9
1 2 1 2
3 0 2 2
1 3 2 2
(i) . 3 (ii) 6 42 24 54 .2
2 4 3 4
21 21 0 15
3 7 4 6
2 2 0 6 2 2 0 6
4 2 0 2 4 2 0 2
(iii) .3 (iv) .3
1 1 0 3 1 1 0 3
1 2 1 2 1 2 1 2
0 4 12 8 9
1 2 5
(v) 0 2 6 2 5 : (vi) 4 1 6 .
0 1 3 6 4
6 3 4
0 8 24 3 1
1 2 3 4 1 1 2
(vii) 3 4 1 2 : (ix) 1 2 2 .3
4 3 1 2 2 2 3
3 2 5 7 12
(x) 1 1 2 3 5 .2
3 3 6 9 15
1
0
13. The rank of the diagonal matrix 1 is (a)2 , (b)1 , (c)4, (d*)3 .
0
4
0
14. If A is a non-zero column vector ( 1) then rank of (A)2, (B)n, (C)0, (d*)1 .
1 0
15.The rank of the matrix 0 1 is 2, for is equal to (a) Arbitrary (b*) 1, (c) 2,
1 0
(d)3.
2 0 0 1 2 3
16. If A= 0 2 0 and B= 0 1 3 , the value of |AB| is (a) 4, (b)8,(c*)16 ,(d)32
0 0 0 0 0 2
x+a b c d
a x+b c d
17. The determinate of is (a*) x (x + a + b + c + d),
a b x+c d
a b c x+d
(b)(x+a)(x+b)(x+c)(x+d),(c)x + ax + bx + cx + d (d) None of the above.
x x 1+x
18. If y y 1 + y =0, then =(a*) -1, (b)1, (c) 0, (d) 3
z z 1+z
19. If the system of equations + 4 + = 0, + 3 + = 0, + 2 + = 0 has a
non-trivial solution, then , , , are in (a) A.P, (b)G.P, (c*) H.P., (d) None of these.
1 log y log z
20. If , , , then log x 1 log z is (a) 1, (b*)0,(c)-1 ,(D) 2
log x log y 1
a x a a a a a
21. If b b x b =0, for = 1,2,3 then the value of b b b (a) -6,
c c c x c c c
(b*)6, (c) -4, (d) 4

10
1 2 3
22. If A= 1 3 4 then |AdjA| is (a) -4, (b*)4,(c)-2 ,(d)2
1 4 3
23. If A be a square matrix such that A,A , A are non-zero matrices but A is a zero
matrix.Then (I A) is (a) A+ A + A (b)I+A+ A (c) I+ A + A , (D*)I+A+ A + A
1 2 3
24. The rank of the matrix 2 4 7 is (a) 1, (b*)2, (c)3 ,(d) None of these.
3 6 10
25. If (A )=2 and (B )=3 then (AB)= (a) 5, (b)3,(c*)2 ,(d) 1
26. If the system of equations 3 + = 1, 2 + + = 2, + 2 = 1 has a
unique solution if = (a) any value, (b) , (c*) , (d)
27. If A and B are square matrices of the same order, which of the following is true
(a)( + ) = +2 + ,( )( + )( ) = ,( ) ( ) ( + ) =
,( ) ( + )( ) + ( ) ( + ) = 2 2
28. Prove that the eigen values of a triangular matrix are just diagonal elements of the matrix.
29. Prove that all the eigen values of a Hermitian matrix are real.
30. Find all the eigen values and eigenvectors of the matrices
3 1 4 1 1 3
(i) A= 0 2 6 . Ans: 2,3,5 ; 1 , 0 , 2 (UPTU SE 2002,04)
0 0 5 0 0 1
2 2 3 c c
(ii) B= 2 1 6 Ans: -3,-3,5 ; d , 2c , (Utt.TU 2006)
1 2 0 c
1 2 2
(iii)C= 0 2 1 . Ans: (UPTU 2006)
1 2 2
31. Verify Cayley-Hamilton theorem for the following matrices and hence find A
4 3 1 5 1 7
(i) A= 2 1 2 A = 4 3 10 (UPTU 2001)
1 2 1 3 5 2
2 1 1 3 1 1
(ii) A= 1 2 1 A = 1 3 1 (UPTU 2003,04,05)
1 1 2 1 1 3
1 2 2
32. A square matrix is defined by A= 1 2 1 , Is it diagonalizable ? If yes, reduce it
1 1 0
into diagonal form. Ans: Diag(1, 1,3)
cos sin cos n sin n
33. If A= , show that A = , where n
sin cos sin n cos n
1 0 0 0
1 1 3
1 1 0 0
34. Prove (i) 5 2 6 is a nilpotent matrix of index 3; (ii). is an
1 2 1 0
2 1 3
1 3 3 1
cos sin 1 1+
involutory matrix (iii) is an orthogonal matrix.; (iv) is
sin cos 1 1
a unitary matrix.
| |
35. Prove that if s an eigen value of a matrix A, then k , , , , are the eigen values
of kA, A , , , Adj A respectively.

11
36. If A has all entries 1 then rank of A is (a) 7; (b*) 1; (c) 5; (d) 0.
37. The rank of a matrix where < , can not be more than (a*) m; (b) n; (c) mn; (d)
None.
1 0 1
38. If R= 2 1 1 then top row of R is (a) [5 6 4]; (b*) [5 3 1]; (c)
2 3 2
[2 0 1]; (d) [2 1 1/2]
5 0 2 1 0 1 0 1 1 2 2 2
39. If A= 0 3 0 then A is (a) 0 0 0 ; (b) 1 1 1 ; (c) 2 2 2 ;
2 0 1 0 1 1 1 0 1 10 2 2
1/2 1/2 1/2
(d*) 1/2 1/2 1/2
0 0 1
40. Check the consistency for the following linear systems. If system is consistence find the
+ +2 + = 5 2 + 6 + 11 = 0 +2 = 3
2 +3 2 =2 6 + 20 6 = 3 3 +2 =1
4x1+5 +3 = 7, 6 18 + 1 = 0 2 2 +3 = 2
Ans: No solution Ans: No solution + = 1
Ans: = , = , =
+ + =6 2 +5 +3 = 1 7 + 2 + 3 = 16
2 +3 2 = 2 +2 + =2 2 + 11 + 5 = 25
5 + +2 =0 + + =0 + 3 + 4 = 13
Ans: = , = , = Ans: = , = ; = Ans: = , = , =

41. For what values of and so that the equations + + = 6; + 2 + 3 = 10;


+2 + = have (i) No solution; (ii) a unique solution and (iii) an infinite number
of solutions ANS: (i) = 3 no solution; (ii) 3, 10 , unique solution;
(iii) = 3, = 10 , In inite many solutions.
42. Determine b such that the system 2 + + 2 = 0; + + 3 = 0; 4 + 3 + =0
has (i) trivial solution(ii) non- trivial solution.
43. Find the values of a , b and c for which the system has (i) No solution (ii) unique
solution (iii) Infinitely many solutions for
2 + + = ; + + =3
2 + = ; +2 +2 =
+ 2 = +5 +3 =9
Ans: ( ) + + 0 Ans: (i) = 1, 6
( ) + + = 0,Infinite solutions (ii) 1, ,
(iii) = 1, = 6 solutions.
44. Solve the following system of homogeneous equations

+ 3 2 = 0; 4 + 2 + + 3 = 0;
2 + 4 = 0; 6 + 3 + 4 + 7 = 0;
11 + 14 = 0 2 + + =0
Ans: = , = , Ans: = , = ,
= 2 , =
= ( ).

12
Tutorial-8, B.Tech Sem-I
(Vector Calculus)
Note: In this exercise bold face letters (say) F represents vector f and i,j,k represents unit vectors
,,k respectively.
Problem1: Evaluate the following (i) (r ) (ii) (r ( )) (iii) ((r ))(iv) grad Div( )
Ans: (i)6r , (ii)3r , (iii)2r , (iv)
Problem2:If A= 2yz i x y j + xz k , B= x i + yz j xy k and =2x yz , then find
(i) ( ) ; Ans:
(ii) ;
(iii) ( ) ;
(iv) ( ) ;
(v) ( )
Problem3: If A and B are differentiable vector functions, and are differentiable scalar
functions of position ( , , ), then prove the following results:
(i) ( + ) = +
(ii) ( )= ( ) +
(iii) ( )= ( ) +
(iv) ( ) = ( ) ( )
(v) ( ) =( ) +( ) + ( )+ ( )
(vi) ( ) =0
Problem4: Evaluate the grad. of log| | Ans:
Problem5: Show that is a vector perpendicular to the surface ( , , )= constt.
Problem6: Find the directional derivative of ( , , )=x yz + 4xz at (1, 2, 1) in the
direction 2 2 . Ans:37/3
Problem7:Prove that the vector A= 3y z i + 4x z j 3x y k is solenoidal.
Problem8: Prove following identities
(i) Div()=0
(ii) If A and B are irrotational then AB is solenoidal.
(iii) ( )=0
(iv) Div ( f g) = f g+f g
( )( )
(v) b(a ) = where a and b are constt. Vectors.
(vi) ( )=U
(vii) =
Problem9: (a) Prove that = ( + ) + (2 4) + (3 + 2) is a
conservative force field.
(b)Find the scalar potential for F. Ans: ( + 4 +2 + )
(c)Find the work done in moving an object in this field from (0,1, 1) to ( , 1,2).
Ans: 15+4
Problem10: Show that V=2xyz + ( +2 ) + is irrotational. Express V as gradient
of a scalar function .
( , )
Problem11: Evaluate ( , ) (10 x 2xy )dx 3x y dy along the path x 6xy = 4y
Ans: 60. {Hint: Use Exact
differential}
Problem12: Use Greens theorem to evaluate ( )dx + sinx cosy dy ,where

13
C + = . Ans: 0 .
Problem13: Verify Greens theorem in the plane for (xy + y ) dx + x dy ,where C is the
closed curve of the region bdd. by the line = and curve = x .
Problem14: Find the work done in moving a particle once around a circle C in the xy-plane, if
circle has center at the origin and radius 3, Force field is given by = (2 +
) + ( + ) + (3 2 + 4 ) Ans: 18
Problem15: State and prove Greens theorem.
Problem16: Prove that the area bounded by a simple closed curve C is given by
xdy ydx{Hint: Use Greens theorem }
Problem16: Find the constants a,b,c such that V= ( + 2 + ) + ( 3 ) + (4 +
+ 2 ) is irrotational. Express V as gradient of a scalar function .
Problem18: State Greens theorem and hence evaluate (cosy)dx + (x xsiny) dy ,where C
is the closed curve + = .
Problem19: Use Greens theorem to evaluate ( + )dx + ( x + y ) dy ,where C is the
square formed by the lines = 1, = 1.
Problem20: Use Stokes theorem to evaluate ( + 2 )dx + (x z) dy + (y z)dz ,where C
is the boundary of the with the vertices (2,0,0),(0,3,0),(0,0,6) oriented in the
anticlockwise direction.
Problem21: Verify Stokes theorem for F= x y k y j + xzi and S is the surface of the region
bounded by = 0, = 0, = 0,2 + + 2 = 8, which is not included in the xz-
plane .
Problem22: Evaluate ( ) n ds ,where F= y i + (x 2xz) j xy k and S is the
surface of the sphere + + = above the xy-plane.
Ans: Zero.
Problem23: Evaluate n ds ,where F= 4xz i y j + yz k and S is the surface of the
cube bounded by = 0, = 1, = 0, = 1, = 0, = 1.
Ans: .
Problem24: If F= (x + y 4)i + 3xy j + (2xz + z ) k ,evaluate ( ) n ds , and S
is the surface of the sphere + + = above the xy-plane.
Ans: 16
Problem25: Evaluate (y z i + x z j + x y k) n ds ,where S is the part of the surface of
the sphere + + = above xy-plane.
Problem26: If V is the volume enclosed by the surface S, Find the value of n ds
Ans: 3V
Problem27: State and prove Gauss Divergence theorm.
Problem28: Evaluate (ax i + by j + cz k) n ds ,where S is the surface of the sphere
+ + = . [P.U. 2004]
Problem29: Evaluate n ds ,where A= 18z i 12 j + 3y k and S is the part of the
plane 2 + 3 + 6 = 12 which is located in the first octant. Ans: 24

14
Problem30: Evaluate n ds ,where A= z i + x j 3y z k and S is the surface of the
cylinder + = included in the first octant between = 0 and = 5.
Ans: 90

Problem 31: Prove that (i) ds= ; (ii) ds= 5r dv
(iii) ( ) n ds =0 for any closed surface S.
Problem 32: Use divergence theorem to evaluate ds ,where A= 4x i 2y j + z k and
S is the surface of the cylinder + = bdd. between = 0 and = 3.
Problem 33: Verify Stokes theorem for F= (x + y ) i 2xy j taken around rectangle bdd. by
the lines = ; = 0, .

15
Tutorial1, B. Tech. Sem III, 24 July, 2016

(Root Findings and Linear System of Equations)

1. Find the root of the equation ex = 3x lying in [0,1] correct to three decimal places using
Bisection, Regula-Falsi and Newton-Raphson methods.
2. Use the interval halving method to improve a root of the equation x4 + 2x3 x 1 = 0
lying in [0,1] correct to three decimal places.
3. For smallest positive root of the equation: x3 5x + 1 = 0 correct to 3 D places. Use the

I
methods Bisection, Regula-Falsi and Newton-Raphson methods.

ON
4. Find positive root of the equation: tan x + tanh x = 0 by using Bisection and Regula-Falsi
methods correct to 4 D places.
5. Solve the following system of equations up to 2D places by Gauss Seidel
i . 20x + y 2z = 17; 3x + 20y z = 18; 2x 3y + 20z = 25 with Initial root (0,0,0);
Ans: x = 1, y = 1, z = 1
ii . 10x + 2y + z = 9; x + 10y z = 22; 2x + 3y + 10z = 2 with Initial root (0,0,0).
Ans: x = 1, y = 2, z = 3
S
iii . 11x1 7x2 + x3 = 32; x1 + 5x2 2x3 = 18; 2x1 + 2x2 + 7x3 = 19; upto 1D places
with Initial root (5,5,5)
iv . x 2y + z = 8; x + y + 2z = 9; 3x y + z = 6 with Initial root (3,3,2); Ans: .......
6. Verify that each of the following equations has a root on the interval (0,1). Next, perform
K.
the bisection method to determine p3 , the third approximation to the location of the root
(a) ln(1 + x) cos x = 0
(b) x5 + 2x 1 = 0
(c) ex x = 0
(d) cos x x = 0
7. It was noted that the function f (x) = x3 + 2x2 3x 1 has a zero on the interval
R.

(3, 2) and another on the interval (-1,0). Approximate both of these zeroes to within
an absolute tolerance of 5 105 .

8. Approximate 3 13 to three decimal places by applying the bisection method to the equa-
tion x3 13 = 0.
9. Approximate 1/37 to five decimal places by applying the bisection method to the equation
1/x 37 = 0.
10. Consider the function g(x) = cos x.
(a) Graphically verify that this function has a unique fixed point on the real line.
(b) Can we prove that the fixed point is unique using the theorems of this section? Why
or why not?

1
11. Consider the function g(x) = 1 + x 18 x3 .
(a) Analytically verify that this function has a unique fixed point on the real line.
(b) Can we prove that the fixed point is unique using the theorems of this section? Why
or why not?
12. Each of the following equations has a root on the interval (0,1). Perform Newtons method
to determine p4 , the fourth approximation to the location of the root.

I
(a) ln(1 + x) cos x = 0

ON
(b) x5 + 2x 1 = 0
(c) ex x = 0
(d) cos x x = 0

13. The equation x3 + x2 3x 3 = 0 has a root on the interval (1,2), namely x = 3.

14. The equation x7 = 3 has a root on the interval (1,2), namely x = 7 3.
15. The equation 1/x 37 = 0 has a zero on the interval (0.01,0.1) namely x = 1/37.
S
16. Show that when Newtons method is applied to the equation x2 a = 0, the resulting
iteration function is g(x) = 12 (x + xa ).
17. Show that when Newtons method is applied to the equation 1/x a = 0, the resulting
iteration function is g(x) = x(2 ax).
K.
18. For each of the functions given below, use Newtons method to approximate all real roots.
Use an absolute tolerance of 106 as a stopping condition.
(a) f (x) = ex + x2 x 4
(b) f (x) = x3 x2 10x + 7
(c) f (x) = 1.05 1.04x + ln x
19. Each of the following equations has a root on the interval (0,1). Perform the secant
R.

method to determine p4 , the fourth approximation to the location of the root.


(a) ln(1 + x) cos x = 0
(b) x5 + 2x 1 = 0
(c) ex x = 0
(d) cos x x = 0
20. For the following i-iii
(a) Using scaled partial pivoting during the factor step, find matrices L, U and P such
that LU = P A.
(b) Solve the system Ax = b for each of the given right-hand-side vectors.

2

1 2 3 4 10 4 2
1 1 5
2 3 , b3 = 3
5
(i) A =
1 1 1 2 , b1 = 3 , b2 = 3

1
1 1 1 5 4 4 8

1 0 2 0 3 1 3
1 4 3 6 6
, b3 = 8
12
(ii) A =
0 2 5 3 , b1 = 0 , b2 = 4

10

I
3 1 1 0 5 3 2

ON
2 7 5 14 4 3
(iii) A = 6 20 10 , b1 = 36 , b2 = 16 , b3 = 12
4 3 0 7 7 6
21. In Exercises 1-3, use the Gauss-Seidel method to solve the indicated linear system of
equations. Take x(0) = 0, and terminate iteration when ||x(k+1) x(0) || falls below
5 106 .
4x1 + x2 + x3 + x4 = 5
x1 + 8x2 + 2x3 + 3x4 = 23
1.
x1 + 2x2 5x3 = 9
S x1
4x1 x2
+ 2x3 + 4x4
= 2
= 4

2. x1 + 4x2 x3 = 4
x2 + 4x3 = 10
K.
7x1 + 3x2 + = 4
3x1 + 9x2 + x3 = 6
3. x2 + 3x3 x4 = 3
x3 + 10x4 4x5 = 7
4x4 + 6x5 = 2
22. Solve the following system of linear equations by Triangularization /Factorization or
Crouts method:
R.

(a) 2x + 3y + z = 9; 3x + y + 2z = 8; x + 2y + 3z = 6; Ans: ....


(b) x + y + z = 3; 2x y + 3z = 16; 3x + y z = 3; Ans: x = 1, y = 2, z = 4
(c) x+2y+3z+4w = 20; 3x2y+8z+4w = 26; 2x+y4z+7w = 10, 4x+2y8z4w =
2; Ans: x = 4, y = 3, z = 2, w = 1

3
Tutorial2, B. Tech. Sem III, 26 July, 2016
( Difference Operators and Interpolation)
1. Prove the following operator relations:
2
(i). E = 1 + = (1 5)1 , (ii) = + 2 ,
(iii). log f (x) = log(1 + f (x)
f (x)
)
(iv). [f (x)g(x)] = f (x)g(x) + g(x + h)f (x),
(v). 45 = 54 =q4 5 = 2 ,

I
2
(vi). = 21 2 + 1 + 4 , (vii). 3 y 1 = y2 3y1 + 3y0 y1 ,
2

ON
(viii). (fk gk ) = fk (gk ) + gk (fk ),
(ix). 2+
1+
= 2515
, (x). = ehD 1, (xi). = sinh(hD),
(xii). ehD = 1 5,
1
(xiii). f [x, y, z] = x + y + z, f = x3 , (xiv). f [a, b, c] = abc , f = a1 ,
n
(xv). f [a, b, c, ...(n symbols)...l] = 4n!hf (a)
n

2. Find for (h = 1)  
2 3 42
(i). 42 (abex ), (ii). 4Exx3 , (iii). 4 tan1 (ax), (iv). 4n ( x1 ), (v). E
x3 ,
(vi). 4(x + cosx).
S x2
 2   x
3. Prove that: ex = 4E ex . 4 E e
2 ex and
x2
f0 + xf1 + f
2! 2
+ ... = ex (f0 + x 4 f0 + 2!
4 f0 + ...)
4. Let Pn (x) = (x x0 )(x x1 )(x x2 ).......(x xn1 ), where xi = x0 + ih, i being integer;
n!
show that 4Pn = nhPn1 , hence show that 4r Pn = (nr)! hr Pnr , r = 1 (1) n 1, and
K.
n n
4 Pn = n!h .
5. Find the third divided difference with argument 2, 4, 9, 10 of the function f (x) = x3 2x.
6. Form a divided difference table for f (x) = x4 + 6x2 + x 2 for values of x = 3 (1) 3,
show that 5th order diferences are zero.
7. Given that f (0) = 8, f (1) = 68, and f (5) = 123 determine f (2); calculate the error also.
8. Tabulate sin x for x0 = 30 (2) 40 and interpolate sin 310 and sin 330 . Compare with exact
R.

values.
9. Tabulate ex for x = 1.7 (0.1) 2.2 and interpolate at x = 1.71, 2.15.
10. Find log10 1152.5 and log10 1161.3 using the following data:
x 1150 1155 1160 1165 1170 1175 1180
log10 (x) 3.06069 3.06258 3.06445 3.06632 3.06818 3.07003 3.07188
and error in the result.
11. Use Lagranges formula to interpolate the values of f (5) from
x 1 2 3 4 7
f (x) 2 4 8 16 128
How much it deviates from 25 .?

4
12. A third degree polynomial passes through the points (0, 1), (1, 1), (2, 1), (3, 2). Find
the polynomial. Ans: 16 x3 12 x2 + 83 x 1
13. From the following data, find the number of students who obtained less than 45 marks:
Marks 30 40 40 50 50 60 60 70 70 80
No. of students 31 42 51 35 31
Ans: approx. 48

I
14. Find the form of f (x), given that f (0) = 8, f (1) = 11, f (4) = 68 and f (5) = 123 also
determine f (2). Ans: x3 x2 + 3x + 8, 18

ON
15. Given that f (0) = 18, f (1) = 0 = f (3) = f (6), f (5) = 248 and f (9) = 13104 find the
form of f (x) assuming it to be a polynomial of degree 5th. {Hint: f (x) = (x 1)(x
3)(x 6)(x) , (x) is a polynomial of degree 2 with (0) = 1, (5) = 31, (9) = 91 }
Ans: x5 9x4 + 18x3 x2 + 9x 18
16. Find log10 301 using Newtons Divided difference interpolation formula from the data:
x 300 304 305 307
Ans: 2.4786
log10 (x) 2.4771 2.4829 2.4843 2.4871

17. The values of y = x are listed below:
x 4
S 6 7
y 2 2.449 2.646 3.162
10

Compute x corresponding to y = 2.5. Ans: 6.25148


18. What should be the minimum number of tabular points required for the piecewise linear
interpolation for f (x) = cos(x) on [0, ], such that error does not exceed by 1/2 106 ?
K.
Ans: The number of subdivisions required n = 1571.


R.

5
Tutorial3, B. Tech. Sem III, 22 August, 2016
(Numerical Differentiation, Integration and Differential Equations)
1. Find the gradient of the road at the initial point of the elevation above a datum line of
seven points of road which are given below:
x: 0 300 600 900 1200 1500 1800
y : 135 149 157 183 201 205 193

I
2. Find the first three derivatives of the function at x = 1.5 from the data
x: 1.5 2.0 2.5 3.0 3.5 4.0
y : 3.375 7.0 13.625 24.0 38.875 59.0

ON
Ans: 4.75, 9.0, 6.0
3. The table given below reveals the velocity v of a body during the time t specified. Find
its acceleration at t = 1.1
t : 1.0 1.1 1.2 1.3 1.4
v : 43.1 47.7 52.1 56.4 60.8
Ans: 44.917
4. Derive Newton Cotes quadrature formula. Hence deduce (i) Trapezoidal rule, One third
Simpsons and 3/8 Simpsons rule of Numerical integrations. (ii) Calculate Truncation
S
error as well as Max. Global error in the rules.
R1 1
5. Evaluate 0 1+x 2 dx, with h = 0.2 (up to 3D) by using Trapezoidal rule of Numerical

integration. Hence find an approximate value of . Give your decision about the statement
Can we apply One third Simpsons and 3/8 Simpsons rule of Numerical integrations for
this problem
K.
Ans: 3.135, No
R1
6. Evaluate 0 ex dx by One third Simpsons correct to 5 D places with proper choice of h.
Ans: 1.71828
R6 1
7. Evaluate 0 1+x 2 dx, up to 3D by using Trapezoidal rule and Simpsons rules of Numerical

integrations. Also check your results by actual integration.


Ans: 1.411, 1.366, 1.357, Actual value is 1.406
R.

8. Construct the divided difference table for the following data set, and then write out the
Newton form of the interpolating polynomial.
x -1 0 1 2
y 3 -1 -3 1
9. Construct the divided difference table for the following data set, and then write out the
Newton form of the interpolating polynomial.
x -7 -5 -4 -1
y 10 5 2 10
10. Write out the Newton form of the interpolating polynomial for f (x) = sin x that passes
through the points (0, sin 0), (/4, sin /4), and (/2, sin /2).

6
11. Apply Picards Method with 4-iterations upto 4D places to find the values of y at x =
dy
0.1(0.3)0.3, given that dx = y x, y(0) = 2. Ans: 2.21, 2.42, 2.65
12. Apply Picards Method with 3-iterations to find the values of y at x = 0.1, 0.2, given that
dy
dx
= y 2 + x2 , y(0) = 0. Ans: 0.00033, 0.00267
13. By using Picards Method with 5-iterations, find y at x = 0.1, 0.2 upto 4D places from
dy
the differential equation dx = x + x2 y, y(0) = 1. Ans: 1.0053, 1.0227

I
14. Write out the Newton form of the interpolating polynomial for f (x) = ex that passes
through the points (1, e1 ), (0, e0 ), and (1, e1 ).

ON
15. Use RungeKutta method of fourth order to find a numerical solution at x = 0.5 for
dy
dx
= 12 (x y), y(0) = 1 taking h = 0.25.
dy
16. Solve the differential Equation dx = x+y +xy, y(0) = 1 by using Taylors series expansion
to get y at x = 0.1(0.1)0.5 (use terms up to x5 in the expansion ) by shifting the origin
for each value.
dy
17. Evaluate y upto 4D places for x = 0.1(0.1)0.5 by using Taylors series method for dx
=
x + y 2 , y(0) = 0 .
Sdy
18. Solve dx = x + y 2 , y(0) = 0 for x = 0.1(0.1)0.5 by using modified Eulers method correct
to 4D places.
dy
19. Solve dx = x + y + xy, y(0) = 1 by using modified Eulers method correct to 4D places to
obtain y at x = 0.1(0.1)0.5
K.
dy
20. Using RungeKutta fourth order method compute y at x = 0.2(0.2)0.6 for dx
= 1+
y 2 , y(0) = 0.9 correct to 3D places.
dy
21. Using fourth order RungeKutta method solve dx = xy, y(0) = 1 in the interval [0,0.6]
by taking h = 0.2 and compare the result with the values obtained from the exact solution.
dy
22. Using Taylors series method up to 5th terms to get y(1.1), y(1.2) for dx
= x2 +y 2 , y(1) = 2.
Ans 2.6384, 3.7080
dy
23. Find y at x = 0.2 by using Eulers method correct to 3D places to solve dx = xy 2 , y(0) =
R.

1 taking h = 0.1. What is the value if modified Eulers method is applied under same
conditions. Ans 0.8, 0.858

7
24. Review: Numerical Integration Formulae: are based on Polynomial Interpola-
tion

x x0 x1 x2 ... xn
f (x) f (x0 ) f (x1 ) f (x3 ) ... f (xn )
Types of Newton-Cotes Formulae:

I
1. Trapezoidal Rule (Two point formula)

ON
Z b
h
f (x)dx = [f (x0 ) + 2 (f (x1 ) + f (x2 ) f (xn1 )) + f (xn )]
a 2

Error in composite Trapezoidal rule Error in the ith interval [xi1 , xi ] is

(xi xi1 )3 00 h3 00
E CT (I) = f (i ) = f (i )
12 12
where i (xi1 , xi )).
Hence, the Max. error in composite rule (Global Error) is
S E CT
=
h3 X 00
n
f (i )
12 i=1
or
(b a)h2 00
K.
= f (),
12
where a < < b,
2. Simpsons 1/3 Rule (Three Point formula);

Then
Z b Z x2 Z x4 Z x2i Z x2n
f (x)dx = f (x)dx + f (x)dx + + f (x)dx + + f (x)dx
R.

a x0 x2 x2i2 x2n2

h
= {f (x0 ) + 4 [f (x1 ) + f (x3 ) + f (x5 ) + + f (x2n1 )]+
3
+2 [f (x2 ) + f (x4 ) + f (x6 ) + + f (x2n2 )] + f (x2n )}
1
The Error in the Composite Simpsons 3
Rule i.e. error in the ith interval (x2i2 , x2i ) is

h5 (4)
E cs (I) =
f (i ), i (x2i2 , x2i )
90
Max. Error in composite Simpsons 1/3-rule is given by

h4 (4)
E CS = (b a) f (),
180
8
where [a, b]
3
3. Composite Simpsons 8
rule or (Four point formula);
ba 3
The [a, b] is divided into 3n equal subintervals. (h = 3n
.) and we apply 8
rule on each
of the n intervals [x3i3 , x3i ] for i = 1, 2, 3, , n.
Hence, Z b Z x3 Z x6 Z x3n =b
f (x)dx ' f (x)dx + f (x)dx + + f (x)dx

I
a x0 =a x3 x3n3

3h

ON
[f0 + 3f1 + 3f2 + 2f3 + 3f4 + 3f5 + 2f6 + 3f7 + + 3f3n1 + f3n ]
=
8
Remember:
f with suffices of multiple 3 are multiplied by 2. Others by 3, except the end points.
T. Error
3h5 (4)
Es = f (i ), i (x3i3 , x3i )
80
3h4 (4)
G. Error E s = 80
f () where [a, b]
S
K.
R.

9
Tutorial4, B. Tech. Sem III, 5 Sep., 2016
(Introduction of Complex numbers & Analytic functions)
1. Find the locus of z in each of the following relations: (i)|z 5| = 6, (ii)|z + 2i| 1,
(iii)Re(z + 2) = 1, (iv)|z i| = |z + i|, (v)|z + 3| + |z + 1| = 4, (vi)1 |z 3| 2, (vii)
|z + 3| |z + 1| = 1.
2. For which complex number following are true, justify in each (i)z = z, (ii)z = z 1 ,

I
(iii) z = z 1 , (iv)z = z
3. Define an analytic function at a point and in a domain.

ON
4. Prove that an analytic function of constant modulus is always constant.
5. Prove that Real and imaginary parts of an analytic function are harmonic.
6. Prove that an analytic function is always continuous but converse need not be true. Give
an example.
7. State and prove the necessary and sufficient condition for a function f (z) = u + iv to be
analytic.
8. Define an analytic function at a point. Illustrate such a function.
S
9. If f (z) = (z)2
z
,z 6= 0; f (0) = 0 then f (z) satisfies CauchyRiemann equations (CR) at
origin.
10. Using MilneThomson method construct an analytic function f (z) = u + iv for which
2u + 3v = 13(x2 y 2 ) + 2x + 3y.
K.
11. State and Prove CauchyRiemann equations in polar coordinate system.
p
12. Let f (x, y) = (|xy|), then (a) fx , fy do not exist at (0, 0); (b) fx (0, 0) = 1; (c) fy (0, 0) =
0; (d) f is differentiable at (0, 0). Ans: c
 2
13. If function f (z) = zz , when z 6= 0, f (z) = 0 for z = 0, Then f (z) (a) satisfies C.R.
equations at z = 0; (b) is not continuous at z = 0; (c) is differentiable at z = 0; (d) is
analytic at z = 1 Ans: b
R.

2
14. Show that f (z) = (z)z , when z 6= 0, f (z) = 0 for z = 0, satisfies C.R. equations at z = 0;
but is not differentiable at z = 0.
15. The harmonic conjugate of u = x2 y 2 + xy is (a) x2 y 2 xy; (b) x2 + y 2 xy; (c)
1/2(x2 + y 2 ) + 2xy. (d) 2(x2 + y 2 ) + 1/2 Ans: c
16. f (z) = (|z|)2 is (a) continuous everywhere but nowhere differentiable; (b) continuous at
z = 0 but differentiable everywhere; (c) continuous nowhere; (d) none of these. Ans: d
x2 y 5 (x+iy)
17. Examine the nature of the function f (z) = x4 +y 10
; if z 6= 0 , otherwise 0, in a region
including the origin.

10
3 3
18. Prove that the function f (z) = u + iv, where f (z) = x (1+i)y
x2 +y 2
(1i)
; if z 6= 0 , otherwise
0
0 is continuous and that CauchyRiemann equations are satisfied at the origin, yet f (z)
does not exists there.
3
19. If f (z) = x xy(yix)
6 +y 2 ; if z 6= 0 and f (0) = 0, show that f (z)f (0)
z
0 as z 0 along any
radius vector but not as z 0 in any manner.
2
20. If f (z) = xyx2(x+iy)
+y 4
; if z 6= 0 and f (0) = 0, prove that f (z)f (0)
z
0 as z 0 along any

I
radius vector but not as z 0 in any manner.
21. The function f (z) = z is (a) analytic at z = 0, (b) differentiable only at z = 0;(c) satisfies

ON
C.R. equations everywhere; (d) nowhere analytic. Ans: d
22. Derive the C.R. equations for an analytic function f (r, ) = u(r, ) + i v(r, ) and deduce
that urr + 1r ur + r12 u = 0.
23. Find the point where the C.R. equations are satisfied for the function f (z) = xy 2 + ix2 y.
0
In which region f (z) exists?
24. f (z) = (|z|)2 is (a) continuous everywhere but nowhere differentiable except at 0; (b)
continuous at z = 0 but differentiable everywhere (c) continuous nowhere; (d) none of
S
these. Ans: a
25. Prove that the function f (z) = z|z| is nowhere analytic.
0
26. If f (z) is an analytic function such that Ref (z) = 3x2 4y 3y 2 and f (1 + i) = 0 then
f (z) is (a) z 3 + 6 2i, (b) z 3 + 2iz 2 + 6 2i, (c) z 3 + 2iz 2 2i, (d) z 3 + 2z 2 + 6 2i Ans:
K.
b
Hint: ux = 3x2 4y 3y 2 = 1 (x, y)(say), integrating partially w.r.t. y we get u =
x3 4xy 3xy 2 + g(y)
Therefore, uy = 4x 6xy + g 0 (y)
or uy = vx = 2 (x, y)(say) = 4x + 6xy g 0 (y)
Thus 1 (z, 0) = 3z 2 , 2 (z, 0) = 4z + g 0 (0)
Now, applying
R Milne Thomson we get
2
f (z) = (3z + i 4z)dz + constt.
or f (z) = z 3 + 2i z 2 + constt. and applying f (1 + i) = 0 implies Constt = 6 2i
R.

27. The orthogonal trajectory of u = ex (x cos y y sin y) is (a) ex (cos y + x sin y) + c; (b)
ex x sin y + c ; (d) ex (y cos y + x sin y) + c. Ans: c
28. Find the locus of points in the plane satisfying the relation |z + 5|2 + |z 5|2 = 75. Ans:
circle
29. The function f (z) = z is (a) analytic at z = 0, (b) continuous at z = 0;(c) differentiable
only at z = 0; (d) analytic anywhere. Ans: b
30. If f (z) = u + iv is an analytic function and u v = (x y)(x2 + 4xy + y 2 ) then f (z) is
(a)z 3 + c, (b) iz 2 + ic, (c)iz 3 + , (d) z 3 ic, Ans: c
y
31. If f (z) = u + iv, is an analytic function of z and u v = cos x+sin xe
2 cos xey ey
; find f (z) if f (z)
subject to the condition f (/2) = 0. Ans: 1/2{1 cot(z/2)}

11
32. If f (z) = u(r, ) + iv(r, ) is an analytic function and u = r3 sin 3 then construct the
analytic function f (z).
2 sin 2x
33. If f (z) = u + iv, is analytic function of z and u + v = 2 cos 2x+e2y e2y
; find f (z) in terms
of z. Ans: 21 (1 + i) cot z + d
34. Choose the correct code for matching list A and B.

I
A (u is given) B (f (z) = u + iv is an analytic function)
p. x3 3xy 2 + 3x + 1 (i). sin z + ci
q. y 3 3x2 y (ii).z 3 + 3z + 1 + ci

ON
r. sin x cosh y (iii).i(z 3 + c)

i ii iii
(a) p r q
(b) r p q
(c) p q r Ans: b
x2 y2 x2 y2
35. if sin( + i) = x + iy prove that (a) cosh2
+ sinh2
= 1, (b) cos2
sin2
= 1.
S
36. Prove that (
2
+
2
x2 y 2
)|f (z)|2 = 4|f 0 (z)|2 for an analytic function f (z) = u + iv.

37. Find the harmonic conjugate of u = x3 3xy 2 + 3x2 3y 2 + 2x + 1 and corresponding


analytic function f (z) = u + iv. Ans: v = 3x2 y y 3 + 6xy + 2y + d, f (z) = z 3 + 3z 2 +
K.
2z + 1 + id.
38. Find orthogonal trajectory of v = e2x (x cos 2y y sin 2y) Ans: e2x (x sin 2y +y cos 2y)+d


R.

12
Tutorial5, B. Tech. Sem III, 14 Sep., 2016
(Complex Integrations)
1. Define simply and multiply connected regions? State and prove Cauchys theorem for an
analytic function? Is it true for multiply connected regions?
2. State and prove Cauchy integral formula for nth derivative? Is it true for multiply con-
nected regions? If yes, give your explanation with necessary proofs.

I
3. If f (z) is analytic in a simply connected region D and a, z are two points in D, then
Rz
f (z)dz is independent of the path in D joining a and z.

ON
a
R (2,4)
4. Evaluate (0,3) z dz along (a) parabola x = 2t, y = t2 + 3, (b) a straight line joining (0, 3)
and (2, 4). Find whether both values are different, if yes, justify reason why it is so?
R
5. Evaluate C z dz from z = 0 to z = 4 + 2i along the curve C given by (i)z = t2 + it; (ii)
the line from z = 0 to z = 2i and then the line from z = 2i to z = 4 + 2i. Ans: (i) 10 8i3
; (ii)10 8i.
H 2
6. If F (a) = z +2z5 za
dz, where C is the ellipse (x/2)2 + (y/3)2 = 1. Find the value of
S
F (4.5). Ans: Zero
H 1
7. Evaluate za
C

dz, where C is any simple closed curve and z = a is (i) outside C; (ii)
C
inside C. Ans: (i) 0 ; (ii) 2i.
K.
H 1 +
8. Evaluate (za) n dz ; n 6= 1, n Z , where C is any simple closed curve and z = a is
C
inside C. Ans: Zero
9. Suppose f (z)
H is1 analytic
H inside and on a simple closed curve C and z = a is inside C.
1
Prove that za dz = za dz where C1 is a circle center at a and totally contained in
C C1
simple closed curve C.
H
10. Evaluate (z)2 dz ; where C is the circle (i)|z 1| = 1, (ii)|z| = 1 Ans: 4i, 0.
C
R.

H
11. Find zdz around (a) the circle |z 2| = 3, (b) the ellipse |z 3| + |z + 3| = 10, (c) the
C
square with vertices 0, 2, 2i and 2 + 2i. Ans: 18i, 40i, 8i
12. Suppose f (z) is integrable along a curve C having finite length
R l and there exists a positive
real number M such that |f (z)| M on C. Prove that | C f (z)dz| M l.
Remark: This result is helpful to evaluate the upper bound of an integral without
evaluating it.
13. WorkH out the following integrals around the contour prescribed against it
z 2 +sin z 2
(i) cos(z1)(z2) dz; C: |z| = 3. Ans: 4i.
CH
1
(ii) z2 +16 dz; C: |z| = 6. Ans: 0
C

13
e5z
H
(iii) z2i
dz; C: |z 2| + |z + 2| = 6. Ans: 2ie10i
C
1 ez
H
(iv) 2i z2
dz; C: |z| = 3. Ans: e2
C
sin z
H
(v) 2
z 1
dz;
C

C is the rectangle with vertices 2 + i, 2 i, 2 + i, 2 i;


C is the rectangle with vertices i, 2 i, 2 + i, i. Ans: 0, 0.

I
H eiz
(vi) z2 4z+5 dz; C: |z 1 2i| = 2. Ans: ei

ON
CH
ez +cos z
(vii) (z5)(z+5i) dz; where C is the boundary of a triangle with vertices: 1, 1, 7/2i.
C
Ans: 2i(i5)
2 +25 (2 cos i sin i)
H sin 6z
(viii) z/6 dz; C: |z| = 2. Ans: i25
C
2
ez
R
(ix) Show that C z2 (z1i) dz = e2i ; where C consists of |z| = 2 anticlockwise and |z| = 1
clockwise.H
ez
1
(x) 2i (z+2)2
dz; Ans: e2
C:|z|=4

14. Evaluate
S
H
H
C
1
z 2 1
dz around z = i + 5eit . Ans: 0

15. Evaluate (z 2 + 3 + 4/z)dz, C: |z| = 4. Ans: 8i


C

16. Evaluate the upper bound of the integral without evaluating it ?


K.
R
(a) C (z 4 + 1)dz, C : Line segment from 0 to 1 + i. Ans: 5 2.
R
(b) C zdz, C : Line segment from 0 to i. Ans: 1.
R
(c) C 2zdz, C : Line segment from i to 2 + i. Ans: 4 5.
R
(d) C (x2 + iy 2 )dz, C : z = eit , /2 < t < /2. Ans:
17. Which of the following integrals are compatible to apply the Cauchy theorem?
R.

H sinz
1. z+2i dz, C : |z| = 1.
C
sinz
H
2. z+2i
dz, C : |z + 3i| = 1.
C
H
3. ez dz, C : |z 3i| = 6.
C
1
H
4. z 2 +bz+1
dz, 0 < b < 1.
|z|=b

1
H
5. 1ez
dz.
|z|=3

14
1+i
R
6. z 3 dz, along y = x.
0
1
H
1. Evaluate z 3 z
dz, Ans: 0
|z|=3

z2
H
2. Evaluate z2
dz where C is the boundary of a triangle with vertices 1, 0 and 2i. Ans:
C
0

I
3. Evaluate
H z+ezthe following integrals around the contour prescribed against it
it
(i) (z+i) dz; C: z = 7e , 0 t 2. Ans: i.

ON
3
CH
1 3i
(ii) z3 (z2) 2 dz; C: |z 3| = 2, Ans: 8
.
C
H z+ez it
(iii) (z+i) 3 dz; C: z = 7e , 0 t 2. Ans: i.
CH
1
(iv) z3 (z2) 3 dz; C: |z 1| = 3, Ans: 0.
C
1 3z 4
H
(v) 2i (z6i)
dz. Ans: 3 64
H|z|=10 1
(vi) z 4 1
dz. Ans: 0.

(vii)
S
|z1|=5
H
|zi|=3/2
1
z 4 (z+i)
dz. Ans: 4i.
ez z 2
H
(viii) (z2)3
dz. Ans: 2i(2e4 1);
|z1|=3
H sin z
(ix) (z1)2
dz. Ans: 2i cos 1;
K.
|z|=2
cos z
H
(x) (z+3i)6
dz. Ans: 0;
|z+i|=3/2
3
H
(xi) z2 (z+i) 2 dz; C: |z| = 5, Ans: 0.
C H
ez
(xii) z 2 +a2
dz. Ans: 2i a
sin a;
|z|=2a
ez ia
dz. Ans: e a
H
(xiii) z 2 +a2
|zia|=a
z
(xiv) z+1e
H
dz; C: |z i| = 2, Ans: 0.
R.

z(z+3)
C
H
4. Evaluate (x2 + iy 2 )ds; C: |z| = 2 where s is the arc length. Ans: 8(1 + i)
C

15
Tutorial6, B. Tech. Sem III, 5 October, 2016
(Laurents Expansions)
1. State and prove Laurents series expansion of a function f (z).
2. Find Laurents series expansion about the indicated singularity for each of the following
functions. Name the singularity in each case and give the region of convergence of each
series.
e2z

I
(i). (z1) 3; z = 1

1 2 2 4 2(z1)
Ans: e2 [ (z1) 3 + (z1)2 + (z1) + 3 + 3
+ ........; z = 1 is a pole of order 3 and Series

ON
converges for all z 6= 1.
1
(ii). (z 3) sin (z+2) ; z = 2;
5 1 5 1
Ans: 1 z+2 6(z+2) 2 + 6(z+2)3 + 120(z+2)4 ........; z = 2 is an essential singularity and

Series converges for all z 6= 2.


(iii). zsinz
Z3
; z = 0;
2 4
Ans: 3!1 z5! z7! ..; z = 0 is a removable singularity. Series converges for all z.
z
(iv). (z+1)(z+2) ;z = 2;
2
Ans: z+2 + 1 + (z + 2) + (z + 2)2 + ..; z = 2 a simple pole and Series converges for all
S
z in 0 < |z + 2| < 1.
1
(v). z2 (z3) 2; z = 3

1 2 1 (z3)
Ans: 9(z3) 2 27(z3) + 27 243 + ........; z = 3 is a pole of order 2 and Series converges

for all z in 0 < |z 3| < 3.


K.
1
3. Expand f (z) = (z+1)(z+3) in a Laurent s series valid for expansion (a). 1 < |z| < 3,
(b). |z| > 3, (c) 0 < |z + 1| < 2, (d) |z| < 1,
z2 z3
Ans: (a). 2z14 + 2z13 2z12 + 2z 1
16 + 18
z
54 + 162 ;
1 4 13 40
(b). z2 z3 + z4 z5 + ;
1
(c). 2(z+1) 14 + (z+1)
8
16 1
(z + 1)2 + ;
(d). 31 4z
9
+ 13
27
z2 + .
z
4. Expand f (z) = (z1)(2z) in a Laurents series valid for (a). 1 < |z| < 2; (b). |z| > 2;
R.

(c). 0 < |z 2| < 1; (d). |z| < 1; (e). |z 1| > 1.


2 3
Ans: (a). 1 + z2 + z4 + z8 + z1 + z12 + ;
(b). 12 z32 z73 15 z4
;
2
(c). 1 z2 (z 2) + (z 2)2 (z 2)3 + (z 2)4 ;
(d). 21 z 43 z 2 87 z 3 19
15 4
z .
1 2 2
(e). (z1) (z1)2 (z1)3 ;
1
5. Expand f (z) = z3 in a Laurents series valid for (a). |z| < 3; (b). |z| > 3
Ans: (a). (i)1/3 1/9z 1/27z 2 1/81z 3 ;
(ii) z 1 + 3z 2 + 9z 3 + 27z 4 + .
1
6. Expand f (z) = z(z2)
in a Laurents series valid for (a). 0 < |z| < 2; (b). |z| > 2
n1
1 zn 2
P P
Ans: (a). 2z 2n+2
; (b). z n+1
;
n=0 n=1

16
7. Find Taylors series expansion about the indicated points for each of the following func-
tions. Give the region of convergence of each series.
(i). ezz+1 ; z = 0;
(ii). zsin z
2 +4 ; z = 0

Ans: (i) |z| < ; (ii) |z| < 2

I
S ON
K.
R.

17
Tutorial7, B. Tech. Sem IV, 24 Feb, 2016
(Singularities and Residues)
1. Discuss the types of each singularity for the following functions:
1
(i). cos zsin z
; z = /4 is a simple pole.
cot z
(ii) . (za)2 ; z = a is a pole of order 2.
ez
(iii). z2 +1 ; z = i is a simple pole and z = is an essential singularity.
z
(iv). (z+1)(z+2) ; z = 1, 2 are simple poles.

I
ez
(v). sin z ; z = k, k I is a simple pole.
1
(vi). arcsin z ; z = 1/n, (n I+ ) simple pole and z = 0 an isolated essential singularity.

ON
(vii). arctan z; z = 1/n, (n I+ ) is simple pole and z = 0 isolated essential singularity.
(viii). zsinz
z3
; z = 0 is a removable singularity.
(ix). zz+1
2 2z ; z = 0, 2 are simple poles.
2
(x). (z1)z2 (z+2) ; z = 2 is a simple pole and z = 1 is a pole of order 2.
(i+2k)
(xi). z41+1 ; e 4, k = 0, 1, 2, 3, 4 each is a simple pole.
iz
(xii). zze
2 +a2 ; z = ai are simple poles.
2z
(xiii). 1e
z3
; z = 0, a pole of order 2.
e2z
S
(xiv). 1+ez ;
e2z
(xv). 1e z;
zsinz
(xvi). z5 ;
z = i is a simple pole.
z = 0 is a simple pole.
z = 0 is a pole of order 2.
(xvii). z2 (e1z 1) ; z = 0 is a pole of order 3.
(xviii). sinh
z2
z
; z = 0 is a simple pole.
K.
(z +1)ez
2
(xix). (z1)3 (zi) ; z = 1 is a pole of order 3 and z = i is a removable singularity.
2. Examine the nature of singularity for the following functions and find its residue:
1
Hint: residue is defined as coefficient of (zz 0)
in Laurents series expansion of a function
f (z) about a point z0 . Thus,
1 dm1 m
residue for a pole of order m is (m1)! lim dz m1 ((z z0 ) f (z))
zz0
residue for a removable singularity=0.
1
residue for an essential singularity z0 can be find by finding coefficient of zz0
in its
R.

Laurents series expansion about a point z0 .


z
(i). e +sin
z4
z
; z = 0 is a removable singularity, Residue= 0
1
(ii). (z+i)3 ; z = i is a a pole of order 3. Residue = 6i3 .
zeiz ea ea
(iii). z 2 +a2
; z = ai are simple poles. Residues are = 2
. and= 2
.
1
(iv). z 2 sinh z
; z = 0 a pole of order 3. Residue= 61
3. Using R 2Residue theorem evaluate the following integrals:
1 2
(i). 0 2+cos d, Ans: 3
.
R 2 1
(ii). 0 12a cos +a2 d, a > 1 Ans: ....
R 1+2 cos
(iii). 0 5+3 cos d, Ans: 24 .
R 2 cos 2
(iv). 0 5+4 cos d, Ans: 6
R 2 cos2 3
(v). 0 54 cos 2 d, Ans: 3 8

18
Tutorial9, B. Tech. Sem III, 22 October, 2016
(Bays rule)
1. Two third of the students in a class are girls and rest boys. If it is known that probability
of a boy getting first class is 0.25, that of a girl is 0.28. Find the probability that a student
chosen at random will get first class. Ans: ...
2. The chances that doctor will diagnose a disease X correctly is 60%. The chances that a

I
patient will die by his treatment after correct diagnosis is 40% and the chance of death
by wrong diagnosis is 70%. A patient of the doctor, who had disease X, died. What is
6

ON
the probability that his disease was diagnose correctly. Ans: 13
3. A manufacturing firm produces steel pipes in three plants with daily production volumes
of 500, 1000 & 2000 units respectively. According to past experience it is known that the
fraction of defective output produced by three plants are respectively 0.005, 0.008, 0.01.
What is the probability that a pipe is selected at random from a days output and found
to be defective? Ans: ...
4. In a university 4% of male students and 1% of female students are taller than 6 feet.
Further 60% of the students are female. Now, if a randomly selected student is taller
than 6 feet, what is the probability that the student is a female? Ans: ...
S
5. Two shipment of parts are received. The first shipment contained 1000 parts with 10%
defectives and the second shipment contains 1500 parts with 5% defectives. One shipment
is selected at random, two parts were tested at random and found to be good. Find the
probability that tested parts were selected from first shipment. Ans: ...
K.
6. A coin is tossed. If it turn up H, two balls will be drown from urn A, otherwise 2 balls will
be drawn from urn B. Urn A contains 3 Red and 5 Blue balls, Urn B contains 7 Red and
5 Blue balls. What is the probability that urn A is used, given that both balls are blue.
(Find in both cases, when balls were chosen with replacement and without replacement).
Ans: ...
7. There are 10 urns of which each of 3 contains 1W, 9B balls, each others 3 contains 9W,
1B balls and remaining 4 each contains 5W, 5B balls. One of the urn was selected at
R.

random and a ball was chosen from it. (i) what is the probability that it is white ball?
(ii) If ball is black, what is the chance that it comes out from the urn consisting 9W and
1B balls. Ans: ...
8. There are two bags A and B. A contains n white and 2 black balls and B contains 2 white
and n black balls. One of the two bags is selected at random and two balls are chosen
from it without replacement. If both the balls drawn are white and the probability that
the bag A was used to draw the ball is 67 , find out the value of n. Ans: n = 4.

19
Numerical Integration

If f : [a, b] R is differentiable then, we obtain a new function f 0 :

[a, b] R, called the derivative of f . Likewise, if f : [a, b] R is

integrable, then we obtain a new function F : [a, b] R defined by

Z x
F (x) = f (t)dt x [a, b].
a

Z b
Observation: If f is nonnegative function, then f (x)dx is represent
a
the area under the curve f (x).

Dr. Raj Kumar, VBSPU Jaunpur. 1


Antiderivative

Antiderivative: Let F : [a, b] R be such that f = F 0, then F is called

an antiderivative of f .

Recall

Fundamental Theorem of Calculus: Let f : [a, b] R is integrable

and has an antiderivative F , then


Z b
f (x)dx = F (b) F (a).
a

Dr. Raj Kumar, VBSPU Jaunpur. 2


Basic Problems

Difficult to find an antiderivative of the function (for example f (x) =


2
ex )

Function is given in the tabular form.

Dr. Raj Kumar, VBSPU Jaunpur. 3


Newton-Cotes Methods/Formulae

The derivation of Newton-Cotes formula is based on Polynomial

Interpolation.

x x0 x1 x2 ... xn

f (x) f (x0) f (x1) f (x3) ... f (xn)

Dr. Raj Kumar, VBSPU Jaunpur. 4


The idea is:

Rb
Replace f by pn(x) and evaluate a
pn(x)dx

That is,
Z b Z b n
Z bX
f (x)dx ' pn(x)dx = li(x)f (xi)dx
a a a i=0
n
X Z b
= f (xi) li(x)dx
i=0 a

Xn
= Aif (xi)
i=0
Rb
Where Ai = l (x)dx
a i
called weights.

Dr. Raj Kumar, VBSPU Jaunpur. 5


Types of Newton-Cotes Formulae

Trapezoidal Rule (Two pint formula)

Simpsons 1/3 Rule (Three Point formula)

Simpsons 3/8 Rule (Four point formula)

Dr. Raj Kumar, VBSPU Jaunpur. 6


Trapezoidal Rule

Since it is two point formula, it uses the first order interpolation

polynomial P1(x).

Z b Z x1
f (x) P1(x)dx
a x0

P1(x) = f (x0) + sf (x0)


x x0
s=
h
Now, dx = h ds at x = x0, s = 0 and at x = x1, s = 1.

Dr. Raj Kumar, VBSPU Jaunpur. 7


Hence,

Z b Z 1
h
f (x)dx (f (x0) + sf (x0))hds = [f (x0) + f (x1)]
a 0 2

OR
b
ba
Z
f (x)dx [f (a) + f (b)]
a 2
Error
T (b a)3 00
E = f (),
12
where a < < b

Remark: x0 = a and x1 = b.

Dr. Raj Kumar, VBSPU Jaunpur. 8


1
Basic Simpsons 3 Rule

Rb
To evaluate a
f (x)dx.

f will be replaced by a polynomial of degree 2 which interpolates f at


a+b a+b
a, 2 and b. . Here, x0 = a, x1 = 2 , x2 =b

b
ba
Z
b+a
f (x) dx = [f (a) + 4f ( ) + f (b).]
a 6 2

Error
sh5f (4)()
E =
90
for some (a, b).

Dr. Raj Kumar, VBSPU Jaunpur. 9


3
Basic Simpsons 8 Rule

f is replaced by p3(x) which interpolates f at x0 = a, x1 = a +


ba
h, x2 = a + 2h, x3 = a + 3h = b. where h = 3 . Thus we get:

Z b
3h
f (x)dx ' [f0 + 3f1 + 3f2 + f3]
a 8

s 3h5 (4)
Error: E = f (), where a < < b.
80

Dr. Raj Kumar, VBSPU Jaunpur. 10


Example

1
R2 4
R2
Using Trapezoidal and Simpson 3 rules find 0
x dx and 0
sinxdx and

find the upper bound for the error.

Dr. Raj Kumar, VBSPU Jaunpur. 11


Composite Rules

Note that if the integral [a, b] is large, then the error in the Trapezoidal

rule will be large.

Idea

Error can be reduced by dividing the interval [a, b] into equal subinterval

and apply quadrature rules in each subinterval.

Composite Trapezoidal Rule

ba
h= , xi = x0 + ih
n
Dr. Raj Kumar, VBSPU Jaunpur. 12
Composite Rule

Z b Z xn n Z
X xi
f (x)dx = f (x)dx = f (x)dx
a x0 i=1 xi1

Now apply Trapezoidal rule on each [xi1, xi], we have

Z b
h
f (x)dx = [f (x0) + 2 (f (x1) + f (x2) f (xn1)) + f (xn)]
a 2

Dr. Raj Kumar, VBSPU Jaunpur. 13


Error in composite Trapezoidal rule

CT h2 00
E = (b a) f (), [a, b]
12

Dr. Raj Kumar, VBSPU Jaunpur. 14


1
The Composite Simpsons 3 Rule

[a, b] will be will be divided into 2n equal subintervals and we apply


1
basic Simpsons 3 rule on each of the n intervals [x2i2, x2i] for i =

1, 2, 3, , n.

ba
Thus here h = 2n .

Then
Z b Z b=x2n
f (x)dx = f (x)dx
a a=x0
Z x2 Z x4 Z x2i Z x2n
= f (x)dx + f (x)dx + + f (x)dx + + f (x)dx
x0 x2 x2i2 x2n2

Dr. Raj Kumar, VBSPU Jaunpur. 15


h h
= [f (x0) + 4f (x1) + f (x2)] + [f (x2) + 4f (x3) + f (x4)]+
3 3
h
+ + [f (x2n2) + 4f (x2n1) + f (x2n)]
3
h
= {f (x0) + 4 [f (x1) + f (x3) + f (x5) + + f (x2n1)]+
3
+2 [f (x2) + f (x4) + f (x6) + + f (x2n2)] + f (x2n)}

CS h4 (4)
E = (b a) f (),
180
where [a, b]

Dr. Raj Kumar, VBSPU Jaunpur. 16


Example

R1 1
Evaluate the integral 1
x2 exp(x)dx by composite Simpsons 3 rule

with spacing h = 0.25

1
Solution: According to composite Simpsons 3 rule:

Z 1
h
x2 exp(x)dx = [f (x0) + 4f (x1) + 2f (x2) + 4f (x3) + 2f (x4)+
1 3

+4f (x5) + 2f (x6) + 4f (x7) + f (x8)]

Here f (x0) = f (1) = 2.7183

f (x1) = f (0.75) = 1.1908

Dr. Raj Kumar, VBSPU Jaunpur. 17


f (x2) = f (0.5) = 0.4122

f (x3) = f (0.25) = 0.0803

f (x4) = f (0) = 0

f (x5) = f (0.25) = 0.0487

f (x6) = f (0.50) = 0.1516

f (x7) = f (0.75) = 0.2657

f (x8) = f (1) = 0.3679

Dr. Raj Kumar, VBSPU Jaunpur. 18


Substituting these values in the above formula we get:

Z 1
x2 exp(x)dx ' 0.87965
1

Dr. Raj Kumar, VBSPU Jaunpur. 19


Example

Find the minimum no. of subintervals, used in composite Trapezoidal


Z 1
x4
and Simpsons 1/3 rule in order to find the integral e dx such that
0
the error can not exceed by .00001.

Sol. For the composite Trapezoidal rule, we have

00
13 max0<<1 |f ()|
.00001
12n2trap
For the composite Simpson 1/3 rule, we have

Dr. Raj Kumar, VBSPU Jaunpur. 20


14 max0<<1 |f (4)()|
.00001
180n4simp
Now,

00
max |f ()| 3.5, max |f (4)()| 95
0<<1 0<<1

(Please verify )

Hence

ntrap = 171, nsimp = 16

Dr. Raj Kumar, VBSPU Jaunpur. 21


3
Composite Simpsons 8 rule

ba 3
[a, b] is divided into 3n equal subintervals. (h = 3n . and we apply 8

rule on each of the n intervals [x3i3, x3i] for i = 1, 2, 3, , n.)

Hence,

Z b Z x3 Z x6 Z x3n =b
f (x)dx ' f (x)dx + f (x)dx + + f (x)dx
a x0 =a x3 x3n3

3h 3h
= [f0 + 3f1 + 3f2 + f3] + [f3 + 3f4 + 3f5 + f6]+
8 8
3h
+ + [f3n3 + 3f3n2 + 3f3n1 + f3n]
8
Dr. Raj Kumar, VBSPU Jaunpur. 22
3h
= [f0 + 3f1 + 3f2 + 2f3 + 3f4 + 3f5 + 2f6 + 3f7 + + 3f3n1 + f3n]
8
Remember:

f with suffices of multiple 3 are multiplied by 2.

Others by 3, except the end points.

Dr. Raj Kumar, VBSPU Jaunpur. 23


Example

3
Use composite simpsons 8 rule, find the velocity after 18 seconds, if a

rocket has acceleration as given in the table:


t= 0 2 4 6 8 10 12 14 16 18

a= 40 60 70 75 80 83 85 87 88 88

f0 f1 f2 f3 f4 f5 f6 f7 f8 f9

3h
Sol: Velocity v = 8 [f0 +3f1 +3f2 +2f3 +3f4 +3f5 +2f6 +3f7 +3f8 +f9 ]=
3
4 [40+360+370+275+380+383+283+285+387+388+88]

= 1389 units.

Dr. Raj Kumar, VBSPU Jaunpur. 24


Method of Undetermined Parameters

The Newton - Cotes integration rules are all of the form

I(f ) ' A0f (x0) + A1f (x1) + A2f (x2) + + Anf (xn)

Also, note that the weights Ais do not depend on the given function.

Hence, if the error is of the form

E(I) = Const f (r+1)().

Then the rule must be exact for all polynomials of degree r

Dr. Raj Kumar, VBSPU Jaunpur. 25


Therefore

If we wish to construct a rule of the form

I(f ) ' A0f (x0) + A1f (x1) + a2f (x2) + + Anf (xn)

(n-fixed) which is exact for polynomials of degree as high as possible,

i.e., we want

E(I) = Const f (r+1)(),

with r as large as possible.

This way of constructing integration rules is called the Method of

Undetermined Parameters.

Dr. Raj Kumar, VBSPU Jaunpur. 26


Example

Suppose we want to derive an integration formula of the form:


Rb
a
f (x)dx = A0 f (a) + A1 f (b) + f 00().

We assume that:The rule is exact for the polynomials 1, x, x2.

Now, taking f (x) = 1, we get b a = A0 + A1

b2 a2
Taking f (x) = x we get 2 = A0 a + A1 b

ba
Solving the above two equations we get, A0 = A1 = 2 .
Z b
ba
Thus, f (x)dx = [f (a) + f (b)] + f 00()
a 2

Dr. Raj Kumar, VBSPU Jaunpur. 27


Now if we take f (x) = x2, we get:

b3 a3 ba 2
=( )(a + b2) + 2!
3 2

(ba)3
= = 12

Thus
b
ba (b a)3 00
Z
f (x)dx = [f (a) + f (b)] f ()
a 2 12

We see that: This is exactly the trapezoidal rule. Similarly, Simpsons


1 3
3 and 8 rules can be derived.

Dr. Raj Kumar, VBSPU Jaunpur. 28


Thus in the Method of Undetermined Parameters

We aim directly for a formula of a preselected type.

Working Method:

We impose certain conditions on a formula of desired form and use

these conditions to determine the values of the unknown coefficients in

the formula.

Dr. Raj Kumar, VBSPU Jaunpur. 29


The Error term in the Simpsons 38 -rule, using Method

of Undetermined Parameters

Start with:

Z x3
3h
f (x)dx = [f0 + 3f1 + 3f2 + f3] + f (4)()
x0 8

for some suitable (x0, x3).

Takeing f (x) = x4 in the above integration rule we get:

x53 x50 3h 4
= [x0 + 3x41 + 3x42 + x43] + 4!
5 8
Dr. Raj Kumar, VBSPU Jaunpur. 30
x53 x50 3h 4
4! = [x0] + 3(x0 + h)4 + 3(x0 + 2h)4 + (x0 + 3h)4
5 8

(x0 + 3h)5 x50 3h 4


= [x0] + 3(x0 + h)4 + 3(x0 + 2h)4 + (x0 + 3h)4
5 8

Without loss of generality, we can take: x0 = 0.


243 5 3h5
We have: 4! = 5 h 8 [0 + 3 + 3 16 + 81] Thus

9 5
4! = h
10

That is,

3 5
= h
80
Dr. Raj Kumar, VBSPU Jaunpur. 31
Therefore the error in the Simpsons rule is =

3 5 (4)
h f ()
80

for some suitable (a, b).

Dr. Raj Kumar, VBSPU Jaunpur. 32


Recall

The Newton - Cotes integration rules are all of the form

I(f ) ' A0f (x0) + A1f (x1) + A2f (x2) + + Anf (xn)

Also, note that the weights Ais do not depend on the given function.

Hence, if the error is of the form E(I) = Const f (r+1)(). Then the

rule must be exact for all polynomials of degree r.

Remark: In these quadrature the points xi are fixed.

Ques: Can we improve the accuracy by choosing some suitable xi

Ans: Using Gaussian Quadrature rule one can improve the accuracy.

Dr. Raj Kumar, VBSPU Jaunpur. 33


Example

Find x0, x1, A0, A1 and so that the following rule is exact for all

polynomials of degree 3.

Z 1
f (x)dx = A0f (x0) + A1f (x1) + f (4)()
1

(There are 4 unknowns and hence we have chosen the 4-th derivative

in the error term.)

Taking f (x) = 1, x, x2, x3 we get:

A0 + A1 = 2

Dr. Raj Kumar, VBSPU Jaunpur. 34


A0 x 0 + A1 x 1 = 0

2
A0x20 + A1x21 = 3

A0x30 + A1x31 = 0

On solving these equations we get:

A0 = A1 = 1 x0 = 13 and x1 = 1 .
3

R1
Thus the integration rule is: 1
f (x)dx = f ( 13 ) + f ( 13 ) + f (4)().

Now if we take f (x) = x4 we get

2 2
= + 4!
5 9
Dr. Raj Kumar, VBSPU Jaunpur. 35
1 8 1
= = ( )=
4! 45 135

Thus the expected integration rule is:

Z 1
1 1 1 (4)
f (x)dx = f ( ) + f ( ) + f ().
1 3 3 135

Dr. Raj Kumar, VBSPU Jaunpur. 36


In general

Giving a positive integer n, we wish to determine 2n + 2 numbers

x0, x1, xn and A0, A1, , An so that the sum

I(f ) ' A0f (x0) + A1f (x1) + A2f (x2) + + Anf (xn),

Z b
provides the exact value of f (x)dx for f (x) = 1, x, x2, x2n+1.
a
Or What we want is that the quadrature rule is exact for all polynomials

of degree 2n + 1.

Remark: Here we have to solve system of nonlinear equations, which is

some time is not an easy job.

Dr. Raj Kumar, VBSPU Jaunpur. 37


Orthogonal Functions

Two functions f, g : [a, b] R are said to be orthogonal if

Z b
f (x)g(x)dx = 0.
a

If
Z b
w(x)f (x)g(x)dx = 0
a

for some function w(x) > 0, x [a, b] then we say that f and g are

orthogonal with respect to w(x)

We call the function w(x) called as weight function.

Dr. Raj Kumar, VBSPU Jaunpur. 38


In general

Let w : [a, b] (0, ) be given.

Functions f1(x), f2(x), f3(x), defined on [a, b] is said to orthogonal

w.r. to the weight function w(x) if


Z b
w(x)fm(x)fn(x)dx = 0 if m 6= n
a

and

Z b
w(x)fm(x)fn(x)dx 6= 0 if m = n
a

Dr. Raj Kumar, VBSPU Jaunpur. 39


Gauss Quadrature Method says that:

Theorem:

Rb
To evaluate a
f (x)dx

Rb Rb
1. Suppose we write the integral as a
f (x)dx = a
w(x)g(x)dx for some

positive function w(x).

and

2. Suppose we are able find a sequence of orthogonal polynomials

p0(x), p1(x), p2(x), w.r.to the weight function w(x).

Dr. Raj Kumar, VBSPU Jaunpur. 40


Then

Given any k. If we choose the points x0, x1, x2, , xk as the zeros of
Rb
the polynomial pk+1(x) and the coefficients Ai = a li(x)w(x)dx (i =

0, 1, 2, k) then the integration formula

I(f ) ' A0g(x0) + A1g(x1) + + Ak g(xk )

will be exact for all polynomials of degree 2k + 1

where
k
Y (x xj ) f (x
, g(x) =
(xi xj ) w(x)
j=0,j6=i

Dr. Raj Kumar, VBSPU Jaunpur. 41


with error term

)
f (2k+2)() b
p2k+1(x)
Z
E(I) = w(x) 2 dx (1)
(2k + 2)! a k+1

Where k+1 is the coefficient of xk+1 in pk+1(x).

Dr. Raj Kumar, VBSPU Jaunpur. 42


Legendre Polynomials

[a, b] = [1, 1]

w(x) 1

1 dn 2 n
pn(x) = (x 1) for n = 0, 1, 2, . (Rodriguess Formula)
n!2n dxn
Then p0(x) = 1, p1(x) = x, p2(x) = 21 (3x2 1), .
Z 1
we know pm(x)pn(x)dx = 0 m 6= n and
1 Z
1
2
if m = n, then pm(x)pn(x)dx = .
1 2n + 1

Hence p0, p1, p2 are orthogonal on [1, 1].

Dr. Raj Kumar, VBSPU Jaunpur. 43


Remark

Z b Z 1
The integral f (x)dx can be convert into g(t)dt
a 1
Choose
ba a+b
x= t+
2 2
Then
b 1  
ba ba
Z Z
a+b
f (x)dx = f t+ dt
a 2 1 2 2

Dr. Raj Kumar, VBSPU Jaunpur. 44


Two point Gauss-Legendre Quadrature

The case when we take only two points x0 and x1 in our integration

formula.

Let
Z 1
f (x)dx = A0f (x0) + A1f (x1) + f (4)(), (1, 1).
1

That is the case k = 1 in the above theorem.

Therefore we take the Legendre polynomial of degree 2, that is p2(x) =


1 2
2 (3x 1)

Dr. Raj Kumar, VBSPU Jaunpur. 45


Find the roots of p2(x) = 0

1 1
p2(x) = 21 (3x2 1) = 0 = x =
3
, 3

1 1
Therefore as said in the above theorem we take x0 =
3
and x1 = 3

To fond A0 and A1:

Here w(x) 1.

Thus
1 1
x x1
Z Z
A0 = w(x)l0(x)dx = 1. dx
1 1 x 0 x 1
Z 1
3 1
= (x )dx = 1
1 2 3

Dr. Raj Kumar, VBSPU Jaunpur. 46


1 1
x x0
Z Z
A1 = w(x)l1(x)dx = 1. dx
1 1 x 0 1 x 0
Z 1
3 1
= (x + )dx = 1
1 2 3
Thus the 2-point Gauss-Legendre Quadrature formula takes the form:

Z 1
1 1
f (x)dx ' f ( ) + f ( )
1 3 3

Error Term:

(4) 1
(3x2 1)2 f (4)() 8
Z
f () 1 (4)
E(I) = dx = = f ()
4! 1 9 24 45 135

Dr. Raj Kumar, VBSPU Jaunpur. 47


Three point Gauss-Legendre Quadrature

We use three points x0, x1 and x2 in our integral formula.

This is the case when k = 2

We take p3(x) = 21 (5x3 3x)

Find the roots of p3(x) = 0


q q
p3(x) = 21 (5x3 3x) = 0 = x = 35 , 0, 35
q q
Take x0 = 35 , x1 = 0, x2 = 35

To fond A0, A1 and A2:

Dr. Raj Kumar, VBSPU Jaunpur. 48


1 1
(x x1)(x x2)
Z Z
A0 = w(x)l0(x)dx = w(x).
1 1 (x0 x1)(x0 x2)
q
Z 1 (x 0)(x 35 )
= q q q
3 3 3
1 (
5 0)( 5 5)
Z 1 r
2 3 5
= (x x)dx = .
1 5 9

q q
3 3
Z 1 (x + 5 )(x 5)
A1 = w(x) q q dx
3 3
1 (0 + 5 )(0 5)
Z 1
3 2 8
= (x )dx =
1 5 9

Dr. Raj Kumar, VBSPU Jaunpur. 49


q
Z (x + 35 )(x 0)
1
5
A2 = q q q dx =
1 ( 3
+ 3
)( 3
0) 9
5 5 5

Error Term:
f (6)() 1 ( 25 x3 32 x)2
Z
E(I) = 5 2 dx
6! 1 (2)
(6) Z 1
f () 3
= (x3 x)2dx
6! 1 5
1
= f (6)().
15750
Thus the 3-point Gauss-Legendre quadrature formula takes the form:
Z 1
5 3 8 5 3
f (x)dx = f ( ) + f (0) + f ( ) + E(I)
1 9 5 9 9 5

Dr. Raj Kumar, VBSPU Jaunpur. 50


Example

Using 3-point Gauss-Legendre quadrature, evaluate the integral


R 1 x sin x
1 1+x2
dx

Solution:
Z 1
x sin x 5 3 8 5 3
2
dx = f ( ) + f (0) + f ( )
1 1 + x 9 5 9 9 5
5 0.5417744 8 5 0.5417744
= [ ] + [0] + [ ] = 0.37623
9 0.599999 9 9 1.599999

Dr. Raj Kumar, VBSPU Jaunpur. 51


Remark

Z b Z 1
The integral f (x)dx can be convert into g(u)du
a 1
Choose
ba a+b
x= u+
2 2
Then
b 1  
ba ba
Z Z
a+b
f (x)dx = f u+ du
a 2 1 2 2

Dr. Raj Kumar, VBSPU Jaunpur. 52


Example

R2
Suppose we want to integrate 0
f (x)dx using 3-point Gauss-Legendre

formula

Let
Z 2
f (x)dx = A0f (x0) + A1f (x1) + A2f (x2) + f (6)()
0

for some (0, 2).

We use the change of variable: x = u + 1

Then x = 0 = u = 1 and x = 2 = u = 1.

Dr. Raj Kumar, VBSPU Jaunpur. 53


Then we get
Z 2 Z 1
f (x)dx = f (u + 1)du
0 1

If we write g(u) = f (u + 1), we get

Z 1
g(u)du ' A0g(u0) + A1g(u1) + A2g(u2)
1

q q
Hence u0 = 35 , u1 = 0, u2 = 35

q
3
Therefore x0 = u0 + 1 = 1 5, x1 = u1 + 1 = 0 + 1 = 1 and x2 =
q
u2 + 1 = 1 + 35

Dr. Raj Kumar, VBSPU Jaunpur. 54


Thus r r
Z 2
5 3 8 5 3
f (x)dx ' f (1 ) + f (1) + f (1 + )
0 9 5 9 9 5

Dr. Raj Kumar, VBSPU Jaunpur. 55


Chebyshev Polynomials

[a, b] = [1, 1]

1
w(x) =
1x2

We know already know that p0(x) = 1, p1(x) = x, Now pn(x) is

recursively defied as

pk+1(x) = 2xpk (x) pk1(x)

For k = 1, 2, 3, . We have p2(x) = 2x2 1, p3(x) = 4x3 3x .

Observation The coefficient of xk+1 in pk+1 is 2k+1.

Dr. Raj Kumar, VBSPU Jaunpur. 56


Note that the Chebyshev Polynomials given above form an sequence
1
of orthogonal polynomials w.r.to the weight function w(x) =
1x2

Hence we can use these polynomials to evaluate integrals of the form

Z 1
1
f (x) dx
1x 2
1

Dr. Raj Kumar, VBSPU Jaunpur. 57


Gauss-Chebyshev Quadrature 2-point formula

Z 1
f (x)
To evaluate dx
1x 2
1

1
Noted above: w(x) = .
1x2

We want to derive 2-point formula, and hence this is the case when

k = 1 in the main theorem.

Therefore we take p2(x) = 2x2 1.

Dr. Raj Kumar, VBSPU Jaunpur. 58


To find the roots of p2(x) = 2x2 1 = 0 :

1 1
= x = ,
2 2

Therefore the two-point Chebyshev forula will be of the form:

1
1
Z
f (x) 1
dx ' A0f ( ) + A1f ( )
1x 2 2 2
1

Dr. Raj Kumar, VBSPU Jaunpur. 59


To find A0 and A1.

1 1
x x1
Z Z
1
A0 = w(x)l0(x)dx = ( )dx
1 1 1 x x0 x1
2
Z 1 Z 1
1 2x 1 1
= dx + dx
2 2 1 1 x 2 2 1 1 x 2

=0+ .
2
Z 1 Z 1
1 x x0
A1 = = w(x)l1(x)dx = ( )dx
2 1 1 1x 1 2 x x 0
q
1
Z 1
1 x + 2
= ( 2 )dx = .
1 1x 2 2
2

Dr. Raj Kumar, VBSPU Jaunpur. 60


Thus,
1
1
Z
f (x) 1

dx ' f ( ) + f ( ).
1 1x 2 2 2 2 2

The Error term is: substitute k = 1in (1) we get:

(4) b
p2(x)2
Z
f () 1
E(I) = dx
4! a 1x 2 4

f (4)()
= 3
2 4!

Dr. Raj Kumar, VBSPU Jaunpur. 61


3-point Gauss-Chebyshev Quadrature

This is the case when k = 2.

Take p3(x) = 4x3 3x

Roots of p3(x) = 0:

3 3 3
4x 3x = 0 = x = 0, 2 , 2


3 3
Thus x0 = 0 x1 = 2 and x2 = 2

Dr. Raj Kumar, VBSPU Jaunpur. 62


Therefore the Three-point Chebyshev forula will be of the form:

1

3
Z
f (x) 3
dx ' A0f ( ) + A1f (0) + A2f ( )
1 1x 2 2 2

To find A0, A1 and A2:


1 1
(x 0)(x 23 )
Z Z
1
A0 = w(x)l0(x)dx = dx
2
1 x ( 2 0)( 2 23 )
3 3
1 1
Z 1 2 3
(x 2 x)
= dx = .
1 1 x3 3

Dr. Raj Kumar, VBSPU Jaunpur. 63


Similarly,

A1 = A2 = .
3

Thus,
1

3
Z
f (x) 3
dx ' [f ( ) + f (0) + f ( )].
1 1x 2 3 2 2

The Error Term: Take k = 2 in (1).

1
f (6)() (4x3 3x)2
Z
1
E(I) = 2
dx
6! 1 1x 2 4

1
f (6)() (16x6 24x4 + 9x2) f (6)()
Z
= dx = 5 .
16 6! 1 1x 2 2 6!

Dr. Raj Kumar, VBSPU Jaunpur. 64


Example

Using three-point Gauss-Chebyshev quadrature formula, evaluate the


Z 1
(1 + x)ex
integral dx
1x 2
1

Solution:

1 x

3
Z
(1 + x)e 3
dx = [f ( )+f (0)+f ( )] = [0.7574258+1+3.24338]
1 1 x2 3 2 2 3

= 5.23683

Dr. Raj Kumar, VBSPU Jaunpur. 65


Laguerre Polynomials

[a, b] = (0, )

w(x) = ex
n
d
pn(x) = ex n (xnex) for n = 0, 1, 2, .
dx
weight function w(x) = ex.

Therefore, we can use these polynomials to evaluate the integrals of


Z
the form: exf (x)dx
0

Dr. Raj Kumar, VBSPU Jaunpur. 66


First few Laguerre Polynomials

p0(x) = 1

p1(x) = (x 1)

p2(x) = x2 4x + 2

p3(x) = x3 + 9x2 18x + 6

Dr. Raj Kumar, VBSPU Jaunpur. 67


2-point Gauss- Laguerre Rule

Take k=1 and proceed in the similar as in Gauss-Legendre case, we

can prove that

f (4)()
Z
x 1
e f (x)dx = [(2+ 2)f ((2 2))+(2 2)f ((2+ 2))]+ .
0 4 6

Remrark: In the similar way one can derived three-point Gauss-

Laguerre Rule.

Dr. Raj Kumar, VBSPU Jaunpur. 68


Example

R ex sin(2x)
Evaluate using 2-point Gauss-Laguerre quadrature. 0 1+x2
dx

sin(2x)
Solution: Here f (x) = 1+x2


ex sin(2x)
Z
1
dx ' [(2 + 2)f (2 2) + (2 2)f (2 + 2)]
0 1 + x2 4

1
' [3.4142 f (0.585786) + 0.585786 f (3.4142)]
4
= 0.5915

Dr. Raj Kumar, VBSPU Jaunpur. 69


Hermite Polynomials

[a, b] = (, )

x2
w(x) = e
n
n x d 2
x2
pn(x) = (1) e (e ) for n = 0, 1, 2, .
dxn
x2
Since w(x) = e , therefore this orthogonal sequence of polynomials
Z
2
can be used to evaluate integrals of the form ex f (x)dx.

Dr. Raj Kumar, VBSPU Jaunpur. 70


First few Hermite Polynomials

Example: P0(x) = 1,

P1(x) = 2x,

P2(x) = 4x2 2,

P3(x) = 8x3 12x

Dr. Raj Kumar, VBSPU Jaunpur. 71


Three -point Gauss-Hermite Quadrature Rule

This is the case when k = 2.

We take p3(x), the Hermite polynomial of degree 3.

To find the roots of p3(x) = 0 :


q q
8x3 12x = 0 = x = 32 , 0, 32
q q
Hence x0 = 32 , x1 = 0, and x3 = 32

After computing A0, A1, A2, the three-point Gauss-Hermite Rule takes

the form:

Dr. Raj Kumar, VBSPU Jaunpur. 72


Z r r
x2 3 3
e f (x)dx ' [f ( ) + 4 f (0) + f ( )]
6 2 2

Error term: Take k = 2 in the equation (1).


f (6)() (8x2 12)2
Z
2
E(I) = ex dx
6! 82
(6)

f () 3 (6)
= = f ().
6! 4 960

Dr. Raj Kumar, VBSPU Jaunpur. 73


Example

Evaluate the integral


Z x2
e
2
dx
1 + x

, using three-point Gauss-Hermite quadrature

1
Solution: Here f (x) = 1+x2
.

q q

I(f ) ' 6 [f ( 32 ) + 4 f (0) + f ( 32 )] = 54 .

Dr. Raj Kumar, VBSPU Jaunpur. 74


Some Properties of orthogonal Sequence of

Polynomials

Let w(x) > 0 for all x.

Let p0(x), p1(x), p2(x), , pk (x), be be a sequence of orthogonal

polynomials w.r.to w(x). Then

Property: 1

If p(x) is any polynomial of degree k then p(x) can be uniquely written

as p(x) = d0p0(x) + d1p1(x) + + dk pk (x)

Dr. Raj Kumar, VBSPU Jaunpur. 75


Property: 2

If p(x) is any polynomial of degree < k then p(x) is orthogonal to pk (x).


Rb
That is a pk (x)p(x)w(x)dx = 0.

Property: 3

Each pk (x) has has k distinct real zeros all lie in [a, b].

Dr. Raj Kumar, VBSPU Jaunpur. 76


Method of Least squares

Problem: Given any n distinct points on X-Y plane say (xi, yi), i =

1, 2 n, we want to determine a continuous function f (x) ( For

example, polynomials, exponential functions, sine and cosine functions)

such that f (xi) yi, i = 1, 2 n.

Dr. Raj Kumar, VBSPU Jaunpur. 1


Use of linear polynomials (straight lines)

The straight line y = a + bx should be fitted through the points so that

the sum of the squares of the distances of these points from the that

straight line is minimum, where the distance is measured in the vertical

direction.

Dr. Raj Kumar, VBSPU Jaunpur. 2


Example

Given points are (1.3, .103), (.1, 1.099), (.2, .808), (1.3, 1.897). Use

method of least squares, fit a straight line.

Sol:

X X X X
n = 4, xi = .1 , x2i = 3.43 yi = 3.907, xiyi = 2.3839

After solving system of linear equation, we obtain a = .9601, b = .6670.

Hence the straight line is

y = .9601 + .6670x

Dr. Raj Kumar, VBSPU Jaunpur. 3


Use of polynomial of degree 1

Let f (x) = a0 + a1x + a2x2 + anxn. Note that here we have n + 1

unknown. Let us consider the case of n = 2.

Example: For points are (0, 1), (.25, 1.2840), (.50, 1.6487),

(.75, 2.1170), (1, 2.7183). Use method of least squares, fit quadratic

polynomial.

Dr. Raj Kumar, VBSPU Jaunpur. 4


Solution:

let f (x) = a0 + a1x + a2x2, then the corresponding normal equations are

5a0 + 2.5a1 + 1.875a2 = 8.768 (1)

2.5a0 + 1.875a1 + 1.5625a2 = 5.4514 (2)

1.875a0 + 1.5625a1 + 1.3828a2 = 8.768 (3)

After solving the above system of equations, we get a0 = 1.0051, a1 =

.86468, a2 = .84316

Dr. Raj Kumar, VBSPU Jaunpur. 5


Other functions

It may also happened that data are exponentialy related i.e.,

y = cedx.

In this case, we have to take f (x) = aebx. Here, we have to solve system

of nonlinear equations.

OR

We can write

lnf (x) = lna + bx. Now using the method of straight line we can find

lnf (x).

Dr. Raj Kumar, VBSPU Jaunpur. 6


Example

For points are (1, 5.10), (1.25, 5.59), (1.50, 6.53), (1.75, 7.45), (2, 8.46).

Use method least squares, fit a exponential function.

Sol: f (x) = aebx or lnf (x) = lna + bx

Using the same procedure, which have been used in case of straight

line, we obtain

b = .5056, lna = 1.122.

Hence,

lnf (x) = 1.222 + .5056x or f (x) = e1.222e.5056x = 3.071e.5056x

Dr. Raj Kumar, VBSPU Jaunpur. 7


Numerical Differentiation

Basic problems

Derive a formula that approximates the derivative of a function in

terms of linear combination of function values ( Function may be

known )

Approximate the value of a derivative of a function defined by discrete

data.

Dr. Raj Kumar, VBSPU Jaunpur. 8


Solution Approaches ..

Use Taylor Series Expansion.

Pass a polynomial through the given data and differentiate the

interpolating polynomial.

Applications

To solve Ordinary and Partial Differential Equations.

Dr. Raj Kumar, VBSPU Jaunpur. 9


First Derivative..

Let f : [a, b] R, then the derivative of f is another function say


0
f : [a, b] R and defined by

f (c + h) f (c)
f 0(c) = lim , c (a, b).
h0 h

Geometrically Speaking f 0(c) is the slope of tangent to the curve f (x)

at x = c.

Dr. Raj Kumar, VBSPU Jaunpur. 10


Taylor Series

Derivative of a function at x = x0

Suppose f has two continuous derivatives. Then, by Taylors Theorem

h2 00
0
f (x0 + h) = f (x0) + hf (x0) + f ()
2

where (x0, x0 + h). Now,

f (x0 + h) f (x0)
f 0(x0)
h

Called Forward Formula

Dr. Raj Kumar, VBSPU Jaunpur. 11


Error

Error = |true value approximate value|

h
|ED (f )| max |f ()|
[a,b] 2

Example

Example1. Using Taylor series find the derivative of f (x) = x2 at x=1,

with h = .1. Also compute the error.

Dr. Raj Kumar, VBSPU Jaunpur. 12


Other formulae

Backward Formula

f (x0 + h) f (x0)
f 0(x0)
h

Central Formula

f (x0 + h) f (x0 h)
f 0(x0)
2h

Similarly, we can drive (Second Derivative )

00 f (x0 + h) 2f (x0) + f (x0 h)


f (x0)
h2

Dr. Raj Kumar, VBSPU Jaunpur. 13


Derivative for discrete data using interpolating

polynomial

x x0 x1 x2 ... xn

f (x) f (x0) f (x1) f (x3) ... f (xn)

Assumption: x0, x1 xn are equispaced i.e., xi xi1 = h. Where

the explicit nature of the function f is not known.

Remark 1: We can used Newtons Forward or Backward formula

depends the location of the point

Remark 2: If data is not equispaced then Lagrange interpolating

polynomial can be used.

Dr. Raj Kumar, VBSPU Jaunpur. 14


Using Newtons forward difference formula

s(s 1) 2 s(s 1)(s 2) 3


f (x) Pn(x) = f (x0)+sf (x0)+ f (x0)+ f (x0)
2! 3!
s(s 1)(s 2) (s n + 1) n
f (x0)
n!

where

x = x0 + sh

We use pn(x) to calculate the derivatives of f.

That is f 0(x) ' p0n(x) for all x [x0, xn].

Dr. Raj Kumar, VBSPU Jaunpur. 15


For a given x,

f 0(x) ' p0n(x)


dpn ds
=
ds dx
1 dpn
=
h ds

Similarly,
2
00 d pn
f (x) '
dx2
d dpn
( )
dx dx
d dpn ds
= ( )
dx ds dx
1 d dpn
= ( )
h dx ds
Dr. Raj Kumar, VBSPU Jaunpur. 16
1 d2 p n 1
= ( 2 )
h ds h
1 d2 p n
= 2 2
h ds

Thus in general,
k
(k) 1 d pn
f (x) = k k
h ds

Dr. Raj Kumar, VBSPU Jaunpur. 17


Example 1

Using Taylor series expansion (forward formula) and Newton forward

divided difference, compute first and second derivative at x = 2 for the

following tabulated function

x 1 2 3 4

f (x) 2 5 7 10

Dr. Raj Kumar, VBSPU Jaunpur. 18


Solution

x f (x) 2 3

1 2

2 5 -1

2 2

3 7 1

4 10

Dr. Raj Kumar, VBSPU Jaunpur. 19


Here h = 1 Using Taylor series

f (2 + h) f (2) f (3) f (2)


f 0(2) = = =2
h 1

00 f (2 + h) 2f (2) + f (2 h) f (3) 2f (2) + f (1)


f (2) = 2
= = 1
h 1

Dr. Raj Kumar, VBSPU Jaunpur. 20


Using Newton forward divided difference formula

s(s 1) 2 s(s 1)(s 2) 3


f (x) Pn(x) = f (x0)+sf (x0)+ f (x0)+ f (x0)
2! 3!
2
 
1 dpn 1 2s 1 2 3s 6s + 2 3
f 0(x) = f (x0) + f (x0) + f (x0)
h ds h 2! 3!
Here x = 2, x0 = 1 , s = 1 and h = 1

0 1 1
f (2) = 3 = 13/6
2 3

00 1 d2 p n 1  2 3

f (x) 2 2 = 2 f (x0) + (s 1) f (x0)
h ds h
f 00(2) = 1

Dr. Raj Kumar, VBSPU Jaunpur. 21


Example 2

Calculate f (4)(0.15)

x 0.1 0.2 0.3 0.4 0.5 0.6

f (X) 0.425 0.475 0.400 0.450 0.525 0.575

Solution:

Newtons forward difference formula:

2 3 2
1 s s 2 s 3s + 2s 3
p5(x) = f (x0) + s4 f (x0) + 4 f (x0) + 4 f (x0)+
2 6

s4 6s3 + 11s2 6s 4 s5 10s4 + 35s3 50s2 + 24s 5


+ 4 f (x0) + 4 f (x0)
24 120
Dr. Raj Kumar, VBSPU Jaunpur. 22
Differentiating this 4-times we get,

d4f dp45 1 4 1 5
' = [4 f (x 0 ) + (5s 10)4 f (x0)
dx4 dx4 h4 5

1 4
= 4 [4 f (x0) + (s 2)45f (x0)]
h

1 2
= [035 + (0.5 2)(0.4)] = 95.00 10
(0.1)4

Dr. Raj Kumar, VBSPU Jaunpur. 23


x f (x) 41 f 42 f 43 f 44 f 45 f

0.1 0.425

0.050

0.2 0.475 -0.125

-0.075 0.25

0.3 0.400 0.125 -0.35

0.050 -0.100 0.4

0.4 0.450 0.025 0.05

0.075 -0.05

0.5 0.525 -0.025

0.050

0.6 0.575 Dr. Raj Kumar, VBSPU Jaunpur. 24


The Basic Problem

Given a set of tabular values ( available from an experiment) of a

function f .

X x0 x1 x2 ... xn

f (X) f (x0) f (x1) f (x3) ... f (xn)

Where the explicit nature of the function f is not known.

To find a function such that (xi) = f (xi) for all 0 i n.

Such a function is called an interpolating function.

Dr. Raj Kumar, VBSPU Jaunpur. 25


Geometrically

The problem is :

To find a function / polynomial whose graph passes through the given

set of (n + 1) points,

(x0, f (x0)), (x1, f (x1)), , (xn, f (xn)).

Dr. Raj Kumar, VBSPU Jaunpur. 26


Polynomial interpolation

If is a polynomial then the process is called polynomial interpolation

and is called an interpolating polynomial.

Dr. Raj Kumar, VBSPU Jaunpur. 27


Some types of interpolation

Polynomial interpolation

Piecewise Polynomial( Spline ) interpolation

Rational interpolation

Trigonometric interpolation

Exponential interpolation

We study : Polynomial and Piecewise polynomial interpolations.

Dr. Raj Kumar, VBSPU Jaunpur. 28


Example

Find a polynomial which fits the following data.

X x0 x1 x2

f (X) f (x0) f (x1) f (x2)

Dr. Raj Kumar, VBSPU Jaunpur. 29


Example

Solution:

(x x1)(x x2) (x x0)(x x2)


P3(x) = f (x0) + f (x1)
(x0 x1)(x0 x2) (x1 x0)(x1 x2)
(x x0)(x x1)
+ f (x2).
(x2 x0)(x2 x1)

This is a polynomial of degree 2.

Called Lagranges interpolation formula.

Dr. Raj Kumar, VBSPU Jaunpur. 30


Lagranges Interpolation formula

Similarly,

(x x1)(x x2) (x xn)


pn(x) = f (x0)
(x0 x1)(x0 x2) (x0 xn)

(x x2)(x x3) (x xn)


+ f (x1)
(x1 x0)(x1 x2) (x1 xn)

(x x0)(x x1) (x xn1)


+ + f (xn)
(xn x0)(xn x1) (x1 xn1)
is the unique polynomial of degree n which interpolates f at the

n + 1 points x0, x1, , xn.

Dr. Raj Kumar, VBSPU Jaunpur. 31


Lagranges Interpolation formula

If we write

(x x0)(x x1) (x xi1)(x xi+1) (x xn)


li(x) =
(xi x0)(xi x1) (xi xi1)(xi xi+1) (xi xn)

then
Pn
pn(x) = i=0 li (x)f (xi )

is the unique(how?) polynomial of degree n which interpolates f at

the n + 1 points x0, x1, , xn.

Dr. Raj Kumar, VBSPU Jaunpur. 32


We proved:

Theorem 1. (Existence and Uniqueness theorem for interpolating

polynomials.) If x0, x1, x2, , xn are n + 1 distinct numbers and f

is a function whose values are given at these numbers, then a unique

polynomial p(x) of degree n with p(xi) = f (xi) for all i = 1, 2, , n.

This polynomial is given by

Pn
p(x) = i=0 li (x)f (xi )

Dr. Raj Kumar, VBSPU Jaunpur. 33


Thus we have:

Given x0, x1, x2, x3, , xn and f (x0), f (x1), , f (xn), we have:

Lagranges polynomial Degree at most interpolates f at

p1(x) 1 x0, x1

p2(x) 2 x0, x1, x2

p3(x) 3 x0, x1, x2, x3


.. .. ..

pn(x) n x0, x1, x2, x3, , xn

Dr. Raj Kumar, VBSPU Jaunpur. 34


Example

Find the interpolating polynomial in Lagrangian form for the data.

x -2 -1 1 3

f (x) -15 -4 0 20

Solution:

We calculate p3(x).

p3(x) = l0(x).f (x0) + l1(x).f (x1) + l2(x).f (x2) + l3(x).f (x3)

Dr. Raj Kumar, VBSPU Jaunpur. 35


(xx1 )(xx2 )(xx3 ) (x+1)(x1)(x3)
Where l0(x) = (x0 x1 )(x0 x2 )(x0 x3 ) = (1)(3)(5)

1 3
= (x 3x2 x + 3)
15

Similarly,
1 3
l1(x) = (x 2x2 5x + 6)
8
1 3
l2(x) = (x 7x 6)
12
1
l3(x) = (x3 + 2x2 x + 2)
40

Therefore,

1 3 1
p3(x) = 15{ (x 3x2 x + 3)} 4{ (x3 2x2 5x + 6)}
15 8
Dr. Raj Kumar, VBSPU Jaunpur. 36
1 3
+0 + 20{ (x + 2x2 x + 2)}
40
= x3 x2 + x 1.

Error: Let f : [a, b] R

En(x) = f n+1()(x x0)(x x1) (x xn)

For some (a, b).

Problem: We do not know , hence it would be difficult to compute error

with this formula. However, this can be used in order to find the upper

bound of the error, i,e., we have a number in hand such that error should

not exceed by this number.

Dr. Raj Kumar, VBSPU Jaunpur. 37


A disadvantage

If more data become available, the work performed to generate the

original Lagrange form cannot be reused to compute a higher-degree

polynomial.

This can be rectified by writing the Lagranges interpolation polynomial

in Newtons form.

For this we use the idea of divided differences.

Dr. Raj Kumar, VBSPU Jaunpur. 38


First order divided differences

f (x1) f (x0)
f [x0, x1] =
x1 x0
f (x2) f (x1)
f [x1, x2] =
x2 x1
.. ..

In general,
f (xi+1) f (xi)
f [xi, xi+1] =
xi+1 xi
.. ..

f (xn) f (xn1)
f [xn1, xn] =
xn xn1

Dr. Raj Kumar, VBSPU Jaunpur. 39


Second order divided differences

f [x1, x2] f [x0, x1]


f [x0, x1, x2] =
x2 x 0

f [x2, x3] f [x1, x2]


f [x1, x2, x3] =
x3 x 1
.. ..
f [xi+1, xi+2] f [xi, xi+1]
f [xi, xi+1, xi+2] =
xi+2 xi
.. ..
f [xn1, xn] f [xn1, xn]
f [xn2, xn1, xn] =
xn xn2

Dr. Raj Kumar, VBSPU Jaunpur. 40


Divided differences of nth( 3) order

Can be defined recursively in a similar way.

f [xi+1, ,xi+n ] f [xi, xi+1, ,xi+n1 ]


f [xi, xi+1, , xi+n] =
xi+n xi

The numbers f (x0), f (x1), , f (xn) are called divided differences of

0-th order and are denoted by f [x0], f [x1], , f [xn] respectively.

We say: f [x0, x1, , xn] as the divided difference of f at the points

x0, x1, , xn.

Dr. Raj Kumar, VBSPU Jaunpur. 41


Divided differences in Tabular form (when n= 3.)

x f (x) First order Second order Third order

x0 f [x0]

f [x0, x1]

x1 f [x1] f [x0, x1, x2]

f [x1, x2] f [x0, x1, x2, x3]

x2 f [x2] f [x1, x2, x3]

f [x2, x3]

x3 f [x3]

Dr. Raj Kumar, VBSPU Jaunpur. 42


Example

Consider the data:

x -2 -1 1 3

f (x) -15 -4 0 20

Dr. Raj Kumar, VBSPU Jaunpur. 43


x f (x) f [, ] f [, , ] f [, , , ]

-2 -15

11

-1 -4 -3
The divided difference table:
2 1

1 0 2

10

3 20

Dr. Raj Kumar, VBSPU Jaunpur. 44


The Newtons form of a polynomial

pn(x) = A0+A1(xx0)+A2(xx0)(xx1)+ +An(xx0)(xx2) (xxn)

We need to compute Ai, i = 0, 1 n.

This can be done easily by using

f (xi) = pn(xi), i = 0, 1 n

Can you see a relation between Ai and divided difference?

Dr. Raj Kumar, VBSPU Jaunpur. 45


Idea

We prove:

Ak = f [x0, x1, , xk ]

for all k = 0, 1, 2, , n.

Dr. Raj Kumar, VBSPU Jaunpur. 46


Thus the Newtons form of pn(x) is:

pn(x) = f [x0]

+f [x0, x1](x x0)

+f [x0, x1, x2](x x0)(x x1)

+f [x0, x1, x2, x3](x x0)(x x1)(x x2)

+ f [x0, x1, , xn](x x0)(x x1)(x x2) (x xn1)

Look at the divided difference table and recognize the coefficients.

Thus the divided difference table helps us to write the interpolation

Dr. Raj Kumar, VBSPU Jaunpur. 47


polynomial easily.
x f (x) First order Second order Third order

x0 f [x0]

f [x0, x1]

x1 f [x1] f [x0, x1, x2]

f [x1, x2] f [x0, x1, x2, x3]

x2 f [x2] f [x1, x2, x3]

f [x2, x3]

x3 f [x3]

Dr. Raj Kumar, VBSPU Jaunpur. 48


Example

Consider the data:

x -2 -1 1 3

f (x) -15 -4 0 20

Dr. Raj Kumar, VBSPU Jaunpur. 49


x f (x) f [, ] f [, , ] f [, , , ]

-2 -15

11

-1 -4 -3
The divided difference table:
2 1

1 0 2

10

3 20

p3(x) = 15 + 11(x + 2) 3(x + 2)(x + 1) + 1(x + 2)(x + 1)(x 1)

= x3 x2 + x 1.

Dr. Raj Kumar, VBSPU Jaunpur. 50


When the points x0, x1, , xn are equispaced.

While forming the divided difference table, we look for the correct

division, and there is chance of mistake. Did you realize this?. Let

xi+1 xi = h for all i = 0, 1, 2, , n 1.

The Newtons divided difference formula can be further simplified to

Newtons forward difference form.

We first define the forward differences.

Dr. Raj Kumar, VBSPU Jaunpur. 51


Forward differences

First order forward differences:

41f (x0) = f (x1) f (x0)

41f (x1) = f (x2) f (x1)

.. ..

41f (xi) = f (xi+1) f (xi)

.. ..

41f (xn1) = f (xn) f (xn1)

Dr. Raj Kumar, VBSPU Jaunpur. 52


We call 41f (xi) as: The first order forward difference of f at xi.

Forward differences of higher order:

can be computed using the recursive following formula.

If n > 1 then we define,

4nf (xi) = 4n1f (xi+1) 4n1f (xi)

Dr. Raj Kumar, VBSPU Jaunpur. 53


Forward difference table

Consider the data:

x -2 -1 0 1

f (x) -15 -4 0 20

Dr. Raj Kumar, VBSPU Jaunpur. 54


x f (x) 41 f 42f 43 f

-2 -15

11

-1 -4 -7
The forward difference table:
4 23

0 0 16

20

1 20

Dr. Raj Kumar, VBSPU Jaunpur. 55


Divided differences in terms of forward differences

Suppose f : [a, b] R is given at the n + 1 equispaced points

x0, x1, , xn with spacing h = xi+1 xi.

f (x1) f (x0) 1
f [x0, x1] = = 4f (x0)
x1 x 0 h

f [x1, x2] f [x0, x1]


f [x0, x1, x2] =
x2 x 0
1 4f (x1) 4f (x0) 1 2
= [ ] = 2 4 f (x0)
2h h 2h
In general,
1 k
f [x0, x1, , xk ] = 4 f (x0).
k! hk

Dr. Raj Kumar, VBSPU Jaunpur. 56


Newtons forward difference formula.

Suppose f : [a, b] R is given at the n + 1 equispaced points

x0, x1, , xn with spacing h = xi+1 xi.

xx0
Given x, let s = h .

Thus x can be written in the form x = x0 + sh for some s

In particular, xi = x0 + ih for all i = 0, 1, 2, , n.

Thus

x xi = x0 + sh (x0 + ih) = (s i)h

Dr. Raj Kumar, VBSPU Jaunpur. 57


Now the interpolating polynomial pn(x) can be written as:

pn(x) = pn(x0 + sh) = f [x0]

+sh f [x0, x1]

+s(s 1)h2 f [x0, x1, x2]

+s(s 1)(s 2)h3f [x0, x1, x2, x3]

+ s(s 1)(s 2) (s n + 1)f [x0, x1, , xn]

n
X
= s(s 1)(s 2) (s k + 1)f [x0, x1, , xk ]
k=0

Dr. Raj Kumar, VBSPU Jaunpur. 58


n  
X s
= k! hk f [x0, x1, , xk ]
k
k=0
n  
X s
= 4k f (x0)
k
k=0

Where
 
s s(s 1) (s k + 1)
= .
k k!

Dr. Raj Kumar, VBSPU Jaunpur. 59


Example

Prepare the forward difference table for the following data. Using

Newtons forward interpolating polynomial , find approximate value of

f (0.1).

x 0 0.2 0.4 0.6 0.8

f (x) 0.12 0.46 0.74 0.90 1.2

Dr. Raj Kumar, VBSPU Jaunpur. 60


x f (x) 41 f 42f 43 f 44 f

0 0.12

0.34

0.2 0.46 -0.06

0.28 -0.06

0.4 0.74 -0.12 0.32

0.16 0.26

0.6 0.90 0.14

0.30

0.8 1.2

Dr. Raj Kumar, VBSPU Jaunpur. 61


To interpolate at x = 0.1

x x0 0.1 0 1
s= = = .
h 0.2 2

Therefore the binomial coefficients are:

  1 1
( 1)
   
s s 1 s 1
= 1, = , =2 2 = .
0 1 2 2 1.2 8

  1 1 3
s s(s 1)(s 3) 2 ( 2 )( 2) 1
= = = .
3 1.2.3 1.2.3 16

Dr. Raj Kumar, VBSPU Jaunpur. 62


  1 1 3 5
s s(s 1)(s 2)(s 3) 2 ( 2 )( 2 )( 2) 5
= = = .
4 1.2.3.4 1.2.3.4 128

Therefore

     
s s s
p4(x0+sh) = f (x0)+s41f (x0)+ 42f (x0)+ 43f (x0)+ 44f (x0).
2 3 4

1 1 1 5
= 0.12 + (0.34) + (0.06) + (0.06) + (0.32) = 0.28125
2 8 16 128

Hence f (0.1) 0.28125

Dr. Raj Kumar, VBSPU Jaunpur. 63


Piece-wise polynomial interpolation

Polynomials with higher degree grow / decay very fast.

When the number of tabular points is more in number, the interpolating

polynomial would be of higher degree and hence there is a chance of

getting more error.

Therefore,

We divide the given interval into small subintervals and on each small

subinterval we fit a polynomial of lower degree.

Dr. Raj Kumar, VBSPU Jaunpur. 64


Piece-wise polynomial interpolation

We study:

1. Piecewise linear interpolation.

2. Piecewise quadratic interpolation.

Dr. Raj Kumar, VBSPU Jaunpur. 65


Error on Piecewise linear interpolation

Assume that the tabular points are equispaced with spacing h = xi+1

xi .
i h2 h 00
Error = max f ()
(a,b) 8

Therefore
M h2
E .
8
00
M = max f ()
(a,b)
ba
h=
n

Dr. Raj Kumar, VBSPU Jaunpur. 66


Example

What should be the minimum number of tabular points required for the

piecewise linear interpolation for f (x) = cos(x) on [0, ], such that error

does not exceed by 1/2 106 ?

Solution:

Given f (x) = cos x on [0, ].

Here a = 0 and b =

ba
Then h = n = n

Dr. Raj Kumar, VBSPU Jaunpur. 67


f 00(x) = cos x

We know

M = max |f 00(x)| = 1
0x

Now choose h such that:

h2M 1
106.
8 2

That is,

h2 4 106

Dr. Raj Kumar, VBSPU Jaunpur. 68


That is

h 2 103

Therefore

n= 103 = 1570.7
n 2

The number of subdivisions required n = 1571.

Total number of tabular points required n + 1 = 1572.

Dr. Raj Kumar, VBSPU Jaunpur. 69


Piecewise quadratic interpolation

In this case the interval [a, b] is divided into 2n equal subintervals.

Therefore
ba
h= .
2n

We write these 2n + 1 points as x0, x1, x2n.

Dr. Raj Kumar, VBSPU Jaunpur. 70


The piecewise interpolating polynomial may not be

smooth.

The piecewise interpolating polynomial constructed as in the above

examples may fail to be smooth/ differentiable at the tabular points.

We use: Cubic Spline functions

Dr. Raj Kumar, VBSPU Jaunpur. 71


We Use the interpolating polynomial

To estimate the missing data.

OR

To approximate f at some intermediate points.

Dr. Raj Kumar, VBSPU Jaunpur. 72


Workshop on Non-Verbal
Communication

Dr. Anurag Singh


Institute of Management Studies
Banaras Hindu University
Non-Verbal Communication
n Nonverbal communication is the
process of communication through
sending and receiving wordless (mostly
visual) cues between people.
Contents:
n OCULESICS: Gaze & Eye Contact
n Paralanguage or Meta-communication
n PROXEMICS: Interpersonal Attitude
n PROXEMICS use of Space, Distance & Proximity
n HAPTICS: Contact vs Non Contact
n CHRONEMCIS: Time Perception
n KINESICS: Body Language
n Gestures
n Orientation
n Body Posture
n Bodily Communication
n Non grammatical Signals
OCULESICS: Gaze & Eye
Contact
n Gaze is the term used to mean looking at a
person

n Eye-contact means mutual gaze, where the


two look at each other at the same time

n Maintaining eye contact signals genuineness

n Avoiding it signals dishonesty


Types of Eye Contact

n Very Direct eye contact: Middle Easterners,


Some Latin Americans, French

n Moderate eye contact: Americans, Northern


European, British

n Minimal eye contact: East Asians, Southeast


Asians, East Indians, Native Americans
What is Paralanguage or
Meta-communication?
n Voice - 'vibrant', 'thick' or 'flat'
n Voice Dynamics
n Intonation (Pitch) no two intonations are alike
n Rhythm accent certain syllables more definitely, lengthen
more vowels, shorten unaccented vowels
n Continuity of Speech speak brokenly, in uneasy plaster of
word groups, speak continuously
n Speed of Speech speak above or below certain socialized
speeds
n Pronunciation
n Vocabulary Personality reflected in choice of words
n Style
Types of Intonation
n Curious: rising pitch when asking question
n Emotional: wide range of emotions
n Grammatical: clarity of 'asking' & 'telling'
n Information structure: convey what is new and
what is already known in an utterance
n Textual: pitch rises at beginning of new item & then
gradually fall
n Psychological: information in portion' more easily
perceived and memorized
PROXEMICS: Interpersonal
Attitude
Attitude:
n Aggressive vs Friendly

n Dominant vs Submissive

Physical Contact:
n Shaking hands, touching, holding, embracing,

pushing, or patting
n Reflect an element of intimacy or

n Feeling of (or lack of) attraction.


PROXEMICS use of Space,
Distance & Proximity
n The distance conveys a non-verbal message.
n In some cultures it is a sign of attraction,

n In others it may reflect status or the intensity of

the exchange
Proximity in white Anglo Saxon cultures:
n Intimate Space less than 18 inches

n Personal Space 18 inches to 4 feet

n Social/Consultative Space 4 - 12 ft

n Public Space more than 12 ft


HAPTICS: Contact vs Non Contact
Use of physical contact when communicating

Non Contact: Contact:


n Arabs: Iraq, Kuwait, Saudi n Asian: China, Indonesia,
Arabia, Syria, United Arab
Japan, Philippines, Thailand;
Republic
Indians and Pakistanis
n Latin Americans: Bolivia,
Cuba, Equator, El Salvador, n Northern Europeans:
Mexico, Paraguay, Peru, Australia, England, Germany,
Puerto Rico, Venezuela; Netherlands, Norway,
Scotland;
n Southern Europeans:
French, Italy, Turky n Americans
CHRONEMCIS: Time Perception
n Americans use M time: n Middle East & Latin
America use P time:

n monochromatic time is n polychromatic


characterized by doing one time is characterized
thing at a time, by several things
concentrate on the job
going on at once
KINESICS: Body Language
n Watching people's actions can bring you
a lot closer to the truth than merely
listening to what they say (which might
be a cover-up)
n Science" of kinesthetic, or "body
language"
n Can be very informative
n Outward expressions of inner feelings.
KINESICS: Body Language
n Openness: Open hands, unbuttoned coat.
n Defensiveness: Arms crossed, sideways
glance, touching-rubbing nose, rubbing eyes,
buttoned coat, drawing away.
n Insecurity: Pinching tissue, chewing pen,
thumb over thumb, biting fingernail.
n Cooperation: Upper body in sprinter's
position, open hands, sitting on edge of chair,
hand to face gestures, unbuttoning coat.
KINESICS: Body Language
n Confidence: Lifted hands, hands behind back,
back hard, hands in coat pockets with thumb
out, hands on collar of coat.
n Nervousness: Clearing throat, "whew" sound,
whistling, smoking, pinching tissue, move
around, covering mouth, jiggling money or
keys, pull ears, wringing hands.
n Frustration: Short breaths, "tsk" sound,
tightly compress hands, wringing hands, fist-
like gestures, rubbing hand through hair,
rubbing hand on neck.
GESTURES - Hand Shake
n Americans use a firm, solid grip
n Middle Easterners and Orientals prefer a
gentle grip a firm grip to them suggests
unnecessary aggressiveness
GESTURES Eye Contact
n Americans are taught to look directly
n Japanese and Koreans are taught to
avoid direct eye contact, direct eye
contact to them is considered a
weakness, and may indicate sexual
overtones
GESTURES O.K. Gesture
n Americans, forming a circle with thumb
and forefinger to signal O.K.
n Means
n "zero" or worthless in France
n Money in Japan
n Calling someone a very bad name in
Germany.
GESTURES
Nodding Yes or No
n For Americans, up and down means
yes, side to side means no
n In Bulgaria, the nods are reversed in
meaning.
GESTURES
Putting Feet on Table
n An American gesture is found to be
offensive to nearly every other country
around the globe
Orientation
n People may present themselves in
various ways:
n face-to-face
n side-to-side
n back-to-back
n cooperating people are likely to sit side-
by-side
n competitors frequently face one
another.
What is Position Sign of?
n Competition
What is Position Sign of?
n Cooperation
What is Position Sign of?
n Conversation
NONVERBAL INTERPRETATION
BEHAVIOR
fast, erect walk Confidence
Standing with hands on Readiness, aggression
hips
Sitting with legs Boredom
crossed, foot kicking
slightly
Sitting, legs apart Open, relaxed
Arms crossed on chest Defensiveness
Walking with hands in Dejection
pockets, shoulders
curved
NONVERBAL INTERPRETATION
BEHAVIOR
Hand to cheek Evaluation, thinking
Touching, slightly Rejection, doubt, lying
rubbing nose
Rubbing the eye Doubt, disbelief
Hands clasped behind Anger, frustration,
back apprehension
Locked ankles Uneasiness
Head resting in hand, Boredom
eyes downcast
Rubbing hands Anticipation
NONVERBAL INTERPRETATION
BEHAVIOR
Sitting with hands Confidence, superiority
clasped behind head,
legs crossed
Open palm Sincerity, openness,
innocence
Pinching bridge of nose, Negative evaluation
eyes closed
Tapping or drumming Impatience
fingers
Showing fingers Authoritative
Patting/fondling hair Lack of self-confidence;
insecurity
NONVERBAL INTERPRETATION
BEHAVIOR

Tilted head Interest

Stroking chin Trying to make a


decision
Looking down, face Disbelief
turned away

Biting nails Insecurity, nervousness

Pulling or tugging at ear Indecision


Body Posture
n It is space for the activities

n It gives evidence about the types of social


activities

n It indicates association, non-association or


dissociation with group members
What is this Posture?
n Affected
What is this Posture?
n Angry
What is this Posture?
n Curious
What is this Posture?
n Determined
What is this Posture?
n Excited
What is this Posture?
n Puzzled
What is this Posture?
n Rejecting
What is this Posture?
n Searching
What is this Posture?
n Shy
What is this Posture?
n Cautious
What is this Posture?
n Watching
What is this Posture?
n Welcoming
What is this Posture?
n True Self
What Bodily Communication
would you use to:
A listener can:
n Take the floor n actually interrupt; there
may be a brief battle,
volume being the
principal weapon
n show impatience, using
rapid head-nods, often
accompanied by verbal
signals such as 'yes',
'but' or 'well'
What Bodily Communication
would you use to:
A speaker can
n To keep the floor n raise the volume when
the other interrupts
and keep the volume
raised if the other
continues to try to
interrupt
n keep a hand in mid-
gesture at the end of
sentences
What Bodily Communication
would you use to:
n A speaker can:
n come to the end of a sentence
n To yield the floor n end by trailing off, or end with
(making Floor something open-ended like
'you know'
Productive) n end on a prolonged rising or
falling pitch
n come to the end of some of the
hand movements
accompanying speech
n gaze at the other or nod
What Bodily Communication
would you use to:
n A listener can
n To decline an offer
n grunt or make 'uh-huh'
of the floor
noises
n complete the sentence
n briefly request
clarification
n briefly restate what the
speaker said
What Non-grammatical
signals would you use?

n For Emphasis To be persuasive


when you are n Gaze more

speaking: n More gestures & head-


nods
n More facial activity

n Higher speed of delivery

n Higher volume & louder

n Hesitate less
What Non-grammatical
signals would you use?
To frame what
n To frame what you you are saying
are saying: use:
n Tone of voice

n Speed

n Pitch

n bodily movements
quote or roll
eyes
What Non-grammatical
signals would you use?
To give feedback
n To provide use:
feedback to your n encouraging
audience when n head-nods
you are speaking n Smiles
n 'uh-huh' sounds etc
n more negative
n Frowns (making face)
n Expressions
n gaze
What Non-grammatical
signals would you use?
To synchronize
n To provide use:
synchronization so as n period of
to match style of adjustment of
speech with other non-verbal signals
party to enable
adjustment of
your styles
So What Does This Mean?
n Let me see!
n Authoritative
n Pondering
n Thinking

n Considering
So What Does This Mean?
n Can I help!
n Trust me!

n Youre in good
hands!
n Helping Hand
So What Does This Mean?
n I cant take it
anymore!
n It hurts so

much!
n Pain
So What Does This Mean?
n So what is
going on?
n I just want all

the facts!
n Suspicious
So What Does This Mean?
n What?
n So that is

what you
were thinking?
n Curious
So What Does This Mean?
n I challenge
you!
n Defensive

n Self-Protective

n Not innocent
So What Does This Mean?
n What do you
mean!
n I am not

going to do
this!
n Angry
So What Does This Mean?
n Wanna fight?
n Aggressive
So What Does This Mean?
n Oh boy, the
cameras are
on me now!
n Unhappy

n Disappointed
So What Does This Mean?
n I am so!
n I can do
everything
better than you!
n Confident
So What Does This Mean?
n Its not fair!
n I want it!

n I will make you

unhappy till I
get it!
So What Does This Mean?
n Oh Brother!
n Thoughtful

n Concentrating
So What Does This Mean?
n I dare you!
n Cold firmness

n Masking

feelings
So What Does This Mean?
n 1: Oh really must I
be here? Why do I
have to sit next to
her? Ive got it!
n 2: Worried. This guy
to my right is
strange.
n 3: Can I pull it off?
This is important to
my future!
So What Does This Mean?
n OH LORD!
n WHY?

n I cant take

this, it is too
much!
So What Does This Mean?
n This guy is
weird!
n Brush him off

and I am free!
n Leave me

alone!
So What Does This Mean?
n Yuck!
n Well this is

what she
deserves!
n Rejecting
So What Does This Mean?
n I will get
through this!
n Determined

n I will not

budge!
So What Does This Mean?
n 1: Im in
charge here!
n Dominating

n Threatening

n 2: Fearful
So What Does This Mean?
n Wow!
n Shocked

n Surprised

n Bad Hair Day!


So What Does This Mean?
n So Let me
see? It was
like this.
n I will give you

what you
want!
So What Does This Mean?
n Now just stop
that!
n Get out of
here!
n Defensive

n Oppositional
So What Does This Mean?
n So tell me
more!
n Open

n Accepting

n Welcoming
So What Does This Mean?
n Oh Yeah!
n Yummy!

n Pleasuring

n Enjoying

n Orally
passionate!
So What Does This Mean?
n So You want
what?
n Tense

n Suspicious

n Hesitant
So What Does This Mean?
n We girls do
have fun!
n We enjoy each
others
company!
n Rapport
n Sociable
So What Does This Mean?
n No! Leave me
alone!
n Oh stop it!

n Withdrawn
Queries?
Non-verbal communication
for interview effectiveness

DR . TA N UJ N A N DAN
MNNIT ALLAHABAD
Friday, September 30, 2016 DR. TANUJ NANDAN, MNNIT ALLAHABAD 2
Deciphering communication
Most of the communication that
takes place between individuals is
non-verbal.
Non-verbal communication
includes:
Tone of Voice
Facial Expressions
Movement
Appearance
Eye Contact
Gestures
Posture

Friday, September 30, 2016 DR. TANUJ NANDAN, MNNIT ALLAHABAD 3


Verbal vs. non-verbal
communication

Friday, September 30, 2016 DR. TANUJ NANDAN, MNNIT ALLAHABAD 4


Constituents of non-verbal
communication

Friday, September 30, 2016 DR. TANUJ NANDAN, MNNIT ALLAHABAD 5


Proxemics: the hidden dimension
Intimate distance for embracing, touching or whispering
Close phase - less than 6 inches (15 cm)
Far phase - 6 to 18 inches (15 - 45 cm)

Personal distance for interactions among good friends


Close phase - 1.5 to 2.5 feet (45 - 75 cm)
Far phase - 2.5 to 4 feet (75 - 120 cm)

Social distance for interactions among acquaintances


Close phase - 5 to 7 feet (1.5 - 2.1 m)
Far phase - 7 to 12 feet (2.1 - 3.6 m)

Public distance for public speaking


Close phase - 12 to 25 feet (3.6 - 7.5 m)
Far phase - 25 feet (7.5 m) or more

Friday, September 30, 2016 DR. TANUJ NANDAN, MNNIT ALLAHABAD 6


Spaces or Zones

Source: Hall, E.T. (1966). The Hidden Dimension, New York: Doubleday

Friday, September 30, 2016 DR. TANUJ NANDAN, MNNIT ALLAHABAD 7


Zones: Intimate

Friday, September 30, 2016 DR. TANUJ NANDAN, MNNIT ALLAHABAD 8


Zones: Personal

Friday, September 30, 2016 DR. TANUJ NANDAN, MNNIT ALLAHABAD 9


Zones: Social

Friday, September 30, 2016 DR. TANUJ NANDAN, MNNIT ALLAHABAD 10


Zones: Public

Friday, September 30, 2016 DR. TANUJ NANDAN, MNNIT ALLAHABAD 11


Secrets of Body Language
A large part of the initial impression that a person creates comes from body
language.
Posture, facial expression, eye contact, and gestures speak louder than the
words the individual speaks.
We all interpret body language all the time on a subconscious level.
Our face, eyes, hands (gestures), and posture express what is going on
inside of us.
They give clues to others and to us as to whether the words we say are
consistent with what we are really feeling.
Being aware of our body language can allow us to send a consistent
message.
Smiling, making eye contact, using open gestures, and using good posture
can bring up our level of self confidence.

Friday, September 30, 2016 DR. TANUJ NANDAN, MNNIT ALLAHABAD 12


Secrets of Body Language
A large part of the initial impression that a person creates comes from body
language.
Posture, facial expression, eye contact, and gestures speak louder than the
words the individual speaks.
We all interpret body language all the time on a subconscious level.
Our face, eyes, hands (gestures), and posture express what is going on
inside of us.
They give clues to others and to us as to whether the words we say are
consistent with what we are really feeling.
Being aware of our body language can allow us to send a consistent
message.
Smiling, making eye contact, using open gestures, and using good posture
can bring up our level of self confidence.

Friday, September 30, 2016 DR. TANUJ NANDAN, MNNIT ALLAHABAD 13


The Face
The face is the most expressive part of the body.

If you are feeling anxious then your facial expression may lead you to
appear aloof, disapproving, or disinterested.

You can break this misrepresentation by making a conscious effort to smile.

Your smile is one of the strongest tools you have in meeting new people.

It will help you appear warm, open, friendly, and confident.

Friday, September 30, 2016 DR. TANUJ NANDAN, MNNIT ALLAHABAD 14


Emotions Expressed Through
Facial Expressions
Just a few examples of emotions that can be expressed via facial
expressions include:
Happiness
Sadness
Anger
Surprise
Disgust
Fear
Confusion
Excitement
Desire
Contempt

Friday, September 30, 2016 DR. TANUJ NANDAN, MNNIT ALLAHABAD 15


Friday, September 30, 2016 DR. TANUJ NANDAN, MNNIT ALLAHABAD 16
Friday, September 30, 2016 DR. TANUJ NANDAN, MNNIT ALLAHABAD 17
Friday, September 30, 2016 DR. TANUJ NANDAN, MNNIT ALLAHABAD 18
Friday, September 30, 2016 DR. TANUJ NANDAN, MNNIT ALLAHABAD 19
Hands
Hands have 27 bones and are a very expressive part of our anatomy. The give
us enormous capability as an evolved species in how we handle our
environment.
Hands are very expressive. Open gestures tend to make you appear open and
honest.
By pointing your finger, or moving your hands closer together, you can draw
emphasis to what you are saying.
Used in moderation, hand gestures can make you seem enthusiastic and
committed to your topic.
Making too many gestures can make you appear nervous and uncontrolled.
Wringing your hands or touching your sleeves, face, etc. can make you appear
tense, nervous, and sometimes dishonest.

Friday, September 30, 2016 DR. TANUJ NANDAN, MNNIT ALLAHABAD 20


Hands

Friday, September 30, 2016 DR. TANUJ NANDAN, MNNIT ALLAHABAD 21


Hands

Locked Hand Gesture

Friday, September 30, 2016 DR. TANUJ NANDAN, MNNIT ALLAHABAD 22


Hands

Steepled Hand Gesture

Friday, September 30, 2016 DR. TANUJ NANDAN, MNNIT ALLAHABAD 23


Palms
Palm up: Submissive, Non-threatening Aggressive palm

Friday, September 30, 2016 DR. TANUJ NANDAN, MNNIT ALLAHABAD 24


Palms

Friday, September 30, 2016 DR. TANUJ NANDAN, MNNIT ALLAHABAD 25


Palms

Friday, September 30, 2016 DR. TANUJ NANDAN, MNNIT ALLAHABAD 26


Friday, September 30, 2016 DR. TANUJ NANDAN, MNNIT ALLAHABAD 27
Friday, September 30, 2016 DR. TANUJ NANDAN, MNNIT ALLAHABAD 28
Friday, September 30, 2016 DR. TANUJ NANDAN, MNNIT ALLAHABAD 29
Friday, September 30, 2016 DR. TANUJ NANDAN, MNNIT ALLAHABAD 30
3. Posture
The way you hold yourself, your posture, makes a big contribution to your body
language and conveys your level of self-confidence. By orienting your body
towards someone, you show attentiveness. By falling away from them or leaning
back, you show a lack of interest and some level of reserve. When we are feeling
low in confidence and want to hide away, we hunch our shoulders and keep our
heads down. When we are feeling aggressive or are trying to defend our space,
we puff ourselves up. A relaxed body posture will help you to appear and feel
more relaxed and confident.

Friday, September 30, 2016 DR. TANUJ NANDAN, MNNIT ALLAHABAD 31


4. Eyes
Our eyes give clues to our emotions. A direct stare implies intensity. It may
also mean romantic interest, aggression, or fear. Making very little eye
contact can either convey shyness or submissiveness. The middle ground of a
gaze says that you are interested, secure, and at ease.

Friday, September 30, 2016 DR. TANUJ NANDAN, MNNIT ALLAHABAD 32


Friday, September 30, 2016 DR. TANUJ NANDAN, MNNIT ALLAHABAD 33
It can work backwards, too!

Friday, September 30, 2016 DR. TANUJ NANDAN, MNNIT ALLAHABAD 34


Friday, September 30, 2016 DR. TANUJ NANDAN, MNNIT ALLAHABAD 35
http://www.howtogetyourownway.com/body_language/hands_and_ar
ms_body_language.html
http://www.palm-reading.org/hand-gestures.html

Friday, September 30, 2016 DR. TANUJ NANDAN, MNNIT ALLAHABAD 36


Lecture Notes
Simplex Method of Linear Programming

Dr. Raj Kumar

Revised: Sep 12, 2015

1 The basic steps of the simplex algorithm


Step 1: Write the linear programming problem in standard
form
Linear programming (the name is historical, a more descriptive term would
be linear optimization) refers to the problem of optimizing a linear objective
function of several variables subject to a set of linear equality or inequality
constraints.
Every linear programming problem can be written in the following stan-
dard form.

Minimize = cT x (1a)

subject to

Ax = b , (1b)
x 0. (1c)

Here x Rn is a vector of n unknowns, A M (m n) with n typically


much larger than m, c Rn the coefficient vector of the objective function,
and the expression x 0 signifies xi 0 for i = 1, . . . , n. For simplicity, we
assume that rank A = m, i.e., that the rows of A are linearly independent.
Turning a problem into standard form involves the following steps.

(i) Turn Maximization into minimization and write inequalities in stan-


dard order.
This step is obvious. Multiply expressions, where appropriate, by 1.

1
(ii) Introduce slack variables to turn inequality constraints into equality
constraints with nonnegative unknowns.
Any inequality of the form a1 x1 + + an xn c can be replaced by
a1 x1 + + an xn + s = c with s 0.
(iii) Replace variables which are not sign-constrained by differences.
Any real number x can be written as the difference of nonnegative
numbers x = u v with u, v 0.
Consider the following example.
Maximize z = x1 + 2 x2 + 3 x3 (2a)
subject to
x1 + x2 x3 = 1 , (2b)
2 x1 + x2 + 2 x3 5 , (2c)
x1 x2 4 , (2d)
x2 + x3 5 , (2e)
x1 0 , (2f)
x2 0 . (2g)
Written in standard form, the problem becomes
minimize = x1 2 x2 3 u + 3 v (3a)
subject to
x1 + x2 u + v = 1 , (3b)
2 x1 x2 2 u + 2 v + s 1 = 5 , (3c)
x1 x2 + s 2 = 4 , (3d)
x2 + u v + s 3 = 5 , (3e)
x1 , x2 , u, v, s1 , s2 , s3 0 . (3f)

Step 2: Write the coefficients of the problem into a simplex


tableau
The coefficients of the linear system are collected in an augmented matrix
as known from Gaussian elimination for systems of linear equations; the
coefficients of the objective function are written in a separate bottom row
with a zero in the right hand column.
For our example, the initial tableau reads:

2
x1 x2 u v s1 s2 s3
1 1 1 1 0 0 0 1
2 1 2 2 1 0 0 5
1 1 0 0 0 1 0 4
0 1 1 1 0 0 1 5
1 2 3 3 0 0 0 0
In the following steps, the variables will be divided into m basic variables
and n m non-basic variables. We will act on the tableau by the rules of
Gaussian elimination, where the pivots are always chosen from the columns
corresponding to the basic variables.
Before proceeding, we need to choose an initial set of basic variables
which corresponds to a point in the feasible region of the linear program-
ming problem. Such a choice may be non-obvious, but we shall defer this
discussion for now. In our example, x1 and s1 , . . . , s3 shall be chosen as the
initial basic variables, indicated by gray columns in the tableau above.

Step 3: Gaussian elimination


For a given set of basic variables, we use Gaussian elimination to reduce
the corresponding columns to a permutation of the identity matrix. This
amounts to solving Ax = b in such a way that the values of the nonbasic
variables are zero and the values for the basic variables are explicitly given
by the entries in the right hand column of the fully reduced matrix. In
addition, we eliminate the coefficients of the objective function below each
pivot.
Our initial tableau is thus reduced to
x1 x2 u v s1 s2 s3
1 1 1 1 0 0 0 1
0 3 0 0 1 0 0 3
0 2 1 1 0 1 0 3
0 1 1 1 0 0 1 5
0 1 4 4 0 0 0 1
The solution expressed by the tableau is only admissible if all basic variables
are non-negative, i.e., if the right hand column of the reduced tableau is free
of negative entries. This is the case in this example. At the initial stage,
however, negative entries may come up; this indicates that different initial
basic variables should have been chosen. At later stages in the process, the

3
selection rules for the basic variables will guarantee that an initially feasible
tableau will remain feasible throughout the process.

Step 4: Choose new basic variables


If, at this stage, the objective function row has at least one negative entry,
the cost can be lowered by making the corresponding variable basic. This
new basic variable is called the entering variable. Correspondingly, one
formerly basic variable has then to become nonbasic, this variable is called
the leaving variable. We use the following standard selection rules.

(i) The entering variable shall correspond to the column which has the
most negative entry in the cost function row. If all cost function
coefficients are non-negative, the cost cannot be lowered and we have
reached an optimum. The algorithm then terminates.

(ii) Once the entering variable is determined, the leaving variable shall be
chosen as follows. Compute for each row the ratio of its right hand
coefficient to the corresponding coefficient in the entering variable col-
umn. Select the row with the smallest finite positive ratio. The leaving
variable is then determined by the column which currently owns the
pivot in this row. If all coefficients in the entering variable column
are non-positive, the cost can be lowered indefinitely, i.e., the linear
programming problem does not have a finite solution. The algorithm
then also terminates.

If entering and leaving variable can be found, go to Step 3 and iterate.


Note that choosing the most negative coefficient in rule (i) is only a
heuristic for choosing a direction of fast decrease of the objective function.
Rule (ii) ensures that the new set of basic variables remains feasible.
Let us see how this applies to our problem. The previous tableau holds
the most negative cost function coefficient in column 3, thus u shall be the
entering variable (marked in boldface). The smallest positive ratio of right
hand column to entering variable column is in row 3, as 13 < 51 . The pivot
in this row points to s2 as the leaving variable. Thus, after going through
the Gaussian elimination once more, we arrive at

4
x1 x2 u v s1 s2 s3
1 1 0 0 0 1 0 4
0 3 0 0 1 0 0 3
0 2 1 1 0 1 0 3
0 3 0 0 0 1 1 2
0 9 0 0 0 4 0 13
At this point, the new entering variable is x2 corresponding to the only
negative entry in the last row, the leaving variable is s3 . After Gaussian
elimination, we find

x1 x2 u v s1 s2 s3
2 1 14
1 0 0 0 0 3 3 3
0 0 0 0 1 1 1 5
1 2 13
0 0 1 1 0 3 3 3
0 1 0 0 0 31 1
3
2
3
0 0 0 0 0 1 3 19
Since there is no more negative entry in the last row, the cost cannot be low-
ered by choosing a different set of basic variables; the termination condition
applies.

Step 4: Read off the solution


The solution represented by the final tableau has all nonbasic variables set to
zero, while the values for the basic variables can be can be read off the right
hand column. The bottom right corner gives the negative of the objective
function.
In our example, the solution reads x1 = 14 2 13
3 , x2 = 3 , x3 = u = 3 , s1 = 5,
v = s2 = s3 = 0, which corresponds to = 19, which can be independently
checked by plugging the solution back into the objective function.
As a further check, we note that the solution must satisfy (2b), (2d),
and (2e) with equality and (2c) with a slack of 5. This can also be checked
by direct computation.

2 Initialization
For some problem it is not obvious which set of variables form a feasible
initial set of basic variables. For large problems, a trial-and-error approach

5
n

is prohibitively expensive due the rapid growth of m , the number of possi-
bilities to choose m basic variables out of a total of n variables, as m and n
become large. This problem can be overcome by adding a set of m artificial
variables which form a trivial set of basic variables and which are penalized
by a large coefficients in the objective function. This penalty will cause
the artificial variables to become nonbasic as the algorithm proceeds.
We explain the method by example. For the problem

minimize z = x1 + 2 x2 + 2 x3 (4a)

subject to

x1 + x2 + 2 x3 + x4 = 5 , (4b)
x1 + x2 + x3 x4 = 5 , (4c)
x1 + 2 x2 + 2 x3 x4 = 6 , (4d)
x 0, (4e)

we set up a simplex tableau with three artificial variables which are initially
basic:
a1 a2 a3 x1 x2 x3 x4
1 0 0 1 1 2 1 5
0 1 0 1 1 1 1 5
0 0 1 1 2 2 1 6
1 2 2 0 0
We proceed as before by first eliminating the nonzero entries below the
pivots:

a1 a2 a3 x1 x2 x3 x4
1 0 0 1 1 2 1 5
0 1 0 1 1 1 1 5
0 0 1 1 2 2 1 6
0 0 0 1 3 2 4 2 5 16
Since, for large, 2 5 is the most negative coefficient in the objective
function row, x3 will be entering and, since 52 < 62 < 51 , a1 will be leaving.
The Gaussian elimination step then yields

6
a1 a2 a3 x1 x2 x3 x4
1 1 1 1 5
2 0 0 2 2
1 2 2
1
12 1 0 1
2 2
0 32 5
2
1 0 1 0 1 0 2 1
1 + 52 0 0 12 1 32 0 1 + 72 5 72
Now x2 is entering, a3 is leaving, and we obtain

a1 a2 a3 x1 x2 x3 x4
1
1 0 12 2
0 1 3
2 2
1
0 1 12 2
0 0 12 2
1 0 1 0 1 0 2 1
0 1 + 23 12 0 0 1 + 12 6 2
The new entering variable is x1 while the criterion for the leaving variable
is tied between a2 and x3 . Since we want the artificial variable to become
nonbasic, we take a2 to be leaving. (Choosing x3 as the leaving variable
would lead us to the same solution, albeit after a few extra steps.) We
obtain
a1 a2 a3 x1 x2 x3 x4
1 1 0 0 0 1 2 0
0 2 1 1 0 0 1 4
1 0 1 0 1 0 2 1
1 + 0 0 0 1 6
The termination condition is now satisfied, and we see that the solution is
z = 6 with x1 = 4, x2 = 1, x3 = 0, x4 = 0.
We close with two remarks.

When using a computer to perform the simplex algorithm numerically,


should be chosen large (one or two orders of magnitude larger than
any of the other coefficients in the problem) but not too large (to avoid
loss of significant digits in floating point arithmetic).

If not all artificial variables become nonbasic, must be increased. If


this happens for any value of , the feasible region is empty.

In the final tableau, the penalty parameter can only appear in arti-
ficial variable columns.

7
3 Duality
The concept of duality is best motivated by an example. Consider the
following transportation problem. Some good is available at location A at
no cost and may be transported to locations B, C, and D according to the
following directed graph:

4 B
1
4
!
A 3
= D
5
2 
*
C

On each of the unidirectional channels, the unit cost of transportation is cj


for j = 1, . . . , 5. At each of the vertices b units of the good are sold, where
= B, C, D. How can the transport be done most efficiently?
A first, and arguably most obvious way of quantifying efficiency would be
to state the question as a minimization problem for the total cost of trans-
portation. If xj denotes the amount of good transported through channel j,
we arrive at the following linear programming problem:

minimize c1 x1 + + c5 x5 (5a)

subject to

x1 x3 x4 = bB , (5b)
x2 + x3 x5 = bC , (5c)
x4 + x5 = bD . (5d)

The three equality constraints state that nothing gets lost at nodes B, C,
and D except what is sold.
There is, however, a second, seemingly equivalent way of characterizing
efficiency of transportation. Instead of looking at minimizing the cost of
transportation, we seek to maximize the income from selling the good. Let-
ting y denote the unit price of the good at node = A, . . . , D with yA = 0
by assumption, the associated linear programming problem is the following:

maximize yB bB + yC bC + yD bD (6a)

8
subject to

yB yA c1 , (6b)
yC yA c2 , (6c)
yC yB c3 , (6d)
yD yB c4 , (6e)
yD yC c5 . (6f)

The inequality constraints encode that, in a free MARKET, we can only


maintain a price difference that does not exceed the cost of transportation.
If we charged a higher price, then some local guy would immediately be
able to undercut our price by buying from us at one end of the channel,
using the channel at the same fixed channel cost, then selling at a price
lower than ours at the high-price end of the channel.
Setting

x1 yB 1 0 1 1 0
x = ... , y = yC , and A = 0 1 1 0 1 , (7)

x5 yD 0 0 0 1 1

we can write (5) as the abstract primal problem

minimize cT x (8a)
subject to Ax = b, x 0 . (8b)

Likewise, (6) can be written as the dual problem

maximize y T b (9a)
subject to y T A cT . (9b)

We shall prove in the following that the minimal cost and the maximal
income coincide, i.e., that the two problems are equivalent.
Let us first remark this problem is easily solved without the simplex al-
gorithm: clearly, we should transport all goods sold at a particular location
through the cheapest channel to that location. Thus, we might perform a
simple search for the cheapest channel, something which can be done effi-
ciently by combinatorial algorithms such as Dijkstras algorithm [2]. The ad-
vantage of the linear programming perspective is that additional constraints
such as channel capacity limits can be easily added. For the purpose of

9
understanding the relationship between the primal and the dual problem,
and for understanding the significance of the dual formulation, the simple
present setting is entirely adequate.
The unknowns x in the primal formulation of the problem not only iden-
tify the vertex of the feasible region at which the optimum is reached, but
they also act as sensitivity parameters with regard to small changes in the
cost coefficients c. Indeed, when the linear programming problem is nonde-
generate, i.e. has a unique optimal solution, changing the cost coefficients
from c to c + c with |c| sufficiently small will not make the optimal
vertex jump to another corner of the feasible region, as the cost depends
continuously on c. Thus, the corresponding change in cost is cT x. If xi
is nonbasic, the cost will not react at all to small changes in ci , whereas if
xi is large, then the cost will be sensitive to changes in ci . This informa-
tion is often important because it gives an indication where to best spend
resources if the parameters of the problemin the example above, the cost
of transportationare to be improved.
Likewise, the solution vector y to the dual problem provides the sensi-
tivity of the total income to small changes in b. Here, b is representing the
number of sales at the various vertices of the network; if the channels were
capacity constrained, the channel limits were also represented as components
of b. Thus, the dual problem is providing the answer to the question if I
were to invest in raising sales, where should I direct this INVESTMENT to
achieve the maximum increase in income?
The following theorems provide a mathematically precise statement on
the equivalence of primal and dual problem.
Theorem 1 (Weak duality). Assume that x is a feasible vector for the
primal problem (8) and y is a feasible vector for the dual problem (9). Then
(i) y T b cT x;

(ii) if (i) holds with equality, then x and y are optimal for their respective
linear programming problems;

(iii) the primal problem does not have a finite minimum if and only if the
feasible region of the dual problem is empty; vice versa, the dual prob-
lem does not have a finite maximum if and only if the feasible region
of the primal problem is empty.
The proof is simple and shall be left as an exercise.
To proceed, we say that x is a basic feasible solution of Ax = b, x 0
if it has at most m nonzero components. We say that it is nondegenerate if

10
it has exactly m nonzero components. If, the the course of performing the
simplex algorithm, we hit a degenerate basic feasible solution, it is possible
that the objective function row in the simplex tableau contains negative
coefficients, yet the cost cannot be lowered because the corresponding basic
variable is already zero. This can lead to cycling and thus non-termination
of the algorithm. We shall not consider the degenerate case further.
When x is a nondegenerate solution to the primal problem (8), i.e., x is
nondegenerate basic feasible and also optimal, then we can be assured that
the simplex method terminates with all coefficients in the objective function
row nonnegative. (If they were not, we could immediately perform at least
one more step of the algorithm with strict decrease in the cost.) In this
situation, we have the following stronger form of the duality theorem.

Theorem 2 (Strong duality). The primal problem (8) has a nondegenerate


solution x if and only if the dual problem (9) has a nondegenerate solution
y; in this case y T b = cT x.

Proof. The proof is based on a careful examination of the termination con-


dition of the simplex algorithm. Assume that x solves the primal problem.
Without loss of generality, we can reorder the variables such that the first
m variables are basic, i.e. !
xB
x= (10)
0
and that the final simplex tableau reads
!
I R xB

. (11)
0T r T z

The last row represents the objective function coefficients and z denotes
the optimal value of the objective function. We note that the termination
condition of the simplex algorithm reads r 0. We now partition the initial
matrix A and the coefficients of the objective function c into their basic and
nonbasic components, writing
!
  cB
A= B N and c= . (12)
cN

Finally, it can be shown that the elementary row operations used in the
Gaussian elimination steps of the simplex algorithm can be written as mul-
tiplication by a matrix from the left, which we also partition into components

11
compatible with the block matrix structure of (11), so that the transforma-
tion from the initial to the final tableau can be written as
! ! !
M v B N b M B + vcTB M N + vcTN Mb

= . (13)
uT cTB cTN 0 uT B + cTB uT N + cTN
T
u b

We now compare the right hand side of (13) with (11) to determine the
coefficients of the left hand matrix. First, we note that in the simplex
algorithm, none of the Gaussian elimination steps on the equality constraints
depend on the objective function coefficients (other than the path taken from
initial to final tableau, which is not at issue here). This immediately implies
that v = 0. Second, we observe that nowhere in the simplex algorithm do
we ever rescale the objective function row. This immediately implies that
= 1. This leaves us with the following set of matrix equalities:

MB = I , (14a)
M b = xB , (14b)
uT B + cTB = 0 , (14c)
T
u N+ cTN = r. (14d)

so that M = B 1 and uT = cTB B 1 . We now claim that

y T = cTB B 1 (15)

solves the dual problem. We compute


 
y T A = cTB B 1 B N
 
= cTB cTB B 1 N
 
= cTB cTN r T
 
cTB cTN = cT . (16)

This shows that y is feasible for the dual problem. Moreover,

y T b = cTB B 1 b = cTB xB = cT x . (17)

Thus, by weak duality, y is also optimal for the dual problem.


The reverse implication of the theorem follows from the above by noting
that the bi-dual is identical with the primal problem.

12
References
[1] G.B. Dantzig, M.N. Thapa, Linear Programming, Springer, New York,
1997.

[2] Dijkstras algorithm. Wikipedia, The Free Encyclopedia. April 4, 2012,


06:13 UTC.
http://en.wikipedia.org/w/index.php?title=Dijkstra%27s_algorithm&oldid=485462965

[3] D. Gale, Linear Programming and the simplex method, Notices of the
AMS 54 (2007), 364369.
http://www.ams.org/notices/200703/fea-gale.pdf

[4] P. Pedregal, Introduction to optimization, Springer, New York, 2004.

13
The Basic Problem

Given a set of tabular values ( available from an experiment) of a

function f .

X x0 x1 x2 ... xn

f (X) f (x0) f (x1) f (x3) ... f (xn)

Where the explicit nature of the function f is not known.

To find a function such that (xi) = f (xi) for all 0 i n.

Such a function is called an interpolating function.

Dr. Raj Kumar, VBS PU Jaunpur. 1


Geometrically

The problem is :

To find a function / polynomial whose graph passes through the given

set of (n + 1) points,

(x0, f (x0)), (x1, f (x1)), , (xn, f (xn)).

Dr. Raj Kumar, VBS PU Jaunpur. 2


Polynomial interpolation

If is a polynomial then the process is called polynomial interpolation

and is called an interpolating polynomial.

Dr. Raj Kumar, VBS PU Jaunpur. 3


Some types of interpolation

Polynomial interpolation

Piecewise Polynomial( Spline ) interpolation

Rational interpolation

Trigonometric interpolation

Exponential interpolation

We study : Polynomial and Piecewise polynomial interpolations.

Dr. Raj Kumar, VBS PU Jaunpur. 4


Example

Find a polynomial which fits the following data.

X x0 x1 x2

f (X) f (x0) f (x1) f (x2)

Dr. Raj Kumar, VBS PU Jaunpur. 5


Example

Solution:

(x x1)(x x2) (x x0)(x x2)


P3(x) = f (x0) + f (x1)
(x0 x1)(x0 x2) (x1 x0)(x1 x2)
(x x0)(x x1)
+ f (x2).
(x2 x0)(x2 x1)

This is a polynomial of degree 2.

Called Lagranges interpolation formula.

Dr. Raj Kumar, VBS PU Jaunpur. 6


Lagranges Interpolation formula

Similarly,

(x x1)(x x2) (x xn)


pn(x) = f (x0)
(x0 x1)(x0 x2) (x0 xn)

(x x2)(x x3) (x xn)


+ f (x1)
(x1 x0)(x1 x2) (x1 xn)

(x x0)(x x1) (x xn1)


+ + f (xn)
(xn x0)(xn x1) (x1 xn1)
is the unique polynomial of degree n which interpolates f at the

n + 1 points x0, x1, , xn.

Dr. Raj Kumar, VBS PU Jaunpur. 7


Lagranges Interpolation formula

If we write

(x x0)(x x1) (x xi1)(x xi+1) (x xn)


li(x) =
(xi x0)(xi x1) (xi xi1)(xi xi+1) (xi xn)

then
Pn
pn(x) = i=0 li (x)f (xi )

is the unique(how?) polynomial of degree n which interpolates f at

the n + 1 points x0, x1, , xn.

Dr. Raj Kumar, VBS PU Jaunpur. 8


We proved:

Theorem 1. (Existence and Uniqueness theorem for interpolating

polynomials.) If x0, x1, x2, , xn are n + 1 distinct numbers and f

is a function whose values are given at these numbers, then a unique

polynomial p(x) of degree n with p(xi) = f (xi) for all i = 1, 2, , n.

This polynomial is given by

Pn
p(x) = i=0 li (x)f (xi )

Dr. Raj Kumar, VBS PU Jaunpur. 9


NOTATION

Suppose f : [a, b] R is given at the n + 1 points x0, x1, , xn.

We use pn(x) to denote the unique polynomial ( Lagranges ) of degree

n which interpolates f at x0, x1, , xn.

Dr. Raj Kumar, VBS PU Jaunpur. 10


Thus we have:

Given x0, x1, x2, x3, , xn and f (x0), f (x1), , f (xn), we have:

Lagranges polynomial Degree at most interpolates f at

p1(x) 1 x0, x1

p2(x) 2 x0, x1, x2

p3(x) 3 x0, x1, x2, x3


.. .. ..

pn(x) n x0, x1, x2, x3, , xn

Dr. Raj Kumar, VBS PU Jaunpur. 11


Example

Find the interpolating polynomial in Lagrangian form for the data.

x -2 -1 1 3

f (x) -15 -4 0 20

Solution:

We calculate p3(x).

p3(x) = l0(x).f (x0) + l1(x).f (x1) + l2(x).f (x2) + l3(x).f (x3)

Dr. Raj Kumar, VBS PU Jaunpur. 12


(xx1 )(xx2 )(xx3 ) (x+1)(x1)(x3)
Where l0(x) = (x0 x1 )(x0 x2 )(x0 x3 ) = (1)(3)(5)

1 3
= (x 3x2 x + 3)
15

Similarly,
1 3
l1(x) = (x 2x2 5x + 6)
8
1 3
l2(x) = (x 7x 6)
12
1
l3(x) = (x3 + 2x2 x + 2)
40

Therefore,

1 3 1
p3(x) = 15{ (x 3x2 x + 3)} 4{ (x3 2x2 5x + 6)}
15 8
Dr. Raj Kumar, VBS PU Jaunpur. 13
1 3
+0 + 20{ (x + 2x2 x + 2)}
40

= x3 x2 + x 1.

Dr. Raj Kumar, VBS PU Jaunpur. 14


A disadvantage

If more data become available, the work performed to generate the

original Lagrange form cannot be reused to compute a higher-degree

polynomial.

This can be rectified by writing the Lagranges interpolation polynomial

in Newtons form.

For this we use the idea of divided differences.

Dr. Raj Kumar, VBS PU Jaunpur. 15


First order divided differences

f (x1) f (x0)
f [x0, x1] =
x1 x0
f (x2) f (x1)
f [x1, x2] =
x2 x1
.. ..

In general,
f (xi+1) f (xi)
f [xi, xi+1] =
xi+1 xi
.. ..

f (xn) f (xn1)
f [xn1, xn] =
xn xn1

Dr. Raj Kumar, VBS PU Jaunpur. 16


Second order divided differences

f [x1, x2] f [x0, x1]


f [x0, x1, x2] =
x2 x 0

f [x2, x3] f [x1, x2]


f [x1, x2, x3] =
x3 x 1
.. ..
f [xi+1, xi+2] f [xi, xi+1]
f [xi, xi+1, xi+2] =
xi+2 xi
.. ..
f [xn1, xn] f [xn1, xn]
f [xn2, xn1, xn] =
xn xn2

Dr. Raj Kumar, VBS PU Jaunpur. 17


Divided differences of nth( 3) order

Can be defined recursively in a similar way.

f [xi+1, ,xi+n ] f [xi, xi+1, ,xi+n1 ]


f [xi, xi+1, , xi+n] =
xi+n xi

The numbers f (x0), f (x1), , f (xn) are called divided differences of

0-th order and are denoted by f [x0], f [x1], , f [xn] respectively.

We say: f [x0, x1, , xn] as the divided difference of f at the points

x0, x1, , xn.

Dr. Raj Kumar, VBS PU Jaunpur. 18


Divided differences in Tabular form (when n= 3.)

x f (x) First order Second order Third order

x0 f [x0]

f [x0, x1]

x1 f [x1] f [x0, x1, x2]

f [x1, x2] f [x0, x1, x2, x3]

x2 f [x2] f [x1, x2, x3]

f [x2, x3]

x3 f [x3]

Dr. Raj Kumar, VBS PU Jaunpur. 19


Example

Consider the data:

x -2 -1 1 3

f (x) -15 -4 0 20

Dr. Raj Kumar, VBS PU Jaunpur. 20


x f (x) f [, ] f [, , ] f [, , , ]

-2 -15

11

-1 -4 -3
The divided difference table:
2 1

1 0 2

10

3 20

Dr. Raj Kumar, VBS PU Jaunpur. 21


The Newtons form of a polynomial

A polynomial in Newtons form having

coefficients a0, a1, a2, , an

and

centers c1, c2, , cn

is the polynomial:

p(x) = a0 +a1(xc1)+a2(xc1)(xc2)+ +an(xc1)(xc2) (xcn)

Dr. Raj Kumar, VBS PU Jaunpur. 22


Remark

Every polynomial in the power form can be put in Newtons form.

Dr. Raj Kumar, VBS PU Jaunpur. 23


Lagranges interpolation polynomial in Newtons form

Suppose f (x) is known at the n + 1 points, x0, x1, , xn.

Suppose the Lagranges interpolation polynomial pn(x) of degree n

can be written in the Newtons form as :

pn(x) = A0 +A1(xx0)+A2(xx0)(xx1)+A3(xx0)(xx1)(xx2)+

+ An(x x0)(x x1)(x x2) (x xn1)

We prove:

Ak = f [x0, x1, , xk ]

for all k = 0, 1, 2, , n.

Dr. Raj Kumar, VBS PU Jaunpur. 24


Thus the Newtons form of pn(x) is:

pn(x) = f [x0]

+f [x0, x1](x x0)

+f [x0, x1, x2](x x0)(x x1)

+f [x0, x1, x2, x3](x x0)(x x1)(x x2)

+ f [x0, x1, , xn](x x0)(x x1)(x x2) (x xn1)

Look at the divided difference table and recognize the coefficients.

Thus the divided difference table helps us to write the interpolation

Dr. Raj Kumar, VBS PU Jaunpur. 25


polynomial easily.

x f (x) First order Second order Third order

x0 f [x0]

f [x0, x1]

x1 f [x1] f [x0, x1, x2]

f [x1, x2] f [x0, x1, x2, x3]

x2 f [x2] f [x1, x2, x3]

f [x2, x3]

x3 f [x3]

Dr. Raj Kumar, VBS PU Jaunpur. 26


Example

Consider the data:

x -2 -1 1 3

f (x) -15 -4 0 20

Dr. Raj Kumar, VBS PU Jaunpur. 27


x f (x) f [, ] f [, , ] f [, , , ]

-2 -15

11

-1 -4 -3
The divided difference table:
2 1

1 0 2

10

3 20

p3(x) = 15 + 11(x + 2) 3(x + 2)(x + 1) + 1(x + 2)(x + 1)(x 1)

= x3 x2 + x 1.

Dr. Raj Kumar, VBS PU Jaunpur. 28


When the points x0, x1, , xn are equispaced.

Let xi+1 xi = h for all i = 0, 1, 2, , n 1.

The Newtons divided difference formula can be further simplified to

Newtons forward difference form.

We first define the forward differences.

Dr. Raj Kumar, VBS PU Jaunpur. 29


Forward difference table

Consider the data:

x -2 -1 0 1

f (x) -15 -4 0 20

Dr. Raj Kumar, VBS PU Jaunpur. 30


x f (x) 41 f 42f 43 f

-2 -15

11

-1 -4 -7
The forward difference table:
4 23

0 0 16

20

1 20

Dr. Raj Kumar, VBS PU Jaunpur. 31


Forward differences

First order forward differences:

41f (x0) = f (x1) f (x0)

41f (x1) = f (x2) f (x1)

.. ..

41f (xi) = f (xi+1) f (xi)

.. ..

41f (xn1) = f (xn) f (xn1)

Dr. Raj Kumar, VBS PU Jaunpur. 32


We call 41f (xi) as: The first order forward difference of f at xi.

meaning full for all i = 0, 2, , n 1

Dr. Raj Kumar, VBS PU Jaunpur. 33


forward differences of higher order:

can be computed using the recursive following formula.

If n > 1 then we define,

4nf (xi) = 4n1f (xi+1) 4n1f (xi)

Dr. Raj Kumar, VBS PU Jaunpur. 34


Divided differences in terms of forward differences

Suppose f : [a, b] R is given at the n + 1 equispaced points

x0, x1, , xn with spacing h = xi+1 xi.

f (x1) f (x0) 1
f [x0, x1] = = 4f (x0)
x1 x 0 h

f [x1, x2] f [x0, x1]


f [x0, x1, x2] =
x2 x 0

1 4f (x1) 4f (x0) 1
= [ ] = 2 42f (x0)
2h h 2h
Dr. Raj Kumar, VBS PU Jaunpur. 35
In general,

1 k
f [x0, x1, , xk ] = 4 f (x0).
k! hk

Dr. Raj Kumar, VBS PU Jaunpur. 36


Newtons forward difference formula.

Suppose f : [a, b] R is given at the n + 1 equispaced points

x0, x1, , xn with spacing h = xi+1 xi.

xx0
Given x, let s = h .

Thus x can be written in the form x = x0 + sh for some s

In particular, xi = x0 + ih for all i = 0, 1, 2, , n.

Thus

x xi = x0 + sh (x0 + ih) = (s i)h

Dr. Raj Kumar, VBS PU Jaunpur. 37


Now the interpolating polynomial pn(x) can be written as:

pn(x) = pn(x0 + sh) = f [x0]

+sh f [x0, x1]

+s(s 1)h2 f [x0, x1, x2]

+s(s 1)(s 2)h3f [x0, x1, x2, x3]

+ s(s 1)(s 2) (s n + 1)f [x0, x1, , xn]

n
X
= s(s 1)(s 2) (s k + 1)f [x0, x1, , xk ]
k=0

Dr. Raj Kumar, VBS PU Jaunpur. 38


n  
X s
= k! hk f [x0, x1, , xk ]
k
k=0
n  
X s
= 4k f (x0)
k
k=0

Where
 
s s(s 1) (s k + 1)
= .
k k!

Dr. Raj Kumar, VBS PU Jaunpur. 39


The error in Newtons forward difference formula.

En(x) = (x x0)(x x1) (x xn) f [x0, x1, , xn, x]

= s(s 1)(s 2) (s n) hn+1 f [x0, x1, , xn, x]

(n+1) f (n+1)()
= s(s 1)(s 2) (s n) h
(n + 1)!

Dr. Raj Kumar, VBS PU Jaunpur. 40


for some (x0, xn)

 
s
= hn+1 f (n+1)()
n+1

Dr. Raj Kumar, VBS PU Jaunpur. 41


Example

Prepare the forward difference table for the following data. Using

Newtons forward interpolating polynomial , find approximate value of

f (0.1).

x 0 0.2 0.4 0.6 0.8

f (x) 0.12 0.46 0.74 0.90 1.2

Dr. Raj Kumar, VBS PU Jaunpur. 42


x f (x) 41 f 42f 43 f 44 f

0 0.12

0.34

0.2 0.46 -0.06

0.28 -0.06

0.4 0.74 -0.12 0.32

0.16 0.26

0.6 0.90 0.14

0.30

0.8 1.2

Dr. Raj Kumar, VBS PU Jaunpur. 43


To interpolate at x = 0.1

x x0 0.1 0 1
s= = = .
h 0.2 2

Therefore the binomial coefficients are:

  1 1
( 1)
   
s s 1 s 1
= 1, = , =2 2 = .
0 1 2 2 1.2 8

  1 1 3
s s(s 1)(s 3) 2 ( 2 )( 2) 1
= = = .
3 1.2.3 1.2.3 16

Dr. Raj Kumar, VBS PU Jaunpur. 44


  1 1 3 5
s s(s 1)(s 2)(s 3) 2 ( 2 )( 2 )( 2) 5
= = = .
4 1.2.3.4 1.2.3.4 128

Therefore

     
s s s
p4(x0+sh) = f (x0)+s41f (x0)+ 42f (x0)+ 43f (x0)+ 44f (x0).
2 3 4

1 1 1 5
= 0.12 + (0.34) + (0.06) + (0.06) + (0.32) = 0.28125
2 8 16 128

Hence f (0.1) 0.28125

Dr. Raj Kumar, VBS PU Jaunpur. 45


We Use the interpolating polynomial

To estimate the missing data.

OR

To approximate f at some intermediate points.

Dr. Raj Kumar, VBS PU Jaunpur. 46


Numerical Integration

If f : [a, b] R is differentiable then, we obtain a new function f 0 :

[a, b] R, called the derivative of f . Likewise, if f : [a, b] R is

integrable, then we obtain a new function F : [a, b] R defined by

Z x
F (x) = f (t)dt x [a, b].
a

Z b
Observation: If f is nonnegative function, then f (x)dx is represent
a
the area under the curve f (x).

Dr. Raj Kumar, VBS PU Jaunpur. 47


Antiderivative

Antiderivative: Let F : [a, b] R be such that f = F 0, then F is called

an antiderivative of f .

Recall

Fundamental Theorem of Calculus: Let f : [a, b] R is integrable

and has an antiderivative F , then


Z b
f (x)dx = F (b) F (a).
a

Dr. Raj Kumar, VBS PU Jaunpur. 48


Basic Problems

Difficult to find an antiderivative of the function (for example f (x) =


2
ex )

Function is given in the tabular form.

Dr. Raj Kumar, VBS PU Jaunpur. 49


Newton-Cotes Methods/Formulae

The derivation of Newton-Cotes formula is based on Polynomial

Interpolation.

x x0 x1 x2 ... xn

f (x) f (x0) f (x1) f (x3) ... f (xn)

Dr. Raj Kumar, VBS PU Jaunpur. 50


The idea is:

Rb
Replace f by pn(x) and evaluate a
pn(x)dx

That is,
Z b Z b n
Z bX
f (x)dx ' pn(x)dx = li(x)f (xi)dx
a a a i=0
n
X Z b
= f (xi) li(x)dx
i=0 a

Xn
= Aif (xi)
i=0
Rb
Where Ai = l (x)dx
a i
called weights.

Dr. Raj Kumar, VBS PU Jaunpur. 51


Types of Newton-Cotes Formulae

Trapezoidal Rule (Two pint formula)

Simpsons 1/3 Rule (Three Point formula)

Simpsons 3/8 Rule (Four point formula)

Dr. Raj Kumar, VBS PU Jaunpur. 52


Trapezoidal Rule

Since it is two point formula, it uses the first order interpolation

polynomial P1(x).

Z b Z x1
f (x) P1(x)dx
a x0

P1(x) = f (x0) + sf (x0)


x x0
s=
h
Now, dx = h ds at x = x0, s = 0 and at x = x1, s = 1.

Dr. Raj Kumar, VBS PU Jaunpur. 53


Hence,

Z b Z 1
h
f (x)dx (f (x0) + sf (x0))hds = [f (x0) + f (x1)]
a 0 2

OR
b
ba
Z
f (x)dx [f (a) + f (b)]
a 2
Error
T (b a)3 00
E = f (),
12
where a < < b

Remark: x0 = a and x1 = b.

Dr. Raj Kumar, VBS PU Jaunpur. 54


1
Basic Simpsons 3 Rule

Rb
To evaluate a
f (x)dx.

f will be replaced by a polynomial of degree 2 which interpolates f at


a+b
a, 2 and b.
a+b
Take x0 = a, x1 = 2 , x2 = b

Now

s(s 1) 2
p2(x) = p2(x0 + sh) = f (x0) + s41f (x0) + 4 f (x0)
2

xx0 xa ba
Where s = h = h and h = 2 .

Dr. Raj Kumar, VBS PU Jaunpur. 55


Hence dx = hds. Also x = a = s = 0 and x = b = s = 2. Thus,

Z b Z b
s
f (x)dx ' I (f ) = p2(x)dx
a a

2
s(s 1) 2
Z
1
= [f (x0) + s4 f (x0) + 4 f (x0)]h.ds
0 2
h
= [f (x0) + 4f (x1) + f (x2)]
3
ba b+a
= [f (a) + 4f ( ) + f (b).]
6 2
Error
s h5f (4)()
E =
90
for some (a, b).

Dr. Raj Kumar, VBS PU Jaunpur. 56


3
Simpsons 8 rule

f is replaced by p3(x) which interpolates f at x0 = a, x1 = a +


ba
h, x2 = a + 2h, x3 = a + 3h = b. where h = 3 .

Thus

b b
s(s 1) 2 s(s 1)(s 2) 3
Z Z
1
f (x)dx ' [f (x0)+s4 f (x0)+ 4 f (x0)+ 4 f (x0)]dx
a a 2 3!

xx0
Where s = h and dx = hds. Also x = a = s = 0 and x = b =

s = 3.

Dr. Raj Kumar, VBS PU Jaunpur. 57


Therefore,

b 3
s2 s 2 s3 3s2 + 2s 3
Z Z
1
f (x)dx ' [f (x0)+s4 f (x0)+ 4 f (x0)+ 4 f (x0)]dx
a 0 2 3!

9 1 9 1 81
= h[3f0+ (f1f0)+ (9 )(f22f1+f0)+ ( 27+9)(f33f2+3f1f0)]
2 2 2 6 4
3h
= [f0 + 3f1 + 3f2 + f3]
8

Thus we get:

Z b
3h
f (x)dx ' [f0 + 3f1 + 3f2 + f3]
a 8

3h5 (4)
s
Error: E = f (), where a < < b.
80

Dr. Raj Kumar, VBS PU Jaunpur. 58


Example

1
R2 4
R2
Using Trapezoidal and Simpson 3 rules find 0
x dx and 0
sinxdx and

find the upper bound for the error.

Dr. Raj Kumar, VBS PU Jaunpur. 59


Composite Rules

Note that if the integral [a, b] is large, then the error in the Trapezoidal

rule will be large.

Idea

Error can be reduced by dividing the interval [a, b] into equal subinterval

and apply quadrature rules in each subinterval.

Dr. Raj Kumar, VBS PU Jaunpur. 60


Composite Trapezoidal Rule

ba
h= , xi = x0 + ih
n

Composite Rule

Z b Z xn n Z
X xi
f (x)dx = f (x)dx = f (x)dx
a x0 i=1 xi1
Now apply Trapezoidal rule on each [xi1, xi], we have

Z b
h
f (x)dx = [f (x0) + 2 (f (x1) + f (x2) f (xn1)) + f (xn)]
a 2

Dr. Raj Kumar, VBS PU Jaunpur. 61


Error in composite Trapezoidal rule

We know error in the ith interval [xi1, xi] is

3 00 3 00
(x i x i1 ) h
E CT (I) = f (i) = f (i)
12 12

where i (xi1, xi)). Hence the error in composite rule is

n
h3 X 00
E CT = f (i)
12 i=1

Using Intermediate value theorem, we can prove that

CT h2 00
E = (b a) f (), [a, b]
12
Dr. Raj Kumar, VBS PU Jaunpur. 62
Dr. Raj Kumar, VBS PU Jaunpur. 63
1
The Composite Simpsons 3 Rule

[a, b] will be will be divided into 2n equal subintervals and we apply


1
basic Simpsons 3 rule on each of the n intervals [x2i2, x2i] for i =

1, 2, 3, , n.

ba
Thus here h = 2n .

Then
Z b Z b=x2n
f (x)dx = f (x)dx
a a=x0
Z x2 Z x4 Z x2i Z x2n
= f (x)dx + f (x)dx + + f (x)dx + + f (x)dx
x0 x2 x2i2 x2n2

Dr. Raj Kumar, VBS PU Jaunpur. 64


h h
= [f (x0) + 4f (x1) + f (x2)] + [f (x2) + 4f (x3) + f (x4)]+
3 3
h
+ + [f (x2n2) + 4f (x2n1) + f (x2n)]
3
h
= {f (x0) + 4 [f (x1) + f (x3) + f (x5) + + f (x2n1)]+
3
+2 [f (x2) + f (x4) + f (x6) + + f (x2n2)] + f (x2n)}

Dr. Raj Kumar, VBS PU Jaunpur. 65


1
The Error in the Composite Simpsons 3 Rule

We know error in the ith interval (x2i2, x2i) is

cs h5 (4)
E (I) = f (i), i (x2i2, x2i)
90
Using Intermediate value theorem, the error in composite Simpsons

rule is given by
CS h4 (4)
E = (b a) f (),
180
where [a, b]

Dr. Raj Kumar, VBS PU Jaunpur. 66


Example

R1 1
Evaluate the integral 1
x2 exp(x)dx by composite Simpsons 3 rule

with spacing h = 0.25

1
Solution: According to composite Simpsons 3 rule:

Z 1
h
x2 exp(x)dx = [f (x0) + 4f (x1) + 2f (x2) + 4f (x3) + 2f (x4)+
1 3

+4f (x5) + 2f (x6) + 4f (x7) + f (x8)]

Here f (x0) = f (1) = 2.7183

f (x1) = f (0.75) = 1.1908

Dr. Raj Kumar, VBS PU Jaunpur. 67


f (x2) = f (0.5) = 0.4122

f (x3) = f (0.25) = 0.0803

f (x4) = f (0) = 0

f (x5) = f (0.25) = 0.0487

f (x6) = f (0.50) = 0.1516

f (x7) = f (0.75) = 0.2657

f (x8) = f (1) = 0.3679

Dr. Raj Kumar, VBS PU Jaunpur. 68


Substituting these values in the above formula we get:

Z 1
x2 exp(x)dx ' 0.87965
1

Dr. Raj Kumar, VBS PU Jaunpur. 69


Example

Find the minimum no. of subintervals, used in composite Trapezoidal


Z 1
x4
and Simpsons 1/3 rule in order to find the integral e dx such that
0
the error can not exceed by .00001.

Sol. For the composite Trapezoidal rule, we have

00
13 max0<<1 |f ()|
.00001
12n2trap
For the composite Simpson 1/3 rule, we have

Dr. Raj Kumar, VBS PU Jaunpur. 70


14 max0<<1 |f (4)()|
.00001
180n4simp
Now,

00
max |f ()| 3.5, max |f (4)()| 95
0<<1 0<<1

(Please verify )

Hence

ntrap = 171, nsimp = 16

Dr. Raj Kumar, VBS PU Jaunpur. 71


3
Composite Simpsons 8 rule

ba 3
[a, b] is divided into 3n equal subintervals. (h = 3n . and we apply 8

rule on each of the n intervals [x3i3, x3i] for i = 1, 2, 3, , n.)

Hence,

Z b Z x3 Z x6 Z x3n =b
f (x)dx ' f (x)dx + f (x)dx + + f (x)dx
a x0 =a x3 x3n3

3h 3h
= [f0 + 3f1 + 3f2 + f3] + [f3 + 3f4 + 3f5 + f6]+
8 8
3h
+ + [f3n3 + 3f3n2 + 3f3n1 + f3n]
8
Dr. Raj Kumar, VBS PU Jaunpur. 72
3h
= [f0 + 3f1 + 3f2 + 2f3 + 3f4 + 3f5 + 2f6 + 3f7 + + 3f3n1 + f3n]
8
Remember:

f with suffices of multiple 3 are multiplied by 2.

Others by 3, except the end points.

Dr. Raj Kumar, VBS PU Jaunpur. 73


Example

3
Use composite simpsons 8 rule, find the velocity after 18 seconds, if a

rocket has acceleration as given in the table:


t= 0 2 4 6 8 10 12 14 16 18

a= 40 60 70 75 80 83 85 87 88 88

f0 f1 f2 f3 f4 f5 f6 f7 f8 f9

3h
Sol: Velocity v = 8 [f0 +3f1 +3f2 +2f3 +3f4 +3f5 +2f6 +3f7 +3f8 +f9 ]=
3
4 [40+360+370+275+380+383+283+285+387+388+88]

= 1389 units.

Dr. Raj Kumar, VBS PU Jaunpur. 74


Method of Undetermined Parameters

The Newton - Cotes integration rules are all of the form

I(f ) ' A0f (x0) + A1f (x1) + A2f (x2) + + Anf (xn)

Also, note that the weights Ais do not depend on the given function.

Hence, if the error is of the form

E(I) = Const f (r+1)().

Then the rule must be exact for all polynomials of degree r

Dr. Raj Kumar, VBS PU Jaunpur. 75


Therefore

If we wish to construct a rule of the form

I(f ) ' A0f (x0) + A1f (x1) + a2f (x2) + + Anf (xn)

(n-fixed) which is exact for polynomials of degree as high as possible,

i.e., we want

E(I) = Const f (r+1)(),

with r as large as possible.

This way of constructing integration rules is called the Method of

Undetermined Parameters.

Dr. Raj Kumar, VBS PU Jaunpur. 76


Example

Suppose we want to derive an integration formula of the form:


Rb
a
f (x)dx = A0 f (a) + A1 f (b) + f 00().

We assume that:The rule is exact for the polynomials 1, x, x2.

Now, taking f (x) = 1, we get b a = A0 + A1

b2 a2
Taking f (x) = x we get 2 = A0 a + A1 b

ba
Solving the above two equations we get, A0 = A1 = 2 .
Z b
ba
Thus, f (x)dx = [f (a) + f (b)] + f 00()
a 2

Dr. Raj Kumar, VBS PU Jaunpur. 77


Now if we take f (x) = x2, we get:

b3 a3 ba 2
=( )(a + b2) + 2!
3 2

(ba)3
= = 12

Thus
b
ba (b a)3 00
Z
f (x)dx = [f (a) + f (b)] f ()
a 2 12

We see that: This is exactly the trapezoidal rule. Similarly, Simpsons


1 3
3 and 8 rules can be derived.

Dr. Raj Kumar, VBS PU Jaunpur. 78


Thus in the Method of Undetermined Parameters

We aim directly for a formula of a preselected type.

Working Method:

We impose certain conditions on a formula of desired form and use

these conditions to determine the values of the unknown coefficients in

the formula.

Dr. Raj Kumar, VBS PU Jaunpur. 79


The Error term in the Simpsons 38 -rule, using Method

of Undetermined Parameters

Start with:

Z x3
3h
f (x)dx = [f0 + 3f1 + 3f2 + f3] + f (4)()
x0 8

for some suitable (x0, x3).

Takeing f (x) = x4 in the above integration rule we get:

x53 x50 3h 4
= [x0 + 3x41 + 3x42 + x43] + 4!
5 8
Dr. Raj Kumar, VBS PU Jaunpur. 80
x53 x50 3h 4
4! = [x0] + 3(x0 + h)4 + 3(x0 + 2h)4 + (x0 + 3h)4
5 8

(x0 + 3h)5 x50 3h 4


= [x0] + 3(x0 + h)4 + 3(x0 + 2h)4 + (x0 + 3h)4
5 8

Without loss of generality, we can take: x0 = 0.


243 5 3h5
We have: 4! = 5 h 8 [0 + 3 + 3 16 + 81] Thus

9 5
4! = h
10

That is,

3 5
= h
80
Dr. Raj Kumar, VBS PU Jaunpur. 81
Therefore the error in the Simpsons rule is =

3 5 (4)
h f ()
80

for some suitable (a, b).

Dr. Raj Kumar, VBS PU Jaunpur. 82


Example

Find A0, x1, A2 so that the following rule is exact for all polynomials of

degree as high as possible and find the error term also.

Z 2
4
f (x)dx = A0f (0) + f (x1) + A2f (2) + E(I)
0 3

where E(I) = f 000() for some [0, 2].

(Note here that, we have chosen the error to be i terms of the third

derivative of f because there are 3 unknowns. )

Taking f (x) = 1 we get: 2 = A0 + 43 + A2

Dr. Raj Kumar, VBS PU Jaunpur. 83


Taking f (x) = x we get: 2 = 43 x1 + 2A2

8
Taking f (x) = 1 we get: 3 = 43 x21 + 4A2

1
On solving these equations we get: x1 = 1, A0 = 3 = A2

Thus the integration rule becomes,

Z 2
1
f (x)dx = [f (0) + 4f (1) + f (2)] + f 000()
0 3

Now to find the error term:

Take f (x) = x3 we get = 0. Thus the rule is exact up to polynomials

of degree 3.

Dr. Raj Kumar, VBS PU Jaunpur. 84


Therefore: Choose the error term as:

E(I) = f (4)()

with 4-th derivative.

Thus the integration rule becomes:

Z 2
4
f (x)dx = A0f (0) + f (x1) + A2f (2) + f (4)()
0 3

32
Now take f (x) = x4 we get 5 = 13 [0 + 4 + 24] + 4!

1
= = .
90
Dr. Raj Kumar, VBS PU Jaunpur. 85
1 (4)
Hence the error term is: 90 f ()

Thus the required integration rule is

Z 2
1 1
f (x)dx = [f (0) + 4f (1) + f (2)] + f (4)()
0 3 90

Note that this is Simpsons 13 -rule.

Dr. Raj Kumar, VBS PU Jaunpur. 86


The Error term in the Simpsons 83 -rule , using Method

of Undetermined Parameters

Start with:

Z x3
3h
f (x)dx = [f0 + 3f1 + 3f2 + f3] + f (4)()
x0 8

for some suitable [x0, x3].

Takeing f (x) = x4 in the above integration rule we get:

x53 x50 3h 4
= [x0 + 3x41 + 3x42 + x43] + 4!
5 8
Dr. Raj Kumar, VBS PU Jaunpur. 87
x53 x50 3h 4
4! = [x0] + 3(x0 + h)4 + 3(x0 + 2h)4 + (x0 + 3h)4
5 8
(x0 + 3h)5 x50 3h 4
= [x0] + 3(x0 + h)4 + 3(x0 + 2h)4 + (x0 + 3h)4
5 8

243 5
Without loss of generality, we can take: x0 = 0. We have: 4! = 5 h
3h5
8 [0 + 3 + 3 16 + 81]

Thus
9 5
4! = h
10

That is,
3 5
= h
80
Dr. Raj Kumar, VBS PU Jaunpur. 88
Therefore the error in the Simpsons rule is =

3 5 (4)
h f ()
80

for some suitable (a, b).

Dr. Raj Kumar, VBS PU Jaunpur. 89


Recall

The Newton - Cotes integration rules are all of the form

I(f ) ' A0f (x0) + A1f (x1) + A2f (x2) + + Anf (xn)

Also, note that the weights Ais do not depend on the given function.

Hence, if the error is of the form E(I) = Const f (r+1)(). Then the

rule must be exact for all polynomials of degree r.

Remark: In these quadrature the points xi are fixed.

Ques: Can we improve the accuracy by choosing some suitable xi

Ans: Using Gaussian Quadrature rule one can improve the accuracy.

Dr. Raj Kumar, VBS PU Jaunpur. 90


Example

Find x0, x1, A0, A1 and so that the following rule is exact for all

polynomials of degree 3.

Z 1
f (x)dx = A0f (x0) + A1f (x1) + f (4)()
1

(There are 4 unknowns and hence we have chosen the 4-th derivative

in the error term.)

Taking f (x) = 1, x, x2, x3 we get:

A0 + A1 = 2

Dr. Raj Kumar, VBS PU Jaunpur. 91


A0 x 0 + A1 x 1 = 0

2
A0x20 + A1x21 = 3

A0x30 + A1x31 = 0

On solving these equations we get:

A0 = A1 = 1 x0 = 13 and x1 = 1 .
3

R1
Thus the integration rule is: 1
f (x)dx = f ( 13 ) + f ( 13 ) + f (4)().

Now if we take f (x) = x4 we get

2 2
= + 4!
5 9
Dr. Raj Kumar, VBS PU Jaunpur. 92
1 8 1
= = ( )=
4! 45 135

Thus the expected integration rule is:

Z 1
1 1 1 (4)
f (x)dx = f ( ) + f ( ) + f ().
1 3 3 135

Dr. Raj Kumar, VBS PU Jaunpur. 93


In general

Giving a positive integer n, we wish to determine 2n + 2 numbers

x0, x1, xn and A0, A1, , An so that the sum

I(f ) ' A0f (x0) + A1f (x1) + A2f (x2) + + Anf (xn),

Z b
provides the exact value of f (x)dx for f (x) = 1, x, x2, x2n+1.
a
Or What we want is that the quadrature rule is exact for all polynomials

of degree 2n + 1.

Remark: Here we have to solve system of nonlinear equations, which is

some time is not an easy job.

Dr. Raj Kumar, VBS PU Jaunpur. 94


Orthogonal Functions

Two functions f, g : [a, b] R are said to be orthogonal if

Z b
f (x)g(x)dx = 0.
a

If
Z b
w(x)f (x)g(x)dx = 0
a

for some function w(x) > 0, x [a, b] then we say that f and g are

orthogonal with respect to w(x)

We call the function w(x) called as weight function.

Dr. Raj Kumar, VBS PU Jaunpur. 95


In general

Let w : [a, b] (0, ) be given.

Functions f1(x), f2(x), f3(x), defined on [a, b] is said to orthogonal

w.r. to the weight function w(x) if


Z b
w(x)fm(x)fn(x)dx = 0 if m 6= n
a

and

Z b
w(x)fm(x)fn(x)dx 6= 0 if m = n
a

Dr. Raj Kumar, VBS PU Jaunpur. 96


Gauss Quadrature Method says that:

Theorem:

Rb
To evaluate a
f (x)dx

Rb Rb
1. Suppose we write the integral as a
f (x)dx = a
w(x)g(x)dx for some

positive function w(x).

and

2. Suppose we are able find a sequence of orthogonal polynomials

p0(x), p1(x), p2(x), w.r.to the weight function w(x).

Dr. Raj Kumar, VBS PU Jaunpur. 97


Then

Given any k. If we choose the points x0, x1, x2, , xk as the zeros of
Rb
the polynomial pk+1(x) and the coefficients Ai = a li(x)w(x)dx (i =

0, 1, 2, k) then the integration formula

I(f ) ' A0g(x0) + A1g(x1) + + Ak g(xk )

will be exact for all polynomials of degree 2k + 1

where
k
Y (x xj ) f (x
, g(x) =
(xi xj ) w(x)
j=0,j6=i

Dr. Raj Kumar, VBS PU Jaunpur. 98


with error term

)
f (2k+2)() b
p2k+1(x)
Z
E(I) = w(x) 2 dx (1)
(2k + 2)! a k+1

Where k+1 is the coefficient of xk+1 in pk+1(x).

Dr. Raj Kumar, VBS PU Jaunpur. 99


Legendre Polynomials

[a, b] = [1, 1]

w(x) 1

1 dn 2 n
pn(x) = (x 1) for n = 0, 1, 2, . (Rodriguess Formula)
n!2n dxn
Then p0(x) = 1, p1(x) = x, p2(x) = 21 (3x2 1), .
Z 1
we know pm(x)pn(x)dx = 0 m 6= n and
1 Z
1
2
if m = n, then pm(x)pn(x)dx = .
1 2n + 1

Hence p0, p1, p2 are orthogonal on [1, 1].

Dr. Raj Kumar, VBS PU Jaunpur. 100


Remark

Z b Z 1
The integral f (x)dx can be convert into g(t)dt
a 1
Choose
ba a+b
x= t+
2 2
Then
b 1  
ba ba
Z Z
a+b
f (x)dx = f t+ dt
a 2 1 2 2

Dr. Raj Kumar, VBS PU Jaunpur. 101


Two point Gauss-Legendre Quadrature

The case when we take only two points x0 and x1 in our integration

formula.

Let
Z 1
f (x)dx = A0f (x0) + A1f (x1) + f (4)(), (1, 1).
1

That is the case k = 1 in the above theorem.

Therefore we take the Legendre polynomial of degree 2, that is p2(x) =


1 2
2 (3x 1)

Dr. Raj Kumar, VBS PU Jaunpur. 102


Find the roots of p2(x) = 0

1 1
p2(x) = 21 (3x2 1) = 0 = x =
3
, 3

1 1
Therefore as said in the above theorem we take x0 =
3
and x1 = 3

To fond A0 and A1:

Here w(x) 1.

Thus
1 1
x x1
Z Z
A0 = w(x)l0(x)dx = 1. dx
1 1 x 0 x 1
Z 1
3 1
= (x )dx = 1
1 2 3

Dr. Raj Kumar, VBS PU Jaunpur. 103


1 1
x x0
Z Z
A1 = w(x)l1(x)dx = 1. dx
1 1 x 0 1 x 0
Z 1
3 1
= (x + )dx = 1
1 2 3
Thus the 2-point Gauss-Legendre Quadrature formula takes the form:

Z 1
1 1
f (x)dx ' f ( ) + f ( )
1 3 3

Error Term:

(4) 1
(3x2 1)2 f (4)() 8
Z
f () 1 (4)
E(I) = dx = = f ()
4! 1 9 24 45 135

Dr. Raj Kumar, VBS PU Jaunpur. 104


Three point Gauss-Legendre Quadrature

We use three points x0, x1 and x2 in our integral formula.

This is the case when k = 2

We take p3(x) = 21 (5x3 3x)

Find the roots of p3(x) = 0


q q
p3(x) = 21 (5x3 3x) = 0 = x = 35 , 0, 35
q q
Take x0 = 35 , x1 = 0, x2 = 35

To fond A0, A1 and A2:

Dr. Raj Kumar, VBS PU Jaunpur. 105


1 1
(x x1)(x x2)
Z Z
A0 = w(x)l0(x)dx = w(x).
1 1 (x0 x1)(x0 x2)
q
Z 1 (x 0)(x 35 )
= q q q
3 3 3
1 (
5 0)( 5 5)
Z 1 r
2 3 5
= (x x)dx = .
1 5 9

q q
3 3
Z 1 (x + 5 )(x 5)
A1 = w(x) q q dx
3 3
1 (0 + 5 )(0 5)
Z 1
3 2 8
= (x )dx =
1 5 9

Dr. Raj Kumar, VBS PU Jaunpur. 106


q
Z (x + 35 )(x 0)
1
5
A2 = q q q dx =
1 ( 3
+ 3
)( 3
0) 9
5 5 5

Error Term:
f (6)() 1 ( 25 x3 32 x)2
Z
E(I) = 5 2 dx
6! 1 (2)
(6) Z 1
f () 3
= (x3 x)2dx
6! 1 5
1
= f (6)().
15750
Thus the 3-point Gauss-Legendre quadrature formula takes the form:
Z 1
5 3 8 5 3
f (x)dx = f ( ) + f (0) + f ( ) + E(I)
1 9 5 9 9 5

Dr. Raj Kumar, VBS PU Jaunpur. 107


Example

Using 3-point Gauss-Legendre quadrature, evaluate the integral


R 1 x sin x
1 1+x2
dx

Solution:
Z 1
x sin x 5 3 8 5 3
2
dx = f ( ) + f (0) + f ( )
1 1 + x 9 5 9 9 5
5 0.5417744 8 5 0.5417744
= [ ] + [0] + [ ] = 0.37623
9 0.599999 9 9 1.599999

Dr. Raj Kumar, VBS PU Jaunpur. 108


Remark

Z b Z 1
The integral f (x)dx can be convert into g(u)du
a 1
Choose
ba a+b
x= u+
2 2
Then
b 1  
ba ba
Z Z
a+b
f (x)dx = f u+ du
a 2 1 2 2

Dr. Raj Kumar, VBS PU Jaunpur. 109


Example

R2
Suppose we want to integrate 0
f (x)dx using 3-point Gauss-Legendre

formula

Let
Z 2
f (x)dx = A0f (x0) + A1f (x1) + A2f (x2) + f (6)()
0

for some (0, 2).

We use the change of variable: x = u + 1

Then x = 0 = u = 1 and x = 2 = u = 1.

Dr. Raj Kumar, VBS PU Jaunpur. 110


Then we get
Z 2 Z 1
f (x)dx = f (u + 1)du
0 1

If we write g(u) = f (u + 1), we get

Z 1
g(u)du ' A0g(u0) + A1g(u1) + A2g(u2)
1

q q
Hence u0 = 35 , u1 = 0, u2 = 35

q
3
Therefore x0 = u0 + 1 = 1 5, x1 = u1 + 1 = 0 + 1 = 1 and x2 =
q
u2 + 1 = 1 + 35

Dr. Raj Kumar, VBS PU Jaunpur. 111


Thus r r
Z 2
5 3 8 5 3
f (x)dx ' f (1 ) + f (1) + f (1 + )
0 9 5 9 9 5

Dr. Raj Kumar, VBS PU Jaunpur. 112


Recall

Theorem:

Rb
To evaluate a
f (x)dx

Rb Rb
1. Suppose we write the integral as a
f (x)dx = a
w(x)g(x)dx for some

positive function w(x).

and

2. Suppose we are able find a sequence of orthogonal polynomials

p0(x), p1(x), p2(x), w.r.to the weight function w(x).

Dr. Raj Kumar, VBS PU Jaunpur. 113


Then

Given any k. If we choose the points x0, x1, x2, , xk as the zeros of
Rb
the polynomial pk+1(x) and the coefficients Ai = a li(x)w(x)dx (i =

0, 1, 2, k) then the integration formula

I(f ) ' A0g(x0) + A1g(x1) + + Ak g(xk )

will be exact for all polynomials of degree 2k + 1

where
k
Y (x xj ) f (x)
li(x) = , g(x) =
(xi xj ) w(x)
j=0,j6=i

Dr. Raj Kumar, VBS PU Jaunpur. 114


with error term

)
f (2k+2)() b
p2k+1(x)
Z
E(I) = w(x) 2 dx (2)
(2k + 2)! a k+1

Where k+1 is the coefficient of xk+1 in pk+1(x).

Property:

Each pk (x) has has k distinct real zeros all lie in [a, b].

Dr. Raj Kumar, VBS PU Jaunpur. 115


Chebyshev Polynomials

[a, b] = [1, 1]

1
w(x) =
1x2

We know already know that p0(x) = 1, p1(x) = x, Now pn(x) is

recursively defied as

pk+1(x) = 2xpk (x) pk1(x)

For k = 1, 2, 3, . We have p2(x) = 2x2 1, p3(x) = 4x3 3x .

Observation The coefficient of xk+1 in pk+1 is 2k+1.

Dr. Raj Kumar, VBS PU Jaunpur. 116


Note that the Chebyshev Polynomials given above form an sequence
1
of orthogonal polynomials w.r.to the weight function w(x) =
1x2

Hence we can use these polynomials to evaluate integrals of the form

Z 1
1
f (x) dx
1x 2
1

Dr. Raj Kumar, VBS PU Jaunpur. 117


Gauss-Chebyshev Quadrature 2-point formula

Z 1
f (x)
To evaluate dx
1x 2
1

1
Noted above: w(x) = .
1x2

We want to derive 2-point formula, and hence this is the case when

k = 1 in the main theorem.

Therefore we take p2(x) = 2x2 1.

Dr. Raj Kumar, VBS PU Jaunpur. 118


To find the roots of p2(x) = 2x2 1 = 0 :

1 1
= x = ,
2 2

Therefore the two-point Chebyshev forula will be of the form:

1
1
Z
f (x) 1
dx ' A0f ( ) + A1f ( )
1x 2 2 2
1

Dr. Raj Kumar, VBS PU Jaunpur. 119


To find A0 and A1.

1 1
x x1
Z Z
1
A0 = w(x)l0(x)dx = ( )dx
1 1 1 x x0 x1
2
Z 1 Z 1
1 2x 1 1
= dx + dx
2 2 1 1 x 2 2 1 1 x 2

=0+ .
2
Z 1 Z 1
1 x x0
A1 = = w(x)l1(x)dx = ( )dx
2 1 1 1x 1 2 x x 0
q
1
Z 1
1 x + 2
= ( 2 )dx = .
1 1x 2 2
2

Dr. Raj Kumar, VBS PU Jaunpur. 120


Thus,
1
1
Z
f (x) 1

dx ' f ( ) + f ( ).
1 1x 2 2 2 2 2

The Error term is: substitute k = 1in (2) we get:

(4) b
p2(x)2
Z
f () 1
E(I) = dx
4! a 1x 2 4

f (4)()
= 3
2 4!

Dr. Raj Kumar, VBS PU Jaunpur. 121


3-point Gauss-Chebyshev Quadrature

This is the case when k = 2.

Take p3(x) = 4x3 3x

Roots of p3(x) = 0:

3 3 3
4x 3x = 0 = x = 0, 2 , 2


3 3
Thus x0 = 0 x1 = 2 and x2 = 2

Dr. Raj Kumar, VBS PU Jaunpur. 122


Therefore the Three-point Chebyshev forula will be of the form:

1

3
Z
f (x) 3
dx ' A0f ( ) + A1f (0) + A2f ( )
1 1x 2 2 2

To find A0, A1 and A2:


1 1
(x 0)(x 23 )
Z Z
1
A0 = w(x)l0(x)dx = dx
2
1 x ( 2 0)( 2 23 )
3 3
1 1
Z 1 2 3
(x 2 x)
= dx = .
1 1 x3 3

Dr. Raj Kumar, VBS PU Jaunpur. 123


Similarly,

A1 = A2 = .
3

Thus,
1

3
Z
f (x) 3
dx ' [f ( ) + f (0) + f ( )].
1 1x 2 3 2 2

The Error Term: Take k = 2 in (2).

1
f (6)() (4x3 3x)2
Z
1
E(I) = 2
dx
6! 1 1x 2 4

1
f (6)() (16x6 24x4 + 9x2) f (6)()
Z
= dx = 5 .
16 6! 1 1x 2 2 6!

Dr. Raj Kumar, VBS PU Jaunpur. 124


Example

Using three-point Gauss-Chebyshev quadrature formula, evaluate the


Z 1
(1 + x)ex
integral dx
1x 2
1

Solution:

1 x

3
Z
(1 + x)e 3
dx = [f ( )+f (0)+f ( )] = [0.7574258+1+3.24338]
1 1 x2 3 2 2 3

= 5.23683

Dr. Raj Kumar, VBS PU Jaunpur. 125


Laguerre Polynomials

[a, b] = (0, )

w(x) = ex
n
d
pn(x) = ex n (xnex) for n = 0, 1, 2, .
dx
weight function w(x) = ex.

Therefore, we can use these polynomials to evaluate the integrals of


Z
the form: exf (x)dx
0

Dr. Raj Kumar, VBS PU Jaunpur. 126


First few Laguerre Polynomials

p0(x) = 1

p1(x) = (x 1)

p2(x) = x2 4x + 2

p3(x) = x3 + 9x2 18x + 6

Dr. Raj Kumar, VBS PU Jaunpur. 127


2-point Gauss- Laguerre Rule

Take k=1 and proceed in the similar as in Gauss-Legendre case, we

can prove that

f (4)()
Z
x 1
e f (x)dx = [(2+ 2)f ((2 2))+(2 2)f ((2+ 2))]+ .
0 4 6

Remrark: In the similar way one can derived three-point Gauss-

Laguerre Rule.

Dr. Raj Kumar, VBS PU Jaunpur. 128


Example

R ex sin(2x)
Evaluate using 2-point Gauss-Laguerre quadrature. 0 1+x2
dx

sin(2x)
Solution: Here f (x) = 1+x2


ex sin(2x)
Z
1
dx ' [(2 + 2)f (2 2) + (2 2)f (2 + 2)]
0 1 + x2 4

1
' [3.4142 f (0.585786) + 0.585786 f (3.4142)]
4
= 0.5915

Dr. Raj Kumar, VBS PU Jaunpur. 129


Hermite Polynomials

[a, b] = (, )

x2
w(x) = e
n
n x d 2
x2
pn(x) = (1) e (e ) for n = 0, 1, 2, .
dxn
x2
Since w(x) = e , therefore this orthogonal sequence of polynomials
Z
2
can be used to evaluate integrals of the form ex f (x)dx.

Dr. Raj Kumar, VBS PU Jaunpur. 130


First few Hermite Polynomials

Example: P0(x) = 1,

P1(x) = 2x,

P2(x) = 4x2 2,

P3(x) = 8x3 12x

Dr. Raj Kumar, VBS PU Jaunpur. 131


Three -point Gauss-Hermite Quadrature Rule

This is the case when k = 2.

We take p3(x), the Hermite polynomial of degree 3.

To find the roots of p3(x) = 0 :


q q
8x3 12x = 0 = x = 32 , 0, 32
q q
Hence x0 = 32 , x1 = 0, and x3 = 32

After computing A0, A1, A2, the three-point Gauss-Hermite Rule takes

the form:

Dr. Raj Kumar, VBS PU Jaunpur. 132


Z r r
x2 3 3
e f (x)dx ' [f ( ) + 4 f (0) + f ( )]
6 2 2

Error term: Take k = 2 in the equation (2).


f (6)() (8x2 12)2
Z
2
E(I) = ex dx
6! 82
(6)

f () 3 (6)
= = f ().
6! 4 960

Dr. Raj Kumar, VBS PU Jaunpur. 133


Example

Evaluate the integral


Z x2
e
2
dx
1 + x

, using three-point Gauss-Hermite quadrature

1
Solution: Here f (x) = 1+x2
.

q q

I(f ) ' 6 [f ( 32 ) + 4 f (0) + f ( 32 )] = 54 .

Dr. Raj Kumar, VBS PU Jaunpur. 134


Some Properties of orthogonal Sequence of

Polynomials

Let w(x) > 0 for all x.

Let p0(x), p1(x), p2(x), , pk (x), be be a sequence of orthogonal

polynomials w.r.to w(x). Then

Property: 1

If p(x) is any polynomial of degree k then p(x) can be uniquely written

as p(x) = d0p0(x) + d1p1(x) + + dk pk (x)

Dr. Raj Kumar, VBS PU Jaunpur. 135


Property: 2

If p(x) is any polynomial of degree < k then p(x) is orthogonal to pk (x).


Rb
That is a pk (x)p(x)w(x)dx = 0.

Property: 3

Each pk (x) has has k distinct real zeros all lie in [a, b].

Dr. Raj Kumar, VBS PU Jaunpur. 136


Proof of the Main Theorem - Gauss Quadrature

Methods.

Rb
To evaluate I(f ) = a
w(x)f (x)dx Where w > 0.

We want to derive an integration formula of the form:


I(f ) ' A0f (x0) + A1f (x1) + A2f (x2) + Ak f (xk ) (3)

Replace f (x) by pk (x).

That is: f (x) = pk (x) + f [x0, x1, xk , x]k (x)

Dr. Raj Kumar, VBS PU Jaunpur. 137


Where k (x) = (x x0)(x x1) (x xk )

This gives:

Z b
I(f ) = I(pk ) + f [x0, x1, xk , x]k (x)w(x)dx
a

[ For, if we write pk (x) in the Lagranges form

pk (x) = l0(x)f (x0) + l1(x)f (x1) + + lk (x)f (xk )

then
Z b
I(pk ) = pk (x)w(x)dx
a

Dr. Raj Kumar, VBS PU Jaunpur. 138


Z b Z b Z b
= f (x0) l0(x)w(x)dx+f (x1) l1(x)w(x)dx+ +f (xk ) lk (x)w(x)dx
a a a

Hence

I(pk ) = A0f (x0) + A1f (x1) + A2f (x2) + Ak f (xk )

Rb
where Ai = l (x)w(x)dx
a i
for i = 0, 1, 2, , k.]

Next to find the Error Term:

Rb
I(f ) I(pk ) = a
f [x0, x1, xk , x]k (x)w(x)dx

Rb
Suppose that: a
k (x)w(x)dx = 0

Dr. Raj Kumar, VBS PU Jaunpur. 139


Rb
Then I(f ) I(pk ) = a
f [x0, x1, xk , xk+1, x]k+1(x)w(x)dx for any

choice of xk+1.

Rb
Again, suppose a
k+1(x)w(x)dx = 0.

Then by the same reasoning,

Rb
I(f ) I(pk ) = a
f [x0, x1, xk , xk+1, xk+2, x]k+2(x)w(x)dx for any

choice of xk+2.

Hence, in general, if for certain x0, x1, , xk , xk+1, , xk+m the

Dr. Raj Kumar, VBS PU Jaunpur. 140


integral


Rb
(x)(x x ) (x x )w(x)dx = 0

k k+1 k+1+i

a
(4)
f or i = 0, 1, 2, m 1

Then for any choice of xk+m+1


Rb
I(f ) I(pk ) = a
f [x0, x1, xk , xk+1, xk+m+1, x]k+m+1(x)w(x)dx
(5)

Let p0(x), p1(x), p2(x), , pk (x), be be a sequence of orthogonal

polynomials w.r.to w(x). Then

Dr. Raj Kumar, VBS PU Jaunpur. 141


Recall: Property-2 stated above for orthogonal polynomials.


Rb
pk+1(x)q(x)w(x)dx = 0

a
(6)
f or all polynomials of degree k

Again, by Property-3

we can write

pk+1(x) = k+1(x 0)(x 1) (x k )

where 0, 1, k are the k + 1 distinct zeros of pk+1 on the interval

(a, b).

Dr. Raj Kumar, VBS PU Jaunpur. 142


The idea is: If we set xi = i for i = 0, 1, 2, k

and xk+1, xk+2, xk+3, x2k+1 be arbitrary points in (a, b), then (4) and

hence (5) are satisfied for m = k

(xxk+1 )(xxk+2 )(xxk+1+i )


[ This is because, Take in (6) q(x) = k+1 for i =

0, 1, 2, , k 1- are polynomials of degree k. ]

Therefore,

Rb
I(f ) I(pk ) = a
f [x0, x1, xk , xk+1, x2k+1, x]2k+1(x)w(x)dx
(7)

Now to find the Error term in terms of the derivative of f :

Dr. Raj Kumar, VBS PU Jaunpur. 143


Choose xk+1 = 0, xk+2 = 1, xk+3 = 2 x2k+1 = k ,

Then

2k+1(x) = (x x0)(x x1) (x x2k+1)

= (x 0)(x 1) (x k ) (x 0)(x 1) (x k )
 2
pk+1(x)
=
k+1
so that 2k+1(x) is of same sign (in fact, nonnegative) on (a, b).

Hence we can apply mean value theorem for integrals, to get:


Z b
I(f ) I(pk ) = f [x0, x1, xk , xk+1, x2k+1, ] 2k+1(x)w(x)dx
a

for some suitable [a, b].

Dr. Raj Kumar, VBS PU Jaunpur. 144


Z b 2
pk+1(x)
= f [x0, x1, xk , xk+1, x2k+1, ] w(x)dx
a k+1
Z b 2
f (2k+2) pk+1(x)
= w(x)dx
(2k + 2)! a k+1

Dr. Raj Kumar, VBS PU Jaunpur. 145


Tutorial1, B. Tech. Sem VII
Standard form of LPP & graphical solutions

1. Express the following LPP in standard form


(a). Min Z = x1 2x2 + x3 , s.t. 2x1 + 3x2 + 4x3 4; 3x1 + 5x2 + 2x3 7, x1 , x2 0 and x3 is
unrestricted in sign.
Ans: Max (Z) = x1 +2x2 x4 +x5 , s.t. 2x1 3x2 4x4 +4x5 +s1 = 4; 3x1 +5x2 +2x4 2x5 s2 =
7, x1 , x2 , x4 , x5 , s1 and s2 0.
(b). Max Z = x1 + 2x2 + x3 , s.t. x1 + x2 + x3 3; x1 3x2 4, x1 , x2 0.
Ans: Max Z = x1 + 2x2 + x4 x5 , s.t. x1 x2 x4 + x5 + s1 = 3; x1 3x2 + s2 = 4, x1 ,x2 ,
x4 , x5 , s1 and s2 0.

2. Determine all basic feasible solutions of the following system of equations


(a). 2x1 + x2 + 4x3 = 11, 3x1 + x2 + 5x3 = 14. Ans: (3, 5, 0), (9/2, 0, 1/2)
(b). 2x1 + 6x2 + 2x3 + x4 = 3, 6x1 + 4x2 + 4x3 + 6x4 = 2. Ans: (0, 1/2, 0, 0) is repeated thrice
(c). 2x1 + 3x2 + 4x3 = 5, 3x1 + 4x2 + 5x3 = 6. Ans: (0, 1, 2), (1/2, 0, 3/2), (2, 3, 0) are not
BFS.

3. Show that the feasible solution (1, 0, 1) and z = 6 to the system of equation x1 + x2 + x3 = 2,
x1 x2 + x3 = 2, xi s 0 which minimize Z = 2x1 + 3x2 + 4x3 is not basic.

4. Define the following terms


(a): Solution to L.P.P.; (b). Feasible solution; (C) Basic solution; (d): Basic feasible solution (non
degenerate and Degenerate); (e): Optimum basic feasible solution;
(f): Unbounded solution; (g): Alternative optimal solution.

5. Solve the following L.P.P. by graphical method


(a). Max Z = 3x1 + 5x2 s.t. x1 + 2x2 2000, x1 + x2 1500, x2 600, x1 , x2 0
(b): Max Z = x1 + x2 , s.t. x1 + 2x2 2000; x1 + x2 1500, x2 600, x1 , x2 0
(c): Max Z = 8000x1 + 7000x2 ,
s.t. 3x1 + x2 66; x1 + x2 45, x1 20, x2 40, x1 , x2 0
(d): Min Z = 23 x1 + 52 x2 , s.t. x1 + 3x2 3; x1 + x2 2, x1 , x2 0
(e) Max Z = 3x1 + 2x2 , s.t. x1 x2 1; x1 + x2 3, x1 , x2 0
(f) Max Z = x1 + 2x2 , s.t. x1 + x2 1; x1 + 2x2 4, x1 , x2 0
(g) Max Z = 3x1 2x2 , s.t. x1 + x2 1; 2x1 + 2x2 4 and x1 , x2 0
(h) Max Z = x1 + x2 , s.t. x1 x2 0; 3x1 + x2 3 and x1 , x2 0
(i) Max Z = 5x1 + 3x2 , s.t. 3x1 + 5x2 = 15; 5x1 + 2x2 = 10 and x1 , x2 0
(j): Max Z = 2x1 + 3x2 s.t. x1 + x2 1, 5x1 x2 0; x1 + x2 6, x1 5x2 0, x2 x1 1,
x2 3,and x1 , x2 0
(k): Max Z = 3x1 + 2x2 , s.t. 2x1 x2 2; x1 + 2x2 8, x1 , x2 0
(l): Max Z = 3x1 + 2x2 , s.t. x1 3; x1 x2 0, x1 , x2 0
(m): Max Z = x1 + 2x2 , s.t. x2 4; x1 + 2x2 10, x1 + x2 1, x1 , x2 0
(n): Max Z = 3x1 + 9x2 , s.t. x1 + 4x2 8, x1 + 2x2 4, x1 , x2 0
(o): Max Z = 3x1 + 2x2 , s.t. 2x1 + x2 2, 3x1 + 4x2 12, x1 , x2 0
Ans: (a). x1 = 1000, x2 = 500, z = 5500;
(b). Alternative optimal solution exists one solution is x1 = 1000, x2 = 500; Max z = 1500
(c). x1 = 10.5, x2 = 34.5, Max Z = 3255000 (d). x1 = 3/2, x2 = 1/2; Min Z = 7/2.
(e). Unbounded solution (f). Optimal solution is max Z = 4 but Sf is unbounded
(g). No solution (h). Sf = (i). Sf = 20/19, 45/19 (j). x1 = 3, x2 = 3, Max Z = 15
(k) Unbounded solution (l) Unbounded solution; (m). .........; (n). .........(0).

1
Tutorial II, B. Tech. Sem VII
(Formulation of LPP)

1. A company produces 3 products each product has to pass through 3 different operations. The
time taken in each operation is given in the following table. Formulate the problem as an LPP to

Table 1: Time per unit (in minutes)

Operations Products Operation Capacity per day (In minutes)


1 2 3

1 1 0 1 480
2 0 3 2 500
3 2 4 0 360
Profit per unit 4 3 6

maximize the profit assuming that each item manufactured is sold.


Hint: Let x1 , x2 and x3 be the number of units of 3 products Then Max Z = 4x1 + 3x2 + 6x3 s.t.
x1 + x3 480, 3x2 + 2x3 500, 2x1 + 4x2 360, x0i s 0 and integers.

2. Suppose in problem 6 that the manager decides to utilize full capacity of operation 2.
Hint: Let x1 , x2 and x3 be the number of units of 3 products Then Max Z = 4x1 + 3x2 + 6x3 s.t.
x1 + x3 480, 3x2 + 2x3 = 500, 2x1 + 4x2 360, x0i s 0 and integers.

3. (PRODUCTION PROBLEM): In electronic company manufactures two radio models, each on


a separate production line. The daily capacity of the first line is 60 radios and that of the second is
75 radios. Each unit of the first model uses 10 pieces of certain electronics component, whereas each
unit of the second model requires 8 pieces of the same component. The maximum daily availability
of the special component is 800 pieces. The profit per unit of model 1 and 2 is Rs. 40 and Rs. 30
respectively. Determine the optimum daily production of each model.
Hint: Let x1 and x2 be the number of units of first and second models then Max Z = 40x1 + 30x2
s.t. 10x1 + 8x2 800, x1 60, x2 75, x1 , x2 0 and integers.

4. (TRANSPORTATION PROBLEM): A steel company is faced with the problem of transport-


ing coal from three coal mines to four of its steel plants. The amount of coal available in the coal
mines is 500, 700, and 800 metric tons. The amount required at the plants is 300, 500, 500 and
700 metric tons. It is possible to shift from any mine to any plant and the transportation cost per
metric ton from mine i to to plant j is cij . The problem is to determine the amount of coal to
be transported from each mine to each steel plant so that total transportation cost is minimum.
Formulate the problem.
Hint: Let cij be the transportation cost of each unit from Pthe source Si to destination Dj and xij
m Pn Pn
be the quantity
Pm transported from S i to D j then Min f = i=1 c x
j=1 ij ij s.t. j=1 ij = ai , 1
x
i m, i=1 xij = bj , 1 j n and xij 0.

5. (SCHEDULING PROBLEM): Consider the problem of scheduling the weekly production of a


certain item for the next five weeks. The production cost of the item is Rs 120 for the first three
weeks, and Rs. 150 for the last 2 weeks. The weekly demands are 400, 700, 1000, 900 and 1000
units, which must be meet. The plant can produce a maximum of 700 units each week. In addition
the company can employ overtime during the second and third weeks. This increases the weekly
production by an additional 300 units, but the cost of production increased by Rs. 20 per item.

2
Excess production can be stored at a cost of Rs. 6 per item per week. Formulate the problem to
determine the production schedule that minimizes the total cost.

6. A coal company produces coal of two grades I and II, the profit being 4 and 3 units (hundred of
rupees) per ton respectively. The cutting machine, the screens and the washing plant can operate
not more than 12, 10 and 8 hours per day. The grade I needs 3, 3, and 4 hours per ton while grade
II, 4, 3 and 2 hours per ton in these processes. It is necessary to produce one ton of coal daily.
Maximize the profit.
Hint: Let x1 and x2 be the number of units of first and second grades then Max Z = 4x1 + 3x2
s.t. 3x1 + 4x2 12, 3x1 + 3x2 10, 4x1 + 2x2 8, x1 , x2 0 and integers.

7. (STAFF ROBLEM): A transport company has the following minimum daily requirement of
drivers: Drivers report at the beginning of each period and work for 8 consecutive hours. The

Period Clock Time (24-hours Day) Minimum Number of drivers required

1 5 AM9 AM 50
2 9 AM1 PM 65
3 1 PM5 PM 40
4 5 PM9 PM 70
5 9 PM1 AM 30
6 1 AM5 AM 10

company wants to determine the minimum number of drivers to employ so that there will be
sufficient number of drivers available for each period. Formulate the problem.
Hint: Let x1 , x2 , x3 , x4 , x5 and x6 be the number of drivers at the beginning of periods 1, 2, ....6
respectively. Then Max Z = x1 + x2 + x3 + x4 + x5 + x6 s.t. x1 + x2 65, x2 + x3 40, x3 + x4 70,
x4 + x5 30, x5 + x6 10, x6 + x1 50, x0i s 0 and integers.

8. (INSPECTION PROBLEM): A company has two grades of inspectors, I and II, who are to be
assigned for a quality control inspection of an item it produces.

. Availability Rate/Hr. Accuracy Wage/Hr. Penalty/error


(to the company)

Grade I 10 25 98 Rs. 6.00 Rs. 2.00


Grade II 12 20 96 Rs. 4.00 Rs. 2.00

It is required that at least 3200 pieces be inspected per 8 hrs. day. Determine the optimal assignment
of inspectors which will minimize the total cost of inspection.

9. A dairy manager decides that each cow should get at least 15, 20 and 24 units of nutrients A, B
and C respectively. The contents of nutrients A, B and C respectively are 1, 2, 3 units per Kg. In
the feed of variety Two contents of nutrients are 3, 2, 2 units per Kg. respectively. The costs of
two varieties are respectively Rs. 2 and Rs 3. per kg. How much of feed of each variety should be
purchased to feed a cow at a minimum cost.
Hint: Let x1 and x2 be the number of units of first and second varieties then Min Z = 2x1 + 3x2
s.t. x1 + 3x2 15, 2x1 + 2x2 20, 3x1 + 2x2 24, x1 , x2 0.

10. A cold drink company has two bottling plants located at two places G and J. Each plant produces
three brands of cold drink namely A, B and C in amounts 1500, 300, 2000 and 1500, 1000, 5000

3
bottles per day respectively. It is estimated that during July, there will be a demand of 20,000
bottles of A, 40,000 of B and 44,000 of C. The operating cost per day for plants G and J are 600
and 400 units of Rupees. Find how many days each plant will be run in July so as to minimize the
production cost while meeting the demand? Formulate the LPP and solve graphically.

4
Tutorial 3, B. Tech. Sem VII
Simplex, BigM & Two phase methods, Subject teacher: Dr. Raj Kumar

1. Solve by Simplex method:


(a). Max Z = 4x1 + 3x2 + 4x3 + 6x4 s.t. x1 + 2x2 + 2x3 + 4x4 80, 2x1 + 2x3 + x4 60,
3x1 + 3x2 + x3 + x4 80, x1 , x2 , x3 and x4 0
(b): Max Z = 5x1 + 7x2 s.t. x1 + x2 4, 3x1 8x2 24; 10x1 + 7x2 35, and x1 , x2 0
280 20 180 2280
Ans: (a). x1 = 13 , x2 = 0, x3 = 13 and x4 = 13 , Max Z = 13 ; (b). x1 = 0, x2 = 4, Max
Z = 28

2. Solve the following LPP by Two phase or BigM method:


(a). Max Z = 3x1 + 2x2 + x3 + 4x4 s.t. 4x1 + 5x2 + x3 3x4 = 5, 2x1 3x2 4x3 + 5x4 = 7,
x1 + 4x2 + 25 x3 4x4 = 6; x1 , x2 and x3 0
(b): Max Z = x1 +2x2 +3x3 x4 s.t. x1 +2x2 +3x3 = 15, 2x1 +x2 +5x3 = 20, x1 +2x2 +x3 +x4 = 10;
and x1 , x2 , x3 and x4 0
(c): Min f = 3x1 + 2x2 , s.t. x1 + x2 2; x1 + 3x2 3, x1 x2 = 1, x1 , x2 0
(d): Min Z = 23 x1 + 52 x2 , s.t. x1 + 3x2 3; x1 + x2 2, x1 , x2 0
(e): Max Z = 3x1 + 2x2 , s.t. x1 x2 1; x1 + x2 3, x1 , x2 0
Ans: (a). No Solution, (b). x1 = x2 = x3 = 52 , x4 = 0; Max Z = 15; (c). x1 = 3/2, x2 = 1/2;
Min Z = 11/2. (d). x1 = 3/2, x2 = 1/2; Min Z = 7/2. (e). Unbounded solution

3. Solve the following LPP by Two phase or BigM method:


(a). Min Z = 4x1 + 2x2 s.t. 3x1 + x2 27, x1 + x2 21, x1 , x2 0
(b): Max Z = 3x1 + 25 x2 s.t. 2x1 + 4x2 40, 3x1 + 2x2 50, and x1 , x2 0
(c): Max f = 5x1 8x2 s.t. x1 + x2 1, 2x1 x2 2, and x1 , x2 0
(d): Max f = 5x1 8x2 s.t. x1 + x2 1, 2x1 + 3x2 6, and x1 , x2 0 (d): Max Z = 2x1 + 3x2
s.t. x1 + x2 1, 5x1 x2 0; x1 + x2 6, x1 5x2 0, x2 x1 1, x2 3, and x1 , x2 0
Ans: (a). x1 = 3, x2 = 18, Min Z = 48; (b). Unbdd Solution; (d). x1 = 3, x2 = 3, Max Z = 15.

4. Solve the following Linear Programming Problem (LPP) :


(a). Max Z = 2x1 + x2 s.t. x1 + 2x2 10, x1 + x2 6, x1 x2 2, x1 2x2 1, x1 , x2 0
(b). Max Z = x1 x2 + 3x3 s.t. x1 + x2 + x3 10, 2x1 x3 2, 2x1 2x2 + +3x3 0, x1 ,
x2 , x3 0
(c). Max Z = 8x1 + 19x2 + 7x3 s.t. 3x1 + 4x2 + x3 25, x1 + 3x2 + 3x3 50, x1 , x2 , x3 0
(d). Max Z = 2x1 + 4x2 + x3 + x4 s.t. 2x1 + x2 + 2x3 + 3x4 12, 3x1 + 2x3 + 2x4 20,
2x1 + x2 + 4x3 16, x1 , x2 , x3 , x4 0
Ans: (a). x1 = 4, x2 = 2, Z = 10; (b). x1 = 0, x2 = 6, x3 = 4, Z = 6; (c). x1 = 0, x2 = 25 9 ,
x3 = 1259 , Z = 150 (d). x 1 = 0 = x 3 = x 4 , x 2 = 12, Z = 48;

5. Solve the following Linear Programming Problem (LPP) :


(a). Max Z = 107x1 +x2 +2x3 s.t. 14x1 +x2 6x3 +3x4 = 7, 16x1 + 21 x2 6x3 5, 3x1 x2 x3 0,
x1 , x2 , x3 0
(b). Max Z = x1 x2 + x3 + x4 + x5 x6 s.t. x1 + x4 + 6x6 = 9, 3x1 + x2 4x3 + 2x6 = 2,
x1 + 2x3 + x5 + 2x6 = 6, xi 0
(c). Min Z = 6x1 2x2 6x3 s.t. 2x1 3x2 +x3 14, 4x1 +4x2 +10x3 46, 2x1 +2x2 4x3 37,
x1 2, x2 1, x3 0;
Ans: (a). Unbdd solution (b). x1 = 32 , x4 = 25 16 43
3 , x5 = 3 , remaining variables are zero, Z = 3 ;
(c). x1 = 1111 373 208 5327
50 , x2 = 25 , x3 = 25 , Z = 25 ;

6. Solve the following Linear Programming Problem (LPP) by Simplex method:


(a). Max Z = 3x1 + 5x2 s.t. x1 + 2x2 2000, x1 + x2 1500, x2 600, x1 , x2 0

5
(b): Max Z = x1 + x2 , s.t. x1 + 2x2 2000; x1 + x2 1500, x2 600, x1 , x2 0
(c): Max Z = 3x1 + 2x2 + 5x3 , s.t. x1 + 2x2 + x3 430; 3x1 + 2x2 460, x1 + 4x2 420, x1 ,
x2 0
(d) Max Z = x1 + 2x2 , s.t. x1 + x2 1; x1 + 2x2 4, x1 , x2 0
(e): Min Z = x2 3x3 + 2x5 , s.t. 3x2 x3 + 2x5 7, 2x2 + 4x3 12, 4x2 + 3x3 + 8x5 10,
x2 , x3 , x5 0
(f) Min f = 8x1 5x2 , s.t. x1 + x2 1, x1 x2 1, 3x1 + 2x2 6 and x1 1, x2 1
(g): Max Z = 5x1 + 2x2 s.t. x1 + x2 1, x1 x2 1; and x1 , x2 0
(h) Min f = 8x1 5x2 , s.t. x1 + x2 1, x1 x2 1, 3x1 + 2x2 6 and x1 , x2 1
(i): Max Z = 3x1 + 2x2 , s.t. x1 + x2 3; 2x1 + x2 5, x1 , x2 0
(j): Max Z = 3x1 + 2x2 , s.t. 2x1 + x2 10; x1 + 3x2 6, x1 + x2 21, x1 , x2 0
Ans: (a). x1 = 1000, x2 = 500, z = 5500; (b). Alternative optimal solution exists one solution is
x1 = 1000, x2 = 500; Max z = 1500; (c). x1 = 0, x2 = 100; Max Z = 1350; (d). Optimal solution
is max Z = 4 but Sf is unbounded: (e). x2 = 4, x3 = 5, x5 = 0, Min Z = 11, (f). x1 = 1,
x2 = 23 ; Min f = 21 : (g). Unbounded Solution; (h) x1 = 1, x2 = 32 ; Min f = 21 ; (i). x1 = 2, x2 = 1;
Max Z = 8; (j). x1 = 24 2 76
5 , x2 = 5 ; Max Z = 5 ;

6
B.Tech Sem VII, Tutorial 4
(Duality Theory and Sensitivity Analysis) -----Dr. Raj Kumar Soni
1. Write the dual of the following Linear Programming Problem (LPP).
(a) Max Z x1 2 x2 4 x3 3x4 ,

Subjected to: x1 x2 3 x3 x4 9, 3x1 5 x2 2 x3 7 x4 5, x1 3x2 5 x4 8,

x1 , x2 , x3 , x4 0.

(b) Minimise Z x1 x2 x3

S.t.: x1 3x2 4 x3 5, x1 2 x2 3, 2 x2 x3 4, x1 , x 2 0 and x3 is


unrestricted.
2. Write the dual of Max Z x1 2 x2 x3 ,

S. t.: x1 x 2 x3 2, x1 x2 x3 1, 2 x1 x2 x3 2, x1 0, x2 0.
and using duality theory show that maximum Z can not exceed one.
3. Show by inspection that the dual of the Max Z 2 x1 3 x2 5 x3 ,

Subjected to: x1 x2 x3 15, x1 , x2 , x3 0 , is infeasible. What can you say


about the solution of the primal ?.
4. Use duality to solve: Minimise Z 4 x1 6 x2 9 x3 ,

Subjected to: x1 x2 x3 2, 2 x1 x2 3x3 3, x1 , x2 , x3 0, {Ans:

Min Z = 13.5}
5. The following table gives the amounts of two vitamins V1 and V2 present in each unit
of two foods f1 and f2, their minimum daily requirement and cost of each food per unit:
food
Vitamin f1 f2 Minimum daily requirement
V1 2 4 40 units
V2 3 2 50 units
Cost per unit of 3 2.5
food
The problem is to determine the quantities of the two foods f1 and f2 so that the minimum
daily requirement of two vitamins for a person at a minimum cost, assuming that an
intake of vitamins more than the prescribed minimum is not harmful for health. Make the
mathematical modal and write its dual problem. Solve the dual and use it to find the
solution of the diet problem. {Ans 15, 5/2}
7
6. Use dual complex method to solve the following:
(i) Minimise Z 34 x1 6 x 2 18 x3

S.t.: x1 3x3 3, x 2 2 x3 5, x1 , x 2 , x3 0 {Ans. Min Z

= 36; (0, 3, 1)}


(ii) Max imise Z 3x1 2 x 2 , subject to x1 x 2 1,

x1 x 2 7, x1 2 x2 10, x 2 3, x1 , x 2 , 0
{Ans. Max Z = -18; (4, 3)}

7.Given the LPP: Max imise Z 3 x1 5 x 2 , subject to x1 4,


2 x 2 12, 3x1 2 x2 18, x1 , x 2 , 0
(a) Determine the optimal solution.
(b) Determine the optimal solution when the second resource is changed from 12
to 24. {0, 9}
(c) Determine the effect of change in first cost coefficient from 3 to 4 on solution
(b).{No change}
8. Maximize Z 5 x1 5 x 2 13x3 ,

subject to: x1 x 2 3x3 20, 12 x1 4 x 2 10 x3 90, x1 , x 2 , 0

Introducing x4 and x5 as slack variables, Simplex method yields the following optimal
set of equations:
Z 2 x3 5 x 4 100 , x1 x 2 3 x3 x4 20 ,

16 x1 2 x3 4 x4 x5 10
Use sensitivity analysis to investigate the following:
(a) Change the R.H.S. to (b1, b2) = (10, 100).
(b) Change the coefficient of x3 in the objective function as c3 = 8.
(c) Change the coefficient of x1 to (c1, a11, a21) = (-2, 0, 5)
9. Given the LPP: Maximize Z x1 2 x 2 x3 subject to the constraints

3x1 x 2 x3 10, x1 4 x 2 x3 6, x 2 x3 4, x1 , x 2 , x3 0 .
Determine the optimal solution. {Ans: (0, 3/2, 0), max Z= -3}

8
10. In the problem 10 of Exercise II determine the effect of following changes of the
optimal solution:
(a) Profit on second product reduces from 12 to10.
(b) All the three machines are available for 1500 hrs.
11. Find the optimal solution to the LPP: Minimise Z x1 2 x2 x3

S.t.: x1 2 x2 2 x3 4, x1 x3 3, 2 x1 x2 2 x3 2, x1 , x 2 , x3 0 . {Ans: (0, 6,

4), max Z= -8}


Investigate the following using sensitivity analysis:
(a) The third constraint is deleted.
(b) The variable x3 is deleted.

9
B. Tech Sem VII Tutorial 5

Dr. R.K. Soni


1. Iron ore is to be transported from three mines to four steal mills situated in
different cities. Find the minimum cost transportation schedule given the
following cost matrix:
Steel Mills
A B C D Ore available
I 14 56 48 27 13
Mines

II 82 35 21 81 19
III 99 31 71 63 16
Ore required 7 14 21 16
Ans: I1 = 7, I4 = 6, II3 = 19, III2 = 14, III3 = 2, Minimum cost
= 1235.
2. In a flood relief operation, there are four bases of operations Bi from where air
crafts can take relief materials to three targets Ti. Because of the difference in air
crafts, range to target and flying altitudes, the relief material (in tons) per aircraft
from any base that can be delivered to any target differs according to following
table:
T1 T2 T3
B1 8 6 5
B2 6 6 6
B3 10 8 4
B4 8 6 4
The daily sortie capacity of each of the four bases is 150 sorties per day and the
daily requirement of sorties on each target is 200. Find the allocation of sorties
that maximizes the total tonnage over all the targets.
Ans: 11 = 100, 13 = 50, 23 = 150, 31 = 100, 32 = 50, 42 =
150, Maximum tonnage = 4250.
3. Solve the following transportation problem for minimum transportation cost
D1 D2 D3 D4 D5 Availability
O1 20 19 14 21 16 40

10
O2 15 20 13 19 16 60
O3 18 15 18 20 --- 70
Requirement 30 40 50 40 60
Where a dash indicates that it is not possible to transport goods from origin O3 to
destination D5.
Ans: 15 = 40, 23 = 50, 25 = 10, 31 = 30, 32 = 40,
Minimum cost = 2590.
4. A company has four plants producing the same product. Product cost differs from
one plant to another as do the cost of raw materials. There are five regional
warehouses. Sales price at each is different. The sales and production are given in
the following table:
Plant 1 2 3 4 Sales Maximum
Production Cost 15 18 14 13 Price Sales
Raw material Cost 10 9 12 8
Warehouses Transportation Cost
1 3 9 5 4 34 80
2 1 7 4 5 32 110
3 5 8 3 6 31 150
4 7 3 8 2 31 100
5 4 5 6 7 31 150
Capacity 150 200 175 100

Determine the transportation schedule which maximizes the over all profit.
Ans: (Maximum Profit = Rs. 1955)
5. A steel company has three open hearth furnaces and five rolling mills, shipping
cost of steel from furnaces to rolling mills are shown in the following table:
Mills M1 M2 M3 M4 M5 Capacities
Furnaces (in quintal)
F1 4 2 3 2 6 8
F2 5 4 5 2 1 12
F3 6 5 4 7 3 14
Requirement 4 4 6 8 8
(in quintal)

What is the optimal shipping schedule?


Ans: 11 = 4, 12 = 4, 24 = 8, 25 = 4, 31 = 0, 33 = 6, 35 = 4.
6. Solve the following transportation problem for minimum cost starting with the
degenerate basis x12 = 30, x21 = 40, x32 = 20, x43 = 60.

11
D1 D2 D3 Availability
O1 4 5 2 30
O2 4 1 3 40
O3 3 6 2 20
(Ans. 300)
O4 2 3 7 60
Demands 40 50 60

7. There are five persons and five jobs. The cost of performing different jobs by
different persons are given in the following table:

Jobs 1 2 3 4 5
Column
1 5 3 4 7 1
2 2 3 7 6 5
3 4 1 5 2 4
4 6 8 1 2 3
5 4 2 5 7 1
Find an assignment for minimum cost? Ans: (2, 1), (5, 2), (4, 3), (3, 4), (1, 5).
8. A Department head has four tasks to be performed and three subordinates. The
subordinates differ in efficiency. The estimates of the time, each subordinate
would take to perform each task, is given below. How should be allocated to task,
not more than one to each person so as to minimize the total man hour. Which job
is left incomplete at the expiry of this period?
Men
I II III
A 9 26 15
B 13 27 6 Ans: IA, IIIB, IIC,
C 35 20 15 Total Time =35 ; Job D.

D 18 30 20

12
9. There are four sales representatives and four territories. Due to their different
abilities, the yearly sales expected to be carried out by each representative in the
different territories are as follows:
Territories

I II III IV
Salesman
A 42 35 28 21
B 30 25 20 15
C 30 25 20 15
D 24 20 16 12
Find the optimum assignment so as to maximize sales. What happens if salesman
D can not be assigned to territory IV.
(i) AI, BIII, CII, DIV, or (ii) AI, BII, CIII, DIV,

13
B. Tech Sem VII Tutorial-6
SEQUENCING Dr. R. K. Soni

1. A book binder has one printing press, one binding machine and the manuscripts of
a number of different books. The times required to perform the printing and
binding for each book are as follows:
Books 1 2 3 4 5 6
Printing 30 120 50 20 90 110
Binding 80 100 90 60 30 10
Determine the optimal sequence and total elapsed time. Ans: Time---430 Hours.

2. Find the sequence that minimizes the total elapsed time required to complete the
following jobs.
Job No 1 2 3 4 5 6
Machine A 4 8 3 6 7 5
Machine B 6 3 7 2 8 4 Ans: Time---35 Hours

3. Determine the optimal sequence for the five jobs, each of which must go through
the machines A, B and C in the order ABC:
Processing Time (In Hours)
Job No 1 2 3 4 5
Machine A 5 7 6 9 5
Machine B 2 1 4 5 3
Machine C 3 7 5 6 7 Ans: Time---40 Hours.

4. Determine the sequence maximizing the total time required to complete the
following tasks on three machines I, II and III without passing:

Tasks A B C D E F G
Machine

I 3 8 7 4 9 8 7
II 4 3 2 5 1 4 3
III 6 7 5 11 5 6 12 Ans: Time---59 Hours.

5. Solve the following sequencing problem giving the total elapsed time and ideal
times on different machines.
(i)
Jobs 1 2 3 4 5
M1 1 2 5 4 1
M2 2 3 6 7 1 Ans: Sequencing [1, 5, 2, 3, 4]
M3 1 6 3 8 5
M4 10 15 12 9 11
(ii)
Jobs A B C D E
M1 10 12 8 15 16
M2 3 2 4 1 5 Ans: Sequencing [C, A, E, D, B],
M3 5 6 7 4 3 76 Hours
M4 14 7 12 8 10

14
B. Tech Sem VII Tutorial 7 (Theory of Games)

Dr. R.K. Soni


1. Find the saddle point and the value of the game for each of the following two
games.
B B
(a) 8 6 2 8 (b) 4 -4 -5 6
A 8 9 4 5 A -3 -4 -9 -2
7 5 3 5 6 7 -8 -9
7 3 -9 5

2. Find the range for the values for p and q that will render the entry (2, 2) a saddle
point in each of the following games.
B B
(a) 1 q 6 (b) 2 4 5
A p 5 10 A 10 7 q
6 2 3 4 p 6

3. Indicate whether the values of the following games are greater than, less than or
equal to zero.
B B
(a) 1 9 6 0 (b) 3 7 -1 3
A 2 3 8 4 A 4 8 0 -6
-5 -2 10 -3 6 -9 -2 4
7 4 -2 -5

B B
(c) -1 9 6 8 (d) 3 6 1
A -2 10 4 6 A 5 2 3
5 3 0 7 4 2 -5
7 -2 8 4

4. Solve the following games graphically:


B B
(a) 1 3 -3 7 (b) 1 2
A 2 5 4 -6 A 5 6
-7 9
-4 -3
2 1

5. Solve the following game by linear programming:


B
-1 1 1
A 2 -2 2
3 3 -3

15
6. Solve the game of question 4 by linear programming.

7. Consider the game:


1 2 3
(a) 1 5 50 50
A 2 1 1 0.1
3 10 1 10
Verify that the strategies (1/6, 0, 5/6) for player A and (49/54, 5/54, 0) for player
B are optimal and find the value of the game.

8. Use the relation of dominance to solve the rectangular game whose pay-off matrix
to A is as follows:
B B
1 2 3 4 5 6 1 2 3 4
(a) 1 0 0 0 0 0 0 (b)
2 4 2 0 2 1 1 1 3 2 4 0
A 3 4 3 1 3 2 2 A 2 3 4 2 4
4 4 3 7 -5 1 2 3 4 2 4 0
5 4 3 4 -1 2 2 4 0 4 0 8
6 4 3 3 -2 2 2

9. A and B take out one or two matches and guess how many matches opponent has
taken. If one of the player guess correctly then the looser has to pay him as many
rupees as the sum of the number held by both players. Otherwise the pay-off is
zero. Write down the pay-off matrix and obtain the optimal strategies for both
players.

10. Colonel A and his enemy B are trying to take over two strategic locations. The
regiments available for A and his enemy B are 2 and 3 respectively. Both sides
will distribute their regiments between the two locations. Let n1 and n2 be the
number of regiments allocated by A to locations 1 and 2 respectively. Also let m1
and m2 be his enemy allocation to the respective locations. The pay-off matrix of
A is computed as follows. If n1 > m1, he receives m1+1 and if n2 > m2 he receives
m2+1. On the other hand, if n1 < m1 he looses n1+1 and if n2 < m2 he looses n2+1.
Finally, if the numbers of regiments from both sides are the same, each side gets
zero. Formulate the problem as a two person zero-sum game and then solve by
linear programming.

16
B. Tech Sem VII Tutorial 8 (Queuing Problem)

Dr. R.K. Soni


1. A TV repairman finds that the time spent on his jobs has an exponential distribution
with 30 minutes. If he repairs in the order in which they come in, and if the arrival of
sets is approximately Poisson with an average of 10 per 8 hours day. What is the
repairmans expected ideal time each day? How many jobs are ahead of the average set
just brought in?

2. An oil refinery receives crude at an average rate of one tanker per day. The unloading
facility handles on an average 2 tankers per day. With usual assumption of Poisson
arrival and exponential service times. Determine
(i) Average number of customers on the system
(ii) Average time spent by the tanker in the system.

3. A supermarket has two girls singing up sales at the counter. If the service time for each
customer is exponential with mean 4 minutes and if people arrive in a Poisson fashion
at the rate of 10 per hour. Then find
(i) What is the probability of having to wait for service?
(ii) What is expected percentage of time, each girl is ideal?

4. Arrivals at a telephone booth are considered to follow Poisson distribution with an


average time of 10 minutes between an arrival and the next length of a phone call is
assumed to be distributed exponentially with mean three minutes.
(i) What is the probability that a Poisson arriving at the booth shall have to wait?
(ii) What is the average queue length that forms time to time?
(iii) The telephone department will install a second booth when concerned that an arrival
would expect to wait at least 3 minutes for phone. By how much the flow of arrivals
should increase as to justify a second booth?

5. Problems arrive at a computer centre in a Poisson fashion at an average rate of 5 per


day. The rules of the computing centre are that, any man waiting to solve his problem
must and the man whose problem is being solved. If the time to solve a problem with
one man has an exponential distribution with mean time of 1/3 day and if the average
solving time is increase by proportional to the number of the people working on the
problem. Approximate the expected time in the centre for a person entering the line.

6. An insurance company has three claim adjusters people with claim against the company
are found to arrive in a Poisson fashion at an average rate of 20 per 8 hour day. The
amount of time spent needed for an adjuster with a claimant is found to have an
exponential distribution with mean service time 40 minutes. Claimants are processed in
order of their appearance. How many hours a week can an adjuster expects to spend
with claimant?

17
Speaking Correctly

The Sound System of


English
Why English Pronunciation?
There is a need
- To improve intelligibility
- To enhance listening comprehension
There is a dream
-To speak like a native speaker
There is a must
-To become more effective in oral communication
English Language

Spoken : Phonetics

Written : Grammar
Spoken English
Phonology of English
44 Speech Sounds
Organs of Speech
Classification of Speech Sounds
Organs of Speech
The Muscles of the chest, the lungs and the wind pipe
Pulmonic egressive airstream mechanism
Trachea, Larynx : The vocal cords.
Vocal Cords Drawn Wide Apart, Vocal Cords Held
Loosely Together, Vocal Cords held Tightly Together.
The Pharynx, The Lips, The Teeth, The Teeth Ridge, The
Hard Palate, The Soft, Palate, The Uvula, The Tongue,
Active and Passive Articulator
The Organs of Speech
In the production of vowel sounds there is, in the
pharynx and the mouth, no obstruction and no narrowing
of a degree that would cause audible friction . In this way
vowels are essentially a tone or a hum issuing from
Glottis , with the vocal cords normally vibrating.
Consonants are described in terms of their place of
articulation, manner of articulation and whether the vocal
cords are vibrating or not. In this way they can be
Bilabial, Labio-dental, Dental, Alveolar, Post-Alveolar,
Retroflex, Palato-alveolar, palatal, Velar, Uvular, Glottal.
In accordance to manner of articulation they can be
Plosives, Afffricate, Nasal, Roll, Tap, flap, lateral,
Fricatives, Frictionless continuant,semi-vowel.
Phonetic Transcription
No one to one correspondence between spelling and sound.
A letter of alphabet may stand for different sounds or combinations of
sounds, in different words, and conversely, a given sound may be
represented by different letters, or combination of letters, in different words.
An example for the first type is the letter u in the words cut, put, rude,
minute, bury and university. An example of second type may be the k-
sound represented differently in different words : by the letter k in Kit, ck in
rock, c in cut, cc in acclaim, ch in chemistry and qu in queen.
To understand the correct pronunciation of words there arose the need of
International Phonetic Alphabet (IPA), devised by the International Phonetic
Association.
Phonetic transcription is useful to represent the pronunciation of the word,
or a longer sequence, unambiguosly.for ex. Take the word Tortoise and
the transcription given is
It is helpful in comparing different varieties of same language. Thus the
English words last, fast and class are transcribed as
Words with transcription
Sell /sel/ See /si / Set /set/ Sat /st/ Pot /p t/

Pack /pk/ Tell /tel/ Thick / k/ Kill /k l/ Leaf /li f/

Feel /fi l/ Deal /di l/ Rat /rt/ Cat /kt/ Top /t p/

Seat /si t/ Read /ri d/ Thief /i f/ Speak /spi k/ Peck pek/

Sack /sk/ Pat /pt/ Tea /ti / Deaf /def/ Death /de/

Dress /dres/ Two /tu / Pea /pi / Loss /l s/ Wrest /rest/


Consonant sounds of English
initially medially finally
1. /p/ pin spin keep
2. /b/ bin tubs club
3. /t/ tell stick light
4. /d/ day heads laid
5. /k/ keep skill stick
6. /g/ get jugs bag
Consonant sounds of English
(continued)

initially medially finally


7. / / chin reached teach
8. / / joke hinged large
9. /m/ met smoke team
10. /n/ net snake tin
11. // rings sing
12. /l/ lot fling full
Consonant sounds of English
(continued)

initially medially finally


13 /f/ fan soft if
14. /v/ van leaves thief
15. / / thin monthsbath
16.// then clothes bathe
Consonant sounds of English
(continued)

initially medially finally


17. /s/ sip list less
18. /z/ zip loser buzz
19. / / ship rushed hush
20. / / measure rouge
Consonant sounds of English
(continued)

initially medially finally


21. /h/ hot behave
22. /r/ rot try
23. /w/ watt sweat
24. /j/ yatch tune
The following table shows the 24
consonant phonemes found in
most dialects of English
First it is necessary for us to have a closer look at the way
in which vowels differ from consonants.

Then we need to analyze vowels phonetically, it means


according to:

- Tongue position: how high in the mouth is the tongue,


and which part of the tongue is the highest?
- Length: are the vowels long or short?
- Rounding: are the lips rounded or not?
- Nasality: is there free passage of air through the nose?
(although in English there arent any nasal vowels).
Difference from Consonants
Even though all the languages of the world contain both vowels and
consonants, and although almost everybody has some idea of
whether a given sound is a vowel or a consonant in his language,
there is actually more than one way to distinguish between the two
classes of sounds. From a phonetic point of view one way of
distinguishing is by considering which sounds have the highest
degree of obstruction. Although vowels have almost no obstruction,
and some consonants (obstruents, nasals, and the lateral) have a
high degree of obstruction, there is a group of consonants (the
approximants) which would be classified as vowels if this criterion
was used: approximants have no more obstruction than vowels.
This can be seen by comparing the approximant /j/ in yeast /ji:st/
with the vowel /i:/ in east /i:st/.
From a phonological point of view, it is possible to
distinguish between vowels and consonants by testing
which sounds may be the nucleus of a syllable, i.e. the part
of a syllable that cannot be left out. If you consider a
syllable such as /kart/ cart, the initial /k/ may be left
out and we still have a syllable, /art] art, the final /t/ may be
left out and we still have a syllable , /kar/ car. In fact /k/ and
/t/ may both be left out, and the remainder is still a syllable,
/ar/ are. If however you try to leave out the vowel, then
there is no syllable anymore:* /kt/. [a:] is then the sound
that cannot be left out. Compare with yeast whereas
/j/ can be left out, giving /i:st/, /i:/ cant:* /jst/. Syllabicity
seems to be the criterion to determine whether a sound is a
vowel or a consonant.
Vowel sounds of English

(Pure Vowels/Monophthongs)
initially medially finally
1. /i:/ east sheet key
2. /I/ it hit duty
3. /e / end send
4. // and sand
5. /a:/ art heart car
6. / / ox fox
Vowel sounds of English
(continued)
initially medially finally
7. / :/ all ball saw
8. /U/ put
9. /u:/ ooze choose chew
10.// up cup
11./:/ earn learn stir
12. // ago police maker
Vowel sounds of English (continued)
Diphthongs

13. /e / eight straight stay


14. /a / ice mice my
15. / / oak joke slow
16./a / out shout how
17. / / oil boil boy
18. / / ears beard peer
19. /e/ airs paired hare
20. /u/ cured tour
With the previous explanation and the previous diagram, we can
locate vowel production and tongue position in this way:

Front Central Back

High

Mid

Low
Length
As you may have seen, there are two types of /i/ sound in English
placed in two different positions. However for the purpose of
description, what is relevant is not the difference of position but that of
the perceived length of the vowel. Thus it is said that /i:/ is a long vowel
and /I/ is a short one. The same is valid for /u:/ - / /, / :/ - //, / :/ - / /.

Remember: Symbols for long vowels all have a colon.

Phonologically, one can establish the rule such as only long vowels may
be the last sound of a syllable, whereas short vowels are always
followed by at least a consonant. If we take away the final /t/ from court,
/k :/ is a possible syllable (core) whereas /k / could not possibly occur.
(Exceptions from this are the three short vowels that occur in
completely unstressed syllables, /sItI/, /Int /, /swet/ [city, into,
sweater]).
Rounding
Vowels may also be different from each other with respect to rounding. If
you compare /i:/ in /t i:z]/(cheese) with /u:/ in /t u:z/ (choose), you will see
that not only is /i:/ a front vowel and /u:/ a back vowel, but /i:/ is also
unrounded where /u:/ is rounded. When pronouncing /u:/ your lips are
rounded, but when pronouncing /i:/ the corners of the mouth are much
further apart.

/i/ /u:/
Close
Lip-rounding
/e/ /:/
Lips
Spread Open
/ / Lip-rounding

// Neutrally
/a:/
Open
Words Number of syllables

1. receive ( )
2. dictionary ( )
3.understanding ( )
4.because ( )
5.considered ( )
6.mispronounced ( )
7.correct ( )
8.important ( )
Decide how many syllables there are in each
sentence.

Example: They worked all night (4)


1. They were in the living room. ( )
2. Did you need a map? ( )
3. The library's open from Monday to Friday. ( )
4. They walked very fast. ( )
5. All the students worked hard. ( )
6. He's going to start a new business. ( )
7. Gorge came first in the competition. ( )
8. Did you see our watches? ( )
9. Don't tell me that he's asleep. ( )
10.They'd rented it before we got there. ( )
Tutorial- 4 B. Tech. Math-II (Legendre Function)
Dr. Raj Kumar Soni

Note: The series solution of the ODE ( ) + ( + ) = .(A) is called

a Legendres Polynomial ( ) of degree n and defined by

. . ( ) ( ) ( )( )( )
( )= [ + ]
! .( ) . . ( )( )

( )!
Or ( ) = [ / ]
( ) where =
! ! !

1. Express x + 2x + 2x x 3 in terms of Legendres polynomial.

Ans: ( )+ ( )+ ( )+ ( ) ( )

2. Show that ( + ) = ( ) ; |x| 1, |h| 1,


Generating Function*
3. Prove the following: (i) P (1) = 1,
{Hint: Put = 1 in * and then equating coefficients of h }
(ii)P (1) = (1) , {Hint: Put = 1 in * and then equating coefficients of h }
(iii)P (1) = n(n + 1) {Hint: P (x) satisfies (A), then put = 1 }
1
(iv)P (1) = (1) . n(n + 1)
2
(v) P (x) = (1) P (x) and hence explain the nature of P (x) when n is even or odd.
{Hint: Replacing by and by in * and then equating both values }
!
(vi) P (0) = 0, and P (0) = (1) ( !)

{Hint: Put = 0 in * and then equating coefficients of h if n is even and odd}

4. Prove that P (x) = x cos (x 1) d, if n is a positive integer. {Hint: let

= 1 and = (x 1) put these values in


= , provided a >
( )

b and then equating coefficients of h }


5. State and prove the orthogonal properties of Legendres Polynomial P (x) of degree n.
6. Prove the following recurrence relations:
(i) n =( ) ( ) , {Hint: differentiating * w.r.t. h and then equating
coefficients of h }
(ii) = , {Hint: differentiating * w.r.t. h & x , and then equating coefficients of
h }

6
(iii)( + ) =
{Hint: differentiating (i) relation w. r. t. x and then use (ii) }
(iv)( + ) = {Hint: subtracting (ii) relation from (iii) }
(v) ( ) = {Hint: use (ii) and (iv) }
(vi) ( ) = ( + )( ) {Hint: use (i) and (v) }

7. Prove ( ) ( ) = {Hint: use recurrence relation(i)}

8. Prove that ( ) ( ) = {Hint: use recurrence relation(ii)}

9. Prove that ( ) ( ) ( ) = , m0 {Hint: use recurrence relation(v) & (iii)}


( )
10. Prove ( ) ( ) = and hence find the value
( )

of ( ) ( ) {Hint: use recurrence relation(i)}

11. Show that ( )= ( ) Rodrigues formula


!

12. Evaluate (i) ( ) Ans: 0 ; (ii) ( ) Ans: 0


13. Making use of Christoffels expansion P = (2n 1)P + (2n 5)P + (2n

9)P + + 3P 0r P as n is even or odd, Prove that ( ) =n(n+1)

14. Prove that, when |x| 1, |h| 1 , ( + ) ( ) =

7
Tutorial- 5 B. Tech. Math-II (Laplace Transforms)
Dr. Raj Kumar Soni

8
Answers:

9
TUTORIAL-6 Mathematics-II (Fourier Series)
DEPARTMENT OF MATHEMATICS,
VBS Purvanchal University Jaunpur
Dr. Raj Kumar
e ax
Preamble: (i). eax sin bx dx (a sin bx b cos bx) c
a2 b2
ax
(ii) eax cos bx dx 2e 2 (a cos bx b sin bx) c
a b
(iii) u v dx uv1 u ' v2 u '' v3 u ''' v4 ..., u, v are functions of x and dashes denote
differentiation and suffixes denote integration with respect to x

(iv) Fourier series for f ( x) in c x c 2 is f ( x) a0 an cos nx bn sin nx , where
2 n 1 n 1
c 2 c 2 c 2
1 1 1
a0 f ( x) dx , an f ( x ) cos nx dx , bn f ( x) sin nx dx
c
c
c

a0 n x n x
(v) Fourier series for f ( x) in c x c 2l is f ( x) an cos bn sin , where
2 n 1 l n 1 l
c 2l c 2l c 2l
1 1 n x 1 n x
a0 f ( x) dx , an f ( x ) cos dx , bn f ( x) sin dx
l c
l c
l l c
l
a0 n x
(vi) The half range cosine series in 0 xl is f ( x) an cos where
2 n 1 l
n x
l l
2 2
f ( x ) dx , dx , and the half range sine series in is
l 0 l 0
a0 an f ( x) cos 0 xl
l

n x 2
l
n x
f ( x) bn sin where bn f ( x) sin dx
n 1 l l 0 l

1
Q.1 Express f ( x) ( x) in a Fourier series in the interval 0 x 2 .
2
1 1 1
sin nx
Deduce that 1 .... Ans: f ( x)
4 3 5 7 n 1 n
Q.2 Find the
Fourier series to represent the function
f ( x ) | sin x |, x .
2 4 cos 2 x cos 4 x cos 2nx
Ans: | sin x | ...., 2 ...
3 15 4n 1
Q.3 Express f ( x ) | x | , x , as a Fourier series. Hence show
1 1 1 2
that .......................... .
12 32 52 8
4 cos3x cos5 x
Ans: | x | cos x 2 2 ...
2 3 5
Q.4 Find the Fourier series for function f ( x) x x 2 ; x , Hence
2 1 1
show that 1 .........
6 22 32
2
cos x cos 2 x cos 3 x sin x sin 2 x sin 3x
Ans: f ( x) 4 2
2 ... 2 ..
3 1 22 3 1 2 3
Q.5 Expand the function f ( x ) x sin x as a Fourier series in the interval
x .

10
1 cos 2 x cos 3x cos 4 x
Ans: x sin x 1 cos x 2 2 2 2 ...
2 2 1 3 1 4 1
k x 0
Q.6 Find the Fourier series to represent the function f ( x)
k 0 x
1 1 1
. Also deduce that 1 ......... Ans: f ( x) 4k sin x sin 3x sin 5x ...
4 3 5 7 3 5
x , x 0
Q.7 Find the Fourier series of f ( x) which is assumed to
x, 0 x
be periodic with period 2 . Ans: f ( x) 4 cos2 x cos23x ... 4 sin x sin 3x ...
2 1 3 1 3
x 0 x 1
Q.8 Obtain Fourier series for function f ( x)
(2 x) 1 x 2
Ans: 4 cos2 x cos 32 x cos52 x ...
2 1 3 5
Q.9 Find the Fourier series to represent f ( x) x 2 2, 2 x 2.
Ans: x 2 2 2 162 cos x 1 cos x 1 cos 3 x ...
3 2 4 9 2
Q.10 Obtain the half range sine series for f ( x) e x , 0 x 1.
n[1 e( 1) n

Ans: e 2
x
sin n x
n 1 1 n
2 2

Q.11 Express sin x as a cosine series in 0 x .


Ans: sin x 2 4 cos 2 x cos 4 x cos 6 x ...
1.3 3.5 5.7
Q.12 Obtain the half-range sine series for the function f ( x) x 2 in the interval
Ans: f ( x) bn sin n x , where bn 18 (1)n 1

36
(0, 3) . (1) n 1
n 1 3 n n3 3

11
Tutorial 7 (PDEs) B. Tech. II Semester-2015
MATHEMATICS-II

Q.1 Form the partial differential equation by eliminating the arbitrary constants from the following

2 2 2
(a) x y z 1 , (b) ( x h) 2 ( y k ) 2 z 2 c 2 , (c) z ( x 2 a )( y 2 b) .
a2 b2 c2

Q.2 Form the partial differential equation by eliminating the arbitrary functions from the following

ny
(a) z e f ( x y ) , (b) z f ( x ay ) g ( x ay ) , (c) z f ( x iy ) F ( x iy ) ,

(d) f ( x y z , x 2 y 2 z 2 ) 0 .

Q.3 Solve the following PDEs by Lagranges method

(a) p cos( x y ) q sin( x y ) z , (b) yzp xzq xy ,

(c) x ( y 2 z ) p y ( x 2 z ) q z ( x 2 y 2 ) ,

(d) ( z 2 2 yz y 2 ) p ( xy xz ) q xy xz , (e) y 2 p xyq x( z 2 y )

u u u
(f) p 3q 5 z tan ( y 3 x ) , (g) x y z xyz .
x y z

Q.4 Obtain the complete solution of the following PDEs by using standard form I, II, III, IV

(a) p q pq , (b) x 2 p 2 y 2 q 2 z 2 , (c) p 2 q 2 z , (d) ( p 3 q 3 ) 27 z ,

(e) p 2 q 2 x y , (f) z xp yq log pq .

Q.5 Solve the following PDEs by Charpits method

(a) z pq , (b) xp yq pq , (c) ( p 2 q 2 ) y qz (d) z 2 pqxy

(e) z xp yq p 2 q 2 , (f) ( p q )( xp yq ) 1 .

Q.6 Obtain the general solution of ( 2 y 2 z ) p ( y 2 x ) q 4 xy z. Also, find the particular


solution which passes through the straight line z 1, y x .

Q.7 Find the equation of the surface satisfying t 6 x 3 y and containing the two lines
y 0, z 0, y 1, z 1 .

12
Answer:

2 2
z z 2z z z 2z
Q1. (a) z x zx 0 and z y zy 0 ,
x x x 2 y y y 2

(b) z 2 ( p 2 q 2 1) c 2 , (c) pq 4 xyz .

2z 2 2z 2z 2z
Q2. (a) p q nz , (b) a , (c) 0,
y 2 x 2 x 2 y 2

(d) ( y z ) p ( x z ) q x y .

Q3. (a) [ z 2 cot { 1 ( x y ) }, log{cos( x y ) sin ( x y )} x y ] 0 ,


2 8

(b) ( x 2 y 2 , x 2 z 2 ) 0 , (c) ( x 2 y 2 2 z , xyz ) 0 ,

(d) ( x 2 y 2 z 2 , y 2 2 yz z 2 ) 0 , (e) ( x 2 y 2 , yz y 2 ) 0 ,

y
(f) [ y 3 x, e 5 x {5 z tan ( y 3 x )}] 0 , (g) ( x , , xyz 3u ) 0 .
y z

ay
Q4. (a) z ax c, (b) z cx a y b , where b 1 a 2 ,
(a 1)

(c) 4(1 a 2 ) z ( x ay b) 2 , (d) (1 a 3 ) z 2 8( x ay b) 3 ,

(e) z b 2 ( x a ) 3 / 2 2 ( y a ) 3 / 2 , (f) z ax by log ab .


3 3

(ax y ) 2
Q5. (a) 2 z ax 1 y b , (b) az b , (c) z 2 a 2 y 2 ( ax b) 2
a 2

(d) z bx a y1 / a , (e) z ax by a 2 b 2 , (f) 1 a z 2 x ay b .

Q6. ( x y 2 z , x 2 yz ) 0 , z (1 y ) x y 2 x 2 1

Q7. z x 3 y 3 y (1 x 3 ) .

13
Tutorial 8 (PDEs) B. Tech. II Semester-2015
MATHEMATICS-II

Q.1 Solve the following PDEs

2z 2z 4z 4z 4z 4z 4z 4z
(a) 0 , (b) 2 2 0 , (c) 0 ,
x 2 y 2 x 4 x 3y xy 3 y 4 x 4 y 4

(d) ( D 2 DD ' D ' 1) z 0 , (e) ( D 2 2 D ' ) z 0 .

Q.2 Find the general solution of the following PDEs

2z 2z 2
(a) a2 x , (b) ( D 2 6 DD ' 9 D' ) z 12 x 2 36 xy ,
x 2 y 2

2z 2z
(c) cos mx sin ny , (d) 4r 4 s t 16 log( x 2 y ) ,
x 2 y 2

2 2 x 2y
(e) ( D 2 DD ' 2 D' ) z ( y 1)e x , (f) ( D 2 2 DD ' D' ) z e x3 ,

y
(g) ( D 2 DD ' D' 1) z cos ( x 2 y ) e .

Q.3 Classify the equation:

2u 2u u 2u 2u 2u
(a) 0 ,(b) c2 , (c) c2 ,
x 2 y 2 t x 2 t 2 x 2

2 2 2
(d) (1 x 2 ) z 2 xy z (1 y 2 ) z x z 3 x 2 y z 2 z 0 .
x 2 xy y 2 x y

Q.4 Using the methods of separation of the variables, solve

2u u u u
(a) 0 ,(b) 3 2 0 , where u ( x, 0) 4e x
x 2 y x y

2u 2u u 2u
(c) c2 , (d) c2 .
t 2 x 2 t x 2

14
u 2
2 u
Q.5 Solve the heat equation c , where u (0, t ) 0 , u (l , t ) 0 t 0 ,
t x 2
A, when 0 x l
u ( x , 0) { 2 .
l
0, when x l
2

Q.6 Find the temperature distribution u ( x, t ) in a thin rod of length l , if the initial temperature
through the rod is f (x ) the ends x 0 and x l , of the rod are insulated.

u 2 u
Q.7 Solve the heat equation , with the boundary conditions u ( x, 0) 3 sin x ,
t x 2
u (0, t ) 0 and u (1, t ) 0 where 0 x 1 , t 0 .

2u 2u
Q.8 Solve 0 , for 0 x , 0 y , u ( x, 0) x 2 , u ( x, ) 0 ,
x 2 y 2

u (0, y ) u ( , y ) 0 .
x x

Answer: Q1. (a) z 1 ( y x) 2 ( y x) ,

(b) z ( y x) ( y x) x ( y x) x 2 ( y x) ,
1 2 3 4

(c) z ( y x ) ( y x ) ( y ix ) ( y ix ) , (d) z e x ( y ) e x ( y x) ,
1 2 3 4 1 2

h( x (h / 2) y )
(e) z Ae , where A and h are arbitrary constant.

3
Q2. (a) z ( y ax) ( y ax) x ,
1 2 6

(b) z ( y 3 x) x ( y 3x) 10 x 4 6 x 3 y ,
1 2

cos mx sin ny
(c) z ( y ix) ( y ix) ,
1 2
m2 n2

(d) z (2 y x) x (2 y x) 2 x 2 log( x 2 y ) , (e) z ( y 2 x) ( y x) ye x ,


1 2 1 2

x 2y x5
(f) z ( y x ) x ( y x ) e ,
1 2 20

1
(g) z e x ( y ) e x ( y x )
y
sin( x 2 y ) xe ,
1 2 2

15
Q3. (a) Elliptic, (b) Parabolic, (c) Hyperbolic (d) if x 2 y 2 1; Hyperbolic, x 2 y 2 1; Parabolic,
x 2 y 2 1; Elliptic

1 (2 x 3 y )
Q4. (a) u ( x , y ) ( Ae k x k x 2 ky , (b) u ( x , y ) 4 e 2
Be )e

2
n c
t
4 A 1 2 n l
Q5. u ( x, t ) sin e sin nx
n 1 n 4 l

2
n c l
A t 2
Q6. u ( x , t )
2
0 A
n
e
l
cos n x
l
, where A
0

l f ( x ) dx ,
n 1 0

l
2 n x dx ,
A
n

l f ( x ) cos
0
l

2 2
Q7. u ( x , t ) 3 e n t sin n x
n 1

(1) n sinh n( y ) cos nx


Q8. u ( y ) 4 .
3 n 1 n 2 sinh n

16
Tutorial I, B.Tech. Sem II
Differential Equations

1. Define the terms with one example supporting to each: (a) Order and Degree of a differential equation.(b)
Linear, and nonlinear differential equation.
dy d2 y h  dy 2 i 32
2. Find the Order and Degree of (i) y = x + k , (ii) k 2 = 1+ (iii) x2 (d2 y/dx2 )3 +
dx dy dx dx
dx
y(dy/dx)4 + y 4 = 0 ;
3. Find the differential equation of the circles passing through the origin and having their centres on the

I
xaxis. Ans: 2xyy 0 = y 2 x2
4. Find the differential equation corresponding to (i) y = ae3x + bex . (ii) x = a sin(wt + b). (iii) y =

ON
Aex + Bex + C.
d2 y dy dy
5. Show that Ax2 + By 2 = 1 is the solution of x{y + ( )2 } = y .
dx2 dx dx
6. Determine the differential equation whose set of independent solution is {ex , xex , x2 ex }. Ans: y 000 3y 00 +
3y 0 y = 0
7. Formulate the differential equation for y = c(x c)2 Ans: y 03 = 4y(xy 0 2y)
8. Which of the following set of functions are L.I. (a). {ex , xex , sinh x}. (b). {1 + x, 1 + 2x, x2 }, (c).
{sin 3x, sin x, sin3 x}, Ans: (a), (b)
S
9. Solve (i)
dy
dx
1+y 2
= 1+x 2 , Ans: y x = c(1 + xy) ; (ii)
dy
dx
x(2 log x+1)
= sin 2
y+y cos y , Ans: y sin y = x log x + c;
dy
(iii) xy = y + 2, where y(1) = 1,Ans: ey = 9e x(y + 2)2
dx
dy
(iv) = cos(x + y), Ans: x + arcsin(x + y) + cot(x + y) + c = 0
K.
dx
p p
(v) y( 1 x2 )dy + x( 1 y 2 )dx = 0. Ans: 1 x2 + 1 y 2 + c = 0
dy 3 3
(vi) = ex+y + x2 ex +y , Ans: ex + ey + 13 ex + c = 0
dx
(vii) y 0 sin x = y log y, y = 1 when x = /2. Ans: y = 1
dy 2
(viii) = xy + x + y + 1. Ans: log(y + 1) = (x+1) 2 + c.
dx
dy
(ix) tan y = sin(x + y) + sin(x y). Ans: sec y + 2 cos x + c = 0.
dx
dy 2
(x) = x+2y3 (v-separable).
R.

dx
2 2
(xi) (x y )dx + 2xydy = 0. (Homogeneous).
dy
(xii) x = y(log y log x + 1). (Homogeneous).
dx
(xiii) ye (x/y) dx = (xe(x/y) + y)dy. (Homogeneous).
dy
10. Integrate (1 + x2 ) + 2xy 4x2 = 0 and obtain the cubic curve satisfying this equation and passing
dx
through origin.
dy
11. Solve (i) x2 y x3 = y 4 cos x Ans: x3 y 3 = c + 3 sin x.
dx
(ii) y(2xy + ex )dx ex dy = 0, Ans: ex = y(c x)y
(iii) ydx xdy + (1 + x2 )dx + x2 sin ydy = 0, (By Ispection ) Ans: x2 y 1 x cos y = cx
2
(iv) (xy 2 + 2x2 y 3 )dx + (x2 y x3 y 2 )dy = 0, Ans: log( xy ) xy
1
=c
(v) (xy + y)dx + 2(x y + x + y )dy = 0, Ans: 3x y + 6xy + 2y 6 = c
3 2 2 4 2 4 2
3 3
(vi) (y 2 + 2x2 y)dx + (2x3 xy)dy = 0, Ans: 4x1/2 y 1/2 32 x 2 y 2 = c
q 2 2 2
a x y
(vii) xdx+ydy
xdyydx = x2 +y 2
, (Change into polar)Ans: y(1 + x2 ) = tan1 x 4
di
13. Solve the equation L + R i = E0 sin t, where L R and E0 are constants and discuss the case when t
dt
increases indefinitely.
14. A resistance of 100 ohms, an inductance of 0.5 Henry are connected in series with a battery of 20 volts.
Find the current in the circuit as a function of time.
dx y
15. Solve (i) = x+ xy . (Homogeneous )
dy
dr

I
(ii) r sin cos = r2 , Ans: 1 = r(c cos + sin );
d
(iii)dr + (2r cot + sin 2)d = 0.

ON
dx
(iv) y log y + x = log y.
dy
dy
(v) x log x + y = 2 log x.
dx
dy 2 2
(vi) + xy = y 2 ex /2 sin x.Ans: ex /2 = (c + cos x)y;
dx
(vii) [y(1 + 1/x) + cos y]dx + [x + log x x sin y]dy = 0.Ans: y(x + log x) + x cos y = c;
1
(viii) y(x2 y 2 + xy + 1)dx + (x2 y 2 xy + 1)xdy = 0. Ans: xy log(x/y) xy = c;
1
(ix) (1 + xy)ydx + (1 xy)xdy = 0. Ans: log(x/y) = c + xy ;
x3
(x) x2 ydx (x3 + y 3 )dy = 0. Ans: y = c e 3y3 .
S
(xi) (y + 31 y 3 + 21 x2 )dx + 14 (x + xy 2 )dy = 0. Ans: 3x4 y + x4 y 3 + x6 = c.
K.
R.
Tutorial II, B.Tech. Sem II
Linear Differential Equations with Constant Coefficients

Solve the following differential equations


1. (D2 + a2 ) y = cosec x: Ans: y = c1 cos ax + c2 sin ax + (1/a2 ) sin ax log sin ax (x/a) cosax
2. (D3 1) y = (ex + 1)2 ;
1
Ans: y = c1 ex + e 2 x [c2 cos( 3x/2) + c3 sin( 3x/2] 1 + (1/7) e2x + (2/3) xex

I
3. (D2 2kD + k 2 ) y = ekx ;
Ans: y = (c1 + c2 x)ekx + (x2 /2) ekx

ON
d3 y 2
4. dx3
+ 2 ddx2y + 4 dx
dy
+ 8 y = x, Ans: y = c1 e2x + c2 cos (2 x + c3 ) + 1
16 (2x 1)
5. (D3 5D2 + 7D 3) y = e2x cos hx; Ans: y = (c1 + c2 x) ex + c3 e2x (1/8)x2 ex + (1/8)x e3x
6. (D3 + D2 D 1) y = cos 2x; Ans: y = c1 ex + (c2 + c3 x) ex (1/25)(2 sin 2x + cos 2x)
7. (D4 1) y = sin 2x; Ans: y = c1 ex + c2 ex + c3 cos x + c4 sin x + (1/15) sin 2x
8. (D2 4) y = cos2 x; Ans: y = c1 e2x + c2 e2x (1/16)(2 + cos 2x)

9. (D3 + 8) y = x4 + 2 x + 1; Ans: y = c1 e2x + ex ( c2 cos x 3 + c3 sin x 3 ) + (1/8)(x4 x + 1)
S
10. (D4 2D3 + 5D2 8D + 4) y = x2 ;
Ans: y = (c1 + c2 x) ex + c3 cos 2x + c4 sin 2x + (1/4)(x2 + 4x + 11/2)
11. (D2 1) y = cos hx cos x; Ans: y = c1 ex + c2 ex + (2/5) sin hx sin x (1/5) cos hx cos x

12. (D3 + 1) y = e2x sin x + ex/2 sin(x 3/2);
Ans: y = c1 ex + ex/2 [c2 cos (x 3/2) + c3 sin (x 3/2)] (x/6) ex/2 [sin (x 3/2) + 3 cos (x 3/2)]
K.
13. (D2 4D + 4) y = 8 x2 e2x sin 2x; Ans: y = (c1 + c2 x)e2x + e2x (3 sin 2x 2x2 sin 2x 4x cos 2x)
14. (D4 + 2D3 3D2 ) y = 3 e2x + 4 sin x;
Ans: y = c1 + c2 x + c3 ex + c4 e3 x + (3/20) e2x + (2/5)(2 sin x + cos x)
d4 y 2
15. dx4
+ ddx2y + y = a x2 + b ex sin 2x;

Ans: y = c1 ex/2 cos (x 3/2+c2 )+c3 ex/2 cos (x 3/2+c4 )+a x2 2a(1/481) b ex [9 sin 2x+20 cos 2x]
d2 y
16. dx2
y = 1, y (0) = 0 and y () f inite; Ans: y = ex 1
R.

d2 y dy
17. dx2
+ 2 dx + 10 y + 37 sin 3x = 0, y(0) = 3 and dy/dx = 0 at 0; Find y at /2 Ans: y = 1
d2 x
18. + 4x = a sin t cos t; Ans: x = c1 cos 2t + c2 sin 2t 81 at cos 2t
dt2
e3x
19. (D2 2D + 1) y = x2 e3x ; Ans: y = (c1 + c2 x) ex + 4 (x
3 2x + 32 )
Tutorial III, B.Tech. Sem II
Linear Differential Equations of II order with variable coefficients, Simultaneous DEs

1. (i) Solve x2 y2 2x(1 + x)y1 + 2(1 + x)y = x3 , Ans: y = c1 xe2x + c2 x x2 /2.


(ii) Solve y2 cot xy1 (1 cot x)y = ex sin x, Ans: y = c1 e2x (2 sin x + cos x) + c2 ex 12 ex cos x.
(iii) Verify that LHS of (sin x x cos x)y x sin xy 0 + sin xy = x, vanishes when y = sin x and hence
obtain the general solution of the whole equation. Ans: y = c1 x + c2 sin x + cos x.
d2 y dy 2 x2 sin 2x, Ans: y = ex2 (c cos x + c sin x + sin 2x).
(iv) Solve dx 2 4x dx + (4x 1)y = 3e 1 2

I
d y2 dy 1
(v) Solve (1 x2 ) dx 2
2 4x dx (1 + x )y = x, Ans: y = (c cos x
1x2 1
+ c2 sin x + x).
2. (i) Solve (1 + x2 )2 y2 + 2x(1 + x2 )y1 + 4y = 0, Ans: (1 + x2 ) y =c1 (1 x2 ) + 2c x.

ON
2
00 3 5
(ii) Solve cos x y + sin x y1 2 cos x y = 2 cos x, Ans: y = c1 e 2 sin x + c2 e 2 sin x + sin2 x.
2 d2 y dy
(iii) Solve (1 + x) dx2 + (1 + x) dx + y = 4 cos log(1 + x),
Ans: y = c1 cos log(1 + x) + c2 sin log(1 + x) + 2 log(1 + x) sin log(1 + x).
(iv) Solve y 00 + (3 sin x cot x) y 0 + 2 sin2 x y = e cos x sin2 x, Ans: y = c1 ecos x + c2 e2 cos x + 61 e cos x .
3 2/3
(v) Solve y 00 + 11 y 0 + ( 11 14 x62 ) y = 0, Ans: y = (c1 x3 + c2 x2 )e 4 x .
x3 4x 3 6x 3
3. Solve by using method of variation of parameter
(i) y2 + n2 y = sec nx, Ans: y = c1 sin x + c2 cos nx + ( nx ) sin nx + ( n12 ) cos nx log cos nx.
d2 y dy
(ii) dx2
2 dx = ex sin x, Ans: y = c1 + c2 e2x 1/2ex sin x.
S
(iii) x2 dx
2
d y dy 2 x 1 x 1 x
2 + x dx y = x e , Ans: y = c1 x + c2 x + e x e .

(iv) (1 x) dx
2
d y dy 2 x
2 + x dx y = (1 x) , Ans: y = c1 x + c2 e (x
2 + x + 1).
2
d y dy 1 x x 1 x
(v) dx 2 2 dx + y = x3 e , Ans: y = (c1 + c2 x)e 2x e .

dy dy
4. Solve (i) dx t dx
dt + 2 dt 2x + 2y = 3e , 3 dt + dt + 2x + y = 4e
2t
K.
1 2t
Ans: x = 2 e 11 e + c1 e3 t (6/5)t 15 t
, y = 22 e 8 c1 e(6/5)t + c2 et .
d d
(ii) ( dt + 2)x + 3y = 0, 3x + ( dt + 2)y = 2e3t
t
Ans: x = c1 e c2 e 5t 8 e , y = c1 et + c2 e5t + 85 e3t .
3 3t
2
(iii) t2 ddt2x + t dx 2 d2 y dy
dt + 2y = 0, t dt2 + t dt 2x = 0
Ans: x = t (c1 cos log t + c2 sin log t) + t1 (c3 cos log t + c4 sin log t),
y = t (c1 sin log t c2 cos log t) + t1 (c3 sin log t + c4 cos log t).
dy
(iv) dx
dt = ny mz, dt = lz nx, and dt = mx ly
dz

Ans: x + y + z = c1 ,lx + my + nz 2 = c2 , lx + my + nz = c3 .
2 2 2 2 2
R.
Tutorial III, B.Tech. Sem II
Linear Differential Equations of II order with variable coefficients, Simultaneous DEs

1. (i) Solve x2 y2 2x(1 + x)y1 + 2(1 + x)y = x3 , Ans: y = c1 xe2x + c2 x x2 /2.


(ii) Solve y2 cot xy1 (1 cot x)y = ex sin x, Ans: y = c1 e2x (2 sin x + cos x) + c2 ex 12 ex cos x.
(iii) Verify that LHS of (sin x x cos x)y x sin xy 0 + sin xy = x, vanishes when y = sin x and hence
obtain the general solution of the whole equation. Ans: y = c1 x + c2 sin x + cos x.
d2 y dy 2 x2 sin 2x, Ans: y = ex2 (c cos x + c sin x + sin 2x).
(iv) Solve dx 2 4x dx + (4x 1)y = 3e 1 2

I
d y2 dy 1
(v) Solve (1 x2 ) dx 2
2 4x dx (1 + x )y = x, Ans: y = (c cos x
1x2 1
+ c2 sin x + x).
2. (i) Solve (1 + x2 )2 y2 + 2x(1 + x2 )y1 + 4y = 0, Ans: (1 + x2 ) y =c1 (1 x2 ) + 2c x.

ON
2
00 3 5
(ii) Solve cos x y + sin x y1 2 cos x y = 2 cos x, Ans: y = c1 e 2 sin x + c2 e 2 sin x + sin2 x.
2 d2 y dy
(iii) Solve (1 + x) dx2 + (1 + x) dx + y = 4 cos log(1 + x),
Ans: y = c1 cos log(1 + x) + c2 sin log(1 + x) + 2 log(1 + x) sin log(1 + x).
(iv) Solve y 00 + (3 sin x cot x) y 0 + 2 sin2 x y = e cos x sin2 x, Ans: y = c1 ecos x + c2 e2 cos x + 61 e cos x .
3 2/3
(v) Solve y 00 + 11 y 0 + ( 11 14 x62 ) y = 0, Ans: y = (c1 x3 + c2 x2 )e 4 x .
x3 4x 3 6x 3
3. Solve by using method of variation of parameter
(i) y2 + n2 y = sec nx, Ans: y = c1 sin x + c2 cos nx + ( nx ) sin nx + ( n12 ) cos nx log cos nx.
d2 y dy
(ii) dx2
2 dx = ex sin x, Ans: y = c1 + c2 e2x 1/2ex sin x.
S
(iii) x2 dx
2
d y dy 2 x 1 x 1 x
2 + x dx y = x e , Ans: y = c1 x + c2 x + e x e .

(iv) (1 x) dx
2
d y dy 2 x
2 + x dx y = (1 x) , Ans: y = c1 x + c2 e (x
2 + x + 1).
2
d y dy 1 x x 1 x
(v) dx 2 2 dx + y = x3 e , Ans: y = (c1 + c2 x)e 2x e .

dy dy
4. Solve (i) dx t dx
dt + 2 dt 2x + 2y = 3e , 3 dt + dt + 2x + y = 4e
2t
K.
1 2t
Ans: x = 2 e 11 e + c1 e3 t (6/5)t 15 t
, y = 22 e 8 c1 e(6/5)t + c2 et .
d d
(ii) ( dt + 2)x + 3y = 0, 3x + ( dt + 2)y = 2e3t
t
Ans: x = c1 e c2 e 5t 8 e , y = c1 et + c2 e5t + 85 e3t .
3 3t
2
(iii) t2 ddt2x + t dx 2 d2 y dy
dt + 2y = 0, t dt2 + t dt 2x = 0
Ans: x = t (c1 cos log t + c2 sin log t) + t1 (c3 cos log t + c4 sin log t),
y = t (c1 sin log t c2 cos log t) + t1 (c3 sin log t + c4 cos log t).
dy
(iv) dx
dt = ny mz, dt = lz nx, and dt = mx ly
dz

Ans: x + y + z = c1 ,lx + my + nz 2 = c2 , lx + my + nz = c3 .
2 2 2 2 2
R.

You might also like